Lower Extremity 01-22 Flashcards

1
Q

A 46-year-old woman presents with new-onset pain following below knee amputation. Medical history includes a Gustilo IIIB left leg injury and failed limb salvage 3 years ago. The patient reports phantom sensation, pain in her great toe, burning that ascends the limb, and several points along the distal stump that are inappropriately tender.On examination, a well - healed amputation stump without evidence of unstable skin or skin changes consistent with pressure - related trauma is noted.Targeted muscle reinnervation surgery is planned. Which of the following is the most likely evolution of neuropathic pain at 4 weeks and 6 months postoperatively in this patient?

A

The correct response is Option C.

Targeted muscle reinnervation (TMR), as originally described by Dumanian and Kuiken, is a procedure in which sensory and/or mixed nerves are transferred or coapted to motor nerve branches, in an effort to promote organized nerve growth and also to improve prosthetic control. While initially explored for improvement of prosthetic functionality, researchers observed a concomitant reduction in neuropathic and residual limb pain.

Though more research is needed, targeted muscle reinnervation may be most successful in decreasing or preventing pain when performed at the time of the amputation. The natural history of pain following TMR performed secondarily (not at the time amputation) includes a period of immediate relief (nerves are cut proximal to the neuroma), followed by activation of the nerves and increased pain (3 to 6 weeks), and plateau and reduction of pain (6 weeks to 6 months).

Nearly all patients report a decrease in pain and improvement in quality of life. The degree with which the pain is decreased and life is improved may be related to the timing of the operation. Data suggest that the earlier the operation is performed, the better the results, perhaps because of the centralization of the somatic pain response, though more work is needed to elucidate this mechanism.

How well did you know this?
1
Not at all
2
3
4
5
Perfectly
2
Q

A 25-year-old man presents with a comminuted tibia plateau fracture sustained during a self-inflicted gunshot wound. A CT scan is shown. During open reduction and internal fixation of the fracture, a 5-cm gap in the common peroneal nerve is noted. Tendon transfer, nerve repair with grafting, and nerve transfer are planned. Which of the following fascicular nerve transfers is most likely to aid in ankle dorsiflexion?

A) Flexor digitorum longus to soleus
B) Flexor digitorum longus to tibialis posterior
C) Flexor hallucis longus to tibialis anterior
D) Peroneal longus to extensor digitorum longus
E) Soleus to extensor digitorum brevis

A

The correct response is Option C.

Common peroneal nerve repairs tend to have less favorable results than their upper extremity counterparts. Traction injuries showed only good outcomes in 42% of patients, whereas sharp injuries showed good results in 61% of patients. Gunshot wounds, on the other hand, showed only 49% good outcomes due to the blast injury. This patient had damage of his nerve from penetrating trauma from the fracture fragments as well as damage from the blast injury. Because of the size of the defect, primary nerve repair would not be feasible. Nerve grafts less than 6 cm have been found to have a more favorable result than those greater than 6 cm. When combined with blast injury, a favorable result has only been found in 31% of patients.

In addition to nerve repairs, tendon transfers of the posterior tibial tendon have been useful for more immediate dorsiflexion. Fascicular nerve transfers have found some success in dorsiflexion. Of the options given, only transfer from the flexor hallucis longus (tibial nerve) to the tibialis anterior (deep peroneal nerve) would result in dorsiflexion. Although flexor digitorum longus is a common donor from the tibial nerve, innervating the soleus would result in plantar flexion and the soleus is also innervated by the tibial nerve, so it was not injured in the gunshot. Transfer from the flexor digitorum longus to the tibialis posterior (tibial nerve) would plantarflex the foot and not dorsiflex it. Although transfer to the extensor digitorum longus can lead to dorsiflexion, transfer from the peroneal longus (superficial peroneal nerve) to extensor digitorum longus would not result in a functional nerve repair, since the peroneal longus is innervated by the damaged nerve. Finally, transfer from the soleus to the extensor digitorum brevis (EDB, deep peroneal nerve) would not be a nearby transfer, nor would the EDB dorsiflex the ankle.

How well did you know this?
1
Not at all
2
3
4
5
Perfectly
3
Q

A 36-year-old man is evaluated for coverage of a 4 × 3-cm middle-third lower extremity soft-tissue defect with an exposed fracture of the mid tibia. Reconstruction with a proximally based medial hemisoleus muscle flap is planned. Which of the following arteries provides the major contribution to the blood supply of this flap?

A) Anterior tibial
B) Dorsalis pedis
C) Medial sural
D) Peroneal
E) Posterior tibial

A

The correct response is Option E.

The soleus muscle is a bipennate muscle that is located in the superficial posterior compartment of the lower extremity, deep to the gastrocnemius muscle. The soleus muscle has a medial head that originates from the posterior aspect of the tibia and a lateral head that originates from the posterior surface of the fibula. The soleus functions in plantar flexion of the foot in conjunction with the gastrocnemius as they unite to form the Achilles tendon. Branches from the popliteal artery contribute to the blood supply of the proximal soleus muscle.

The soleus muscle flap is a reliable local flap for coverage of moderate-sized soft tissue defects of the middle-third of the lower extremity. It is customarily elevated as a muscle-only flap and covered with a split-thickness skin graft. Branches from the posterior tibial artery contribute to the blood supply of the medial hemisoleus muscle flap, which is more reliable when based proximally.

Branches from the peroneal artery contribute to the blood supply of the lateral hemisoleus muscle.

The medial sural artery is the dominant blood supply to the medial gastrocnemius muscle flap.

The anterior tibial artery and the dorsalis pedis artery do not contribute to the blood supply of the medial hemisoleus flap

How well did you know this?
1
Not at all
2
3
4
5
Perfectly
4
Q

A 51-year-old man presents for evaluation of nerve injury following varicose vein stripping of the left leg. Which of the following findings would be expected with saphenous nerve injury in this patient?

A) Anesthesia around the left medial malleolus
B) Hypersensitivity along the dorsum of the left foot
C) Inability to dorsiflex the left foot
D) Increased insertional activity in the tibialis anterior muscle
E) Numbness near the left lateral heel

A

The correct response is Option A.

Nerve injury is a relatively rare but significant complication of varicose vein stripping. Knowledge of anatomy can help identify which nerve is involved in most injuries. Injury to the saphenous nerve would cause anesthesia over the medial calf and medial malleolus. Injury to the deep peroneal nerve would cause weakness in dorsiflexion and would result in increased insertional activity on electromyography of the tibialis anterior. Hypersensitivity on the dorsal foot or numbness over the lateral heel would come from an injury to the superficial peroneal nerve and the sural nerve, respectively.

How well did you know this?
1
Not at all
2
3
4
5
Perfectly
5
Q

A 20-year-old man presents to the emergency department 6 hours after a bicycle accident with an open tibial fracture. The patient was traveling at 10 miles per hour at the time of the accident. The wound is 5 cm in length, and there is moderate contamination. The fracture is a mid-shaft tibial fracture with moderate comminution, with an associated closed fibula fracture. Which of the following Gustilo classifications is most appropriate for this injury?

A) I
B) II
C) IIIA
D) IIIB
E) IIIC

A

The correct response is Option B.

Though it was never designed to predict treatment, the Gustilo classification has stood the test of time as a highly utilized grading system for lower extremity trauma. It is often used to predict the need for flap coverage, to estimate the risk for osteomyelitis, and to guide antibiotic use.

How well did you know this?
1
Not at all
2
3
4
5
Perfectly
6
Q

A 34-year-old woman presents with a 1-year history of progressive ankle and dorsal foot pain and paresthesias in the first dorsal web space. Electrodiagnostic study is significant for changes in the extensor digitorum brevis muscle. Which of the following nerves is the most likely source of this patient’s symptoms?

A) Deep peroneal
B) Saphenous
C) Superficial peroneal
D) Sural
E) Tibial

A

The correct response is Option A.

Anterior tarsal tunnel syndrome, also known as deep peroneal nerve (DPN) entrapment, is the result of compression of the DPN at the superior border of the inferior extensor retinaculum at the ankle joint and beneath the extensor hallucis longus tendon. Entrapment can occur as a result of wearing tight-fitting shoes or boots. It is important to rule out exertional anterior compartment syndrome or common peroneal nerve entrapment as the cause of symptoms. The nerve can also experience traction injury caused by chronic ankle instability due to ankle sprains.

The DPN travels in the leg between the extensor digitorum longus (EDL) and tibialis anterior and distally between the EDL and extensor hallucis longus just proximal to the ankle before dividing into the lateral and medial branches, 1.3 cm proximal to the ankle joint. The lateral branch innervates the extensor digitorum brevis (EDB) and the tarsometatarsal (TMT) and metatarsophalangeal joints. The medial branch, a sensory branch, travels to the first dorsal web space and has a dorsomedial cutaneous branch to the second toe and a dorsolateral cutaneous branch to the great toe. Entrapment of the medial branch can occur from the extensor hallucis brevis (EHB) tendon, as it travels over the nerve at the first and second TMT joints. Patients typically present with pain along the dorsum of the foot with intermittent numbness radiating to the first dorsal web space. A Tinel sign may be elicited over the superior and inferior retinaculum along the DPN, resulting in tingling over the first dorsal web space of the foot. The EDB can be weak or atrophied. Patients may also report aching and tightness along the ankle joint or numbness at the first dorsal web space when the ankle is placed in plantar flexion with the toe extended. Electrodiagnostics should be ordered to confirm the diagnosis and location of the entrapment.

How well did you know this?
1
Not at all
2
3
4
5
Perfectly
7
Q

A 53-year-old man underwent excision and primary closure with extensive wide subcutaneous undermining for squamous cell carcinoma of the leg 7 days ago. Examination shows margins are positive, and reexcision resulting in a wound with exposed tendon with no peritenon is noted. Which of the following is the most appropriate reconstructive option for this patient?

A) Bipedicled advancement flap
B) Keystone flap
C) Mustardé rotational flap
D) Split-thickness skin graft
E) V-Y advancement flap

A

The correct response is Option A.

The appropriate reconstructive option from the given choices for this patient would be bipedicled advancement flap. It is important to understand the blood supply to local skin flaps. The bipedicled advancement flap is a random pattern flap with perfusion from the dermal/subdermal plexus. Unlike a standard, single-pedicle random pattern flap, the bipedicled flap maintains skin connections in two directions allowing for a 2:1 length-to-width ratio. The donor defect of the bipedicled flap is typically skin grafted to minimize opposing tension on the flap and allow for greater advancement. The wide subcutaneous undermining 7 days prior would have allowed for the “delay phenomenon,” potentially making the flap more reliable.

The extensive wide subcutaneous undermining during initial primary closure likely injured perforators to the skin. The V-Y advancement flap is typically islandized during closure and would be compromised by adjacent wide undermining. Similarly, a keystone flap would be compromised if the skin was delaminated from the underlying tissues. An adjacent perforator flap is perfused by a cutaneous perforator (septocutaneous or muscular) which likely would have been injured during wide undermining. Split-thickness skin graft is not appropriate because there is no peritenon on the tendon. A Mustardé rotational flap is used for cheek advancement.

How well did you know this?
1
Not at all
2
3
4
5
Perfectly
8
Q

A 25-year-old man is brought to the emergency department with a large degloving injury with exposed tibia and ankle joint (Gustilo Type IIIB) sustained during a motorcycle collision. After multiple debridements, the plastic surgeon is consulted 6 days later. Routine laboratory testing shows a platelet count of 1.5 million/mL, increased from normal range on admission. Free flap reconstruction is planned. Which of the following is the most appropriate next step in management in this patient?

A) Consult hematology for bone marrow biopsy
B) Delay reconstruction until platelet count is within normal range
C) Initiate antiplatelet therapy
D) Proceed with free tissue transfer

A

The correct response is Option C.

Antiplatelet therapy should be initiated to minimize the risk for flap loss in this patient. This patient has a reactive thrombocytosis which can commonly occur following trauma, infection, inflammation, or major surgery (e.g., post-splenectomy). This temporary elevation in platelet count is usually transient and peaks at approximately 2 weeks after injury. The traditional pharmacologic agents for microvascular thrombosis have minimal (e.g., aspirin) or no anti-platelet effects (e.g., heparin, hirudin, and thrombolytic agents). There are no guidelines or standards of care, but Hollenbeck et al. advocate for administration of glycoprotein IIb/IIIa antagonists (e.g., abciximab, tirofiban, and eptifibatide), which block the final common pathway for platelet aggregation. Thienopyridines (e.g., clopidogrel) often reach steady state after 4 to 7 days, which would further delay surgery. Platelet apheresis has also been described to reduce platelet count and secondary flap salvage.

Acute trauma patients with elevated preoperative platelet counts are at higher risk for both intraoperative and postoperative microvascular complications (especially arterial thrombosis). Proceeding with free tissue transfer without any precautions is high risk for failure.

A hematology consult and bone marrow biopsy is not necessary since the hypercoagulable state is usually self-limited but can take several weeks. This is not a chronic medical condition requiring further workup or treatment.

Multiple studies suggest that patients with complex open fractures requiring soft-tissue coverage have improved outcomes with early free flap coverage, so it is not appropriate to wait until after this condition resolves. The platelet elevations can peak at approximately 2 weeks, which would delay reconstruction and increase morbidity. Delaying reconstruction increases the risks for infection, nonunion, and ultimately failure of limb salvage.

How well did you know this?
1
Not at all
2
3
4
5
Perfectly
9
Q

A patient presents with a traumatic heel loss. The surgeon decides on an innervated medial plantar artery flap. The medial plantar nerve is a terminal branch of which of the following other nerves?

A) Common peroneal
B) Lateral plantar
C) Saphenous
D) Sural
E) Tibial

A

The correct response is Option E.

The answer is tibial nerve. The sciatic nerve comes off the sacral plexus, then it branches into sural nerve, common peroneal nerve, and tibial nerve. The tibial nerve terminal branches are the medial sural cutaneous nerve, medial plantar nerve, and lateral plantar nerve. The medial plantar nerve innervates the abductor hallucis, flexor digitorum brevis, flexor hallucis brevis, and first lumbrical muscle. The cutaneous branches innervate the skin of the medial 2/3 of the sole of the foot as well as plantar digital toes. There are also terminal branches to the intertarsal and tarso-metatarsal joints. The flap is supplied by the medial plantar artery off the posterior tibial artery. The artery is found between the abductor hallucis and flexor digitorum brevis. It travels along the medial border of the foot and anastomosis with the first plantar metatarsal artery. Next, the perforators run between the abductor hallucis muscle and the plantar aponeurosis to the skin of the instep.

How well did you know this?
1
Not at all
2
3
4
5
Perfectly
10
Q

A 50-year-old man undergoes a total glossectomy for tongue cancer. A microvascular free flap reconstruction is planned with an anterolateral thigh free flap. After a vertical thigh incision is made, no perforators are found between the rectus femoris and vastus lateralis muscles. Exploration of the medial thigh demonstrates a large perforator between the rectus femoris and vastus medialis muscles. Which of the following is the most likely origin of this perforator?

A) Ascending branch of the lateral circumflex femoral artery
B) Common femoral artery
C) Descending branch of the lateral circumflex femoral artery
D) Medial circumflex femoral artery
E) Profunda femoris artery

A

The correct response is Option C.

An adequate perforator is not found on dissection of the anterolateral thigh (ALT) free flap in up to 5% of cases. When an ALT flap perforator traveling between the rectus femoris and vastus lateralis or through the vastus lateralis to supply the anterolateral thigh skin is not found, there is a higher chance of finding a perforator that supplies the anteromedial thigh skin. This perforator, present in about 50% of thighs, most commonly arises from the descending branch of the lateral circumflex femoral artery, via a branch that supplies the rectus femoris muscle. This perforator can take a course through the rectus femoris or between the rectus femoris and vastus medialis. Alternately, one or more anteromedial thigh perforators may arise directly from the superficial femoral artery.

When an ALT perforator cannot be located, the surgeon may salvage the situation by changing to an anteromedial thigh (AMT) free flap rather than exploring another donor site. Another alternative is to harvest a tensor fascial lata free flap, based on the ascending branch of the lateral circumflex femoral artery. The medial circumflex femoral artery, common femoral artery, and profunda femoris artery do not give rise to previously described cutaneous perforator flaps.

How well did you know this?
1
Not at all
2
3
4
5
Perfectly
11
Q

A 63-year-old man presents to the office with a 4 × 4-cm heel defect. A local podiatrist has debrided the calcaneus, and there is healthy granulation tissue present. After extensive discussion with the patient, it is decided to proceed with a distally based sural fasciocutaneous flap for coverage. Which of the following risk factors is associated with the highest rate of flap-related complications?

A) Coronary artery disease
B) Diabetes mellitus
C) Hypertension
D) Obesity
E) Venous insufficiency

A

The correct response is Option E.

A recent meta-analysis of 61 papers showed that venous insufficiency is associated with a nine-fold increase in risk of developing a complication in a distally based sural flap. Other risk factors, such as peripheral vascular disease, diabetes, obesity, and hypertension, have been associated with increased rates of complications previously in the literature, but venous insufficiency is associated with the highest rate of complications. The pooled data from this series showed a 26% complication rate, a 3.2% flap loss rate, and a 15.3% partial flap loss rate. As the design of the sural flap is distally based, the physiology of the flap requires reversed flow through the venous system.

Advanced age is also associated with an increased complication rate and there is literature to suggest a delay procedure in this patient population. Other papers have found smoking to have a higher risk of complications as well. There is still much debate over which patients should undergo a delay procedure or venous supercharging, but in general, high-risk patients with multiple comorbidities should be considered for this additional surgery. Coronary artery disease has not been found to be associated with increased risk of flap-related complications.

How well did you know this?
1
Not at all
2
3
4
5
Perfectly
12
Q

A 56-year-old woman with a traumatic defect of the upper third of the tibia undergoes open reduction and internal fixation with tibial nail. Soft tissue coverage with a gastrocnemius flap is planned. Which of the following arteries provides the dominant blood supply for this flap?

A) Anterior tibial
B) Peroneal
C) Popliteal
D) Posterior tibial
E) Sural

A

The correct response is Option E.

Each head of the gastrocnemius muscle is supplied by the sural artery: either the medial sural or lateral sural artery for medial and lateral gastrocnemius, respectively. The arteries arise from the popliteal artery about 3-4 cm above the head of the fibula and enter the medial and lateral heads of the gastrocnemius at about the level of the head of the fibula. The flap can be rotated to cover soft-tissue defects of the anterior distal aspect of the knee. The flap ranges from 5 to 9 cm in width and from 13 to 20 cm in length. It provides a vascular bed for a skin graft and improves the delivery of oxygen and systemic antibiotics. The other listed arteries do not supply the gastrocnemius muscles.

How well did you know this?
1
Not at all
2
3
4
5
Perfectly
13
Q

A 40-year-old man presents to the emergency department because of severe pain after sustaining a crush injury to the left lower extremity from a forklift. On physical examination, the lower leg is tense and swollen circumferentially. Sensation to the foot is diminished. Distal pulses are palpable. X-ray study does not show any fractures. Which of the following is the most appropriate next step in management?

A) Ace wrap compression
B) CT angiography
C) Emergent fasciotomy
D) MRI
E) Observation and leg elevation

A

The correct response is Option C.

The patient displays the signs and symptoms of acute compartment syndrome, a surgical emergency requiring emergent fasciotomy. Acute compartment syndrome requires prompt diagnosis and expeditious treatment in order to minimize morbidity. Compartment syndrome can occur following a substantial soft tissue crush injury, even in the absence of a fracture, such as in this clinical scenario. Severe pain is usually the presenting complaint. It may be out of proportion to the injury and unresponsive to analgesics. The presence of paresthesias can signify nerve hypoxia from elevated compartment pressures. Pallor, paralysis, and pulselessness are very late signs. Nerve and muscle do not tolerate long periods of ischemia and may undergo irreversible damage if surgical decompression is delayed.

Compartment syndrome is primarily a clinical diagnosis, but measurement of compartment pressures can provide additional information especially if the diagnosis of compartment syndrome is less obvious. If compartment pressures are greater than 30 mmHg or if the differential pressure (difference between diastolic blood pressure and compartment pressure) is less than 30 mmHg, then fasciotomy is recommended.

Observation and leg elevation would not be appropriate management in the setting of acute compartment syndrome. CT angiography would not be indicated in this case, where there is a low suspicion of vascular injury. MRI has been used in the diagnosis of chronic exertional compartment syndrome but has little value in the setting of acute trauma.

How well did you know this?
1
Not at all
2
3
4
5
Perfectly
14
Q

A 35-year-old man presents for evaluation of a laceration to the lateral aspect of the right lower leg 5 cm distal to the knee that he sustained when he fell from a bicycle 2 months ago. Findings on electromyography and nerve conduction studies are consistent with an isolated complete injury of the common peroneal nerve. Which of the following deficits is most likely on physical examination?

A) Dorsiflexion of ankle
B) Plantarflexion of great toe
C) Sensation of lateral foot
D) Sensation of medial foot
E) Sensation of plantar foot

A

The correct response is Option A.

The common peroneal nerve forms as the sciatic nerve bifurcates at the apex of the popliteal fossa. It then follows the medial border of the biceps femoris muscle and tendon. The nerve then passes over the posterior aspect of the fibular head and winds around the neck of the fibula. The common peroneal then divides into the deep and superficial peroneal nerve branches. The deep branch supplies the anterior muscles of the leg, the dorsum of the foot, and the skin of the first web space. The superficial branch supplies the peroneus longus and brevis muscles and the skin on the distal third of the lower leg and dorsum of the foot. Because of its relatively superficial position, the common peroneal nerve is the most commonly injured nerve of the lower extremity. Transection of the common peroneal nerve results in paralysis of all muscles in the anterior and lateral compartments of the leg (dorsiflexors and ankle evertors). This pattern of injury results in the classic picture of a foot drop. The distribution of sensory loss would include the anterolateral leg and dorsum of the foot.

Sensation of the medial foot is from the saphenous nerve and branches of the medial plantar nerve. Lateral foot sensation is provided by the sural nerve. Sensation of the plantar aspect of the foot is from the terminal branches of the tibial nerve (medial and lateral plantar nerves). All of the muscles of plantar flexion of the ankle and toes (i.e. gastrocnemius, soleus, plantaris, and tibialis posterior, flexor hallucis longus, flexor digitorum longus, and the intrinsic plantar foot muscles) are innervated by the tibial nerve.

How well did you know this?
1
Not at all
2
3
4
5
Perfectly
15
Q

When a pedicled sural flap is raised to the heel, which of the following is the origin of the arterial blood supply?

A) Descending genicular artery
B) Lateral sural artery
C) Medial femoral circumflex artery
D) Medial plantar artery
E) Peroneal artery

A

The correct response is Option E.

The reverse sural flap is a fasciocutaneous flap often used for ankle or heel wounds. The blood supply of the flap can be from a median superficial artery or the arterial plexus that travels with the sural nerve; the origin is a lower peroneal perforator located approximately 5 cm proximal to the lateral malleolus.

The lateral sural artery would be the appropriate blood supply for perfusion of a pedicled lateral gastrocnemius flap. The gracilis flap blood supply derives from the medial circumflex artery. The descending genicular artery provides the blood supply of the medial femoral condyle flap. The medial plantar artery is the blood supply for the medial plantar artery flap.

How well did you know this?
1
Not at all
2
3
4
5
Perfectly
16
Q

A 19-year-old man is brought to the emergency department because of an injury to the right heel sustained during a lawn mower accident. After serial debridement is performed, there is a 2 x 2-cm soft tissue defect with exposed calcaneus. Which of the following innervated flaps is most appropriate for coverage of this defect?

A) Anterior lateral thigh flap with anterior femoral cutaneous nerve
B) Medial plantar artery flap with division of posterior tibial nerve
C) Radial forearm flap with superficial branch of the radial nerve
D) Reverse sural artery flap with saphenous nerve
E) Ulnar forearm flap with deep branch of ulnar nerve

A

The correct response is Option B.

Heel reconstruction is a difficult surgical problem with limited local options, relatively poor vascularity in the region, and weight-bearing requirements. Flap options include a variety of local flaps including transposition or rotation flaps, fasciocutaneous flaps (e.g., medial plantar); local muscle flaps (e.g., abductor hallucis, flexor digitorum brevis, and abductor digiti minimi); reversed fasciocutaneous flaps (e.g., sural); and free flaps. Although innervated (and therefore potentially sensate) free flaps can be performed, these are less predictable than local options. From the above answer choices, the best option is a local flap based on the medial plantar artery, which had sensation from the medial plantar nerve, a branch of a division of the posterior tibial nerve. Another advantage of the medial plantar artery flap is that it covers the heel with glabrous skin, which may better be able to withstand weight-bearing. The other options are not correctly matched with flaps and nerves. Correct pairings of flap and cutaneous innervation are:

Anterior lateral thigh flap - lateral femoral cutaneous nerve
Ulnar forearm flap - medial antebrachial cutaneous nerve
Radial forearm flap - lateral antebrachial cutaneous nerve
Reverse sural artery flap - sural nerve

How well did you know this?
1
Not at all
2
3
4
5
Perfectly
17
Q

16-year-old boy with a Gustilo Type IIIB open tibial fracture underwent wound coverage with an anterolateral thigh (ALT) flap including muscle. Which of the following is the most likely muscular complication in this patient?

A) Weakness of knee extension
B) Weakness of knee flexion
C) Weakness of thigh abduction
D) Weakness of thigh adduction
E) Weakness of thigh flexion

A

The correct response is Option A.

An understanding of the actions of donor muscles is necessary when using muscle flaps for reconstruction. A knowledge of the specific muscles, which are included as part of specific flaps also helps one understand what donor deficits may be produced when using a particular flap.

The anterolateral thigh (ALT) flap perforators often traverse part of the vastus lateralis muscle, and the muscle may need to be dissected and therefore can be injured during flap harvest. It is rare that any long-term sequelae, such as weakness, are noted without muscle harvest. If the muscle is taken, as is often necessary for the filling of a dead space, weakness of knee extension may be noted as the vastus lateralis is a large part of the quadriceps muscle of the thigh, which is primarily responsible for knee extension.

Thigh abduction is accomplished by the tensor fascia lata and sartorius muscles. The pectineus, adductor longus and brevis, gracilis, and adductor magnus are responsible for thigh adduction. Thigh flexion is achieved by the pectineus, adductor brevis, and adductor magnus. Knee flexion is done primarily by the hamstrings-semitendinosus, semimembranosus, and biceps femoris muscles.

How well did you know this?
1
Not at all
2
3
4
5
Perfectly
18
Q

A 48-year-old man presents to the emergency department because of spontaneous progressive pain, swelling, cyanosis, and edema of the left lower extremity for the past 24 hours. A photograph is shown. Medical history includes prophylactic inferior vena cava (IVC) filter placement in the setting of prolonged immobilization secondary to traumatic closed head injury sustained 2 years ago. Physical examination shows no dyspnea. Oxygen saturation is 98% on room air. Venous ultrasonography and CT scan show total left deep femoral thrombosis extending into the lower IVC at the indwelling filter. Which of the following is the most appropriate next step in management?

A) Catheter-directed thrombolysis
B) Femoral vein to IVC vascular bypass
C) Isolated extracorporeal membrane oxygenation (ECMO) support to the affected extremity
D) Open thrombectomy
E) Oral anticoagulation

A

The correct response is Option A.

The patient is presenting with extensive acute thrombotic occlusion resulting in clinically evident symptomatic venous insufficiency of the extremity. If the occlusion is left untreated, progressive cyanosis and secondary ischemia followed by gangrene develop. Locally delivered thrombolytic agents via catheter-directed thrombolysis with or without percutaneous transluminal angioplasty is an effective first line of treatment in this scenario where the patient presents within a few days of symptom onset (ie, prior to clot fibrosis) and is not high-risk for bleeding. In patients who are high-risk for bleeding (eg, acute intracerebral hemorrhage, gastrointestinal bleeding), alternative methods of restoring venous outflow include clot retrieval through other percutaneous or open techniques (eg, transluminal aspiration thrombectomy, open inferior vena cava (IVC) thrombectomy with or without temporizing groin arteriovenous fistula creation). Systemic thrombolysis can be considered when other first line therapies are not available but has been associated with high frequency of major bleeding complications in several randomized trials (14% for streptokinase).

Systemic anticoagulation infusion helps prevent progression but does not restore acute compromised ischemic limb secondary to venous outflow obstruction. Oral anticoagulation is not indicated for acute management of a limb-threatening thrombosis. Femoral vein to IVC vascular bypass is not a described procedure for venous insufficiency. Limb-threatening thrombo-occlusive venous insufficiency resulting in a painful swollen blue leg, such as that pictured (also known as “phlegmasia cerulea dolens,” literally “painful blue edema”) was first described with heparin-induced thrombocytopenia. It has also been associated with cancer or life-threatening critical illness. More recently, a growing population of patients are at risk due to unretrieved IVC filters. While IVC filter placement may protect the pulmonary vascular bed, it does not lessen thrombotic predisposition or incidence in the lower extremities, and IVC thrombosis with or without phlegmasia cerulea dolens has been reported to occur in 3 to 30% of patients following IVC filter placement. Filter retrieval following its initial indicated need can lessen secondary thrombotic complications, but data suggest that only a fraction of retrievable filters are later removed. In a systemic review, overall retrieval was 34% with a high percentage of nonretrieval occurring for a variety of reasons, including loss to follow up (particularly in trauma centers), limited life expectancy, and/or unresolved underlying conditions.

How well did you know this?
1
Not at all
2
3
4
5
Perfectly
19
Q

A 62-year-old woman with non-insulin-dependent diabetes mellitus is undergoing lower extremity angiogram to determine her suitability for forefoot reconstruction. Which of the following is the most appropriate therapy for the prevention of contrast-induced nephropathy in this patient?

A) Ascorbic acid
B) Intravenous saline
C) N-acetylcysteine
D) Simvastatin
E) Sodium bicarbonate

A

The correct response is Option B.

Contrast-induced nephropathy (CIN) is a significant problem in patients undergoing procedures that require contrast administration. The mechanism is believed to be an ischemic injury to the renal medulla. It is the third most common cause of hospital-acquired renal failure. Independent of renal failure, the development of even mild CIN is associated with increased rates of morbidity and mortality. The major risk factor for developing CIN is pre-existing renal dysfunction. This is particularly associated with patients with diabetes and those who have a creatinine clearance less than 60. The best method of prevention is appropriate risk stratification, intravenous hydration with normal saline and withholding of nephrotoxic medications. Intravenous fluid hydration with normal saline is the mainstay of practice in the prevention of CIN. It is low-risk, carries few side effects, and is cost-effective. Randomized trials have found intravenous hydration with normal saline to be consistently effective. The administration of intravenous fluids increases intravascular volume, promotes diuresis, diminishes the overall intravascular contrast load and supports vasodilation. Although intravenous administration of sodium bicarbonate has also gained popularity in the prevention of CIN, recent publications have demonstrated mixed results. The use of N-acetylcysteine, statin drugs and ascorbic acid are not recommended for the prevention of CIN.

How well did you know this?
1
Not at all
2
3
4
5
Perfectly
20
Q

A 40-year-old man is diagnosed with a posterior thigh sarcoma. He undergoes resection of the tumor as well as some of the surrounding muscle. Partial sacrifice of the sciatic nerve is required, leaving a 40% circumferential defect and an 11-cm gap between proximal and distal ends. A photograph is shown. Which of the following is the most appropriate method of nerve reconstruction?

A) Mobilization and primary coaptation
B) Polyglycolic acid nerve tube
C) Processed human allograft conduit
D) Saphenous vein graft
E) Sural nerve cable graft

A

The correct response is Option E.

Fundamentals of nerve repair include coaptation in a tension-free manner. If there is any tension, nerve grafts or conduits are indicated. In this clinical scenario, there is a large nerve gap that precludes tension-free primary coaptation, even with extensive proximal and distal mobilization. Therefore, a nerve graft is indicated. Common choices include sural, lateral, or medial antebrachial cutaneous. For the size and length of the defect and the fact that multiple cable grafts would be needed, the sural is the most appropriate choice.

Nerve conduits such as PGA tubes and processed human allograft conduits serve as scaffolds to promote nerve regeneration, although these are typically used for gaps less than 3 cm. Given the distance involved, a sural nerve graft using a grouped fascicular or epineurial repair is the most appropriate choice, although a gap this large is almost certain to leave permanent deficits. Appropriate levels of expectation must be set with the patient.

How well did you know this?
1
Not at all
2
3
4
5
Perfectly
21
Q

An 18-year-old woman comes to the office because of a large osteosarcoma of the distal shaft of the right femur. A 15-cm bone resection is planned, with a resulting large intercalary segmental defect. The overlying skin and soft-tissue is not involved. The patient is very motivated to proceed with limb preservation. Which of the following is the most appropriate option for reconstruction of this defect?

A) Bone allograft
B) Contralateral vascularized fibula free flap
C) Contralateral vascularized fibula free flap with bone allograft
D) Ilizarov bone transportation
E) Ipsilateral pedicled vascularized fibula flap

A

The correct response is Option C.

In a young patient who desires limb preservation after sarcoma resection, a contralateral vascularized fibula free flap with bone allograft (Capanna technique) is the most appropriate option for a large intercalary segmental defect. This involves placing the fibula flap within an allograft construct and bridging both osteotomy sites. There are advantages to using the allograft with the fibula flap, as a fibula flap alone may have difficulty with weight-bearing and potential fracture. In select cases a double barrel configuration can be used; however, in this patient the defect is too large. An ipsilateral pedicled flap would have difficulty reaching this large defect and would still have issues with fractures from weight-bearing. Ilizarov bone transportation can be performed for smaller defects (4 to 6 cm), but not in a defect this large. Finally, bone allograft alone is an option; however, this has a high rate of nonunion (34% versus 8 to 10%).

How well did you know this?
1
Not at all
2
3
4
5
Perfectly
22
Q

A 45-year-old woman who underwent Achilles tendon repair through a posterior midline incision 3 weeks ago develops a postoperative wound infection and subsequent skin necrosis. Physical examination shows a 3 x 3-cm wound directly overlying the Achilles tendon in the absence of peritenon. A fasciocutaneous propeller flap from the medial leg is designed to cover this defect. The septal perforators to this flap run between which of the following structures?

A) Flexor hallucis longus and gastrocnemius
B) Gastrocnemius and soleus
C) Peroneus longus and peroneus brevis
D) Soleus and flexor digitorum longus
E) Tibialis anterior and extensor digitorum longus

A

The correct response is Option D.

This defect may be reconstructed with a posterior tibial artery perforator propeller flap. These vessels emerge between the flexor digitorum longus and the soleus muscle. In one anatomic study, there were three clusters of perforators: 4 to 9 cm, 13 to 18 cm, and 21 to 26 cm from the intermalleolar line. The peroneal artery perforators often arise through the posterior peroneal septum, and the anterior tibial artery perforators are often found between the extensor digitorum longus and the peroneus longus or between the tibialis anterior and the extensor digitorum longus.

How well did you know this?
1
Not at all
2
3
4
5
Perfectly
23
Q

A 27-year-old woman sustains a Grade IIIB degloving injury of the left lower extremity in a motor vehicle collision. Latissimus dorsi free flap placement is planned. Which of the following is the most likely outcome in this patient in terms of donor site morbidity?

A) Decreased seroma formation but increased hematoma formation
B) Inability to maintain sitting-up position when back is not supported
C) Initial decreased shoulder range of motion that improves by one year
D) Permanent loss of external rotation of the shoulder and inability to reach forward

A

The correct response is Option C.

Most studies that demonstrate shoulder weakness and loss of motion show that the loss of function is greatest in the early postoperative period and returns to baseline, or close to baseline, at 1 year or more after surgery.

All studies comparing types of latissimus flaps demonstrate less morbidity with perforator or muscle-sparing flaps as compared to traditional or extended latissimus dorsi (LD) flaps. Lower functional morbidity is observed with more native muscle preserved as is other flaps. This assumes that the muscular branches of the motor nerve to the latissimus are spared.

A recent meta-analysis does show higher functional impairment than expected after latissimus flap transfer. The number of patients who required a change in occupation was less than 10%. This was likely because of difficulty with activities such as ladder climbing, painting overhead, and sustained reach overhead.

The function of the latissimus dorsi muscle is shoulder adduction, extension and internal rotation. Other muscles of the rotator cuff perform similar functions and will assist in compensation for the loss of the latissimus. Patients who do develop weakness report it in activities involving shoulder adduction and internal rotation. Paradoxically, limitations in range of motion are mostly in shoulder flexion and abduction possibly related to tight skin closure and internal scarring.

Donor site seroma formation is particularly problematic, with published rates ranging from 3.9 to 79%.

Core muscles such as rectus abdominis, external oblique, gluteus maximus, medius, and minimus, and erector spinae all contribute to rotation, balance, and stabilization during sitting and standing.

How well did you know this?
1
Not at all
2
3
4
5
Perfectly
24
Q

A 21-year-old man undergoes reconstruction with a free flap. Photographs are shown. This procedure places the patient at risk for claw toe with loss of active flexion of the great toe. The muscle responsible for this functional loss is located in which of the following compartments in the lower leg?

A) Anterior
B) Deep posterior
C) Lateral
D) Superficial posterior

A

The correct response is Option B.

Claw toe or loss of active flexion of the great toe interphalangeal joint can result from harvest of the flexor hallucis longus for free fibula flaps. The flexor hallucis longus is present within the deep posterior compartment of the lower leg and should be resuspended to the interosseus membrane and posterior tibial muscles as needed to maintain proper tension. Physical therapy is initiated after adequate wound healing to maintain the mobility of the great toe and ankle. The deep posterior compartment musculature is composed of the tibialis posterior, the flexor digitorum longus, the flexor hallucis longus, and the popliteus.

The superficial posterior compartment musculature is composed of the gastrocnemius, soleus, and plantaris.

The anterior compartment musculature is composed of the tibialis anterior, the extensor digitorum longus, extensor hallucis longus, and the peroneus tertius.

The lateral compartment musculature is composed of the peroneus longus and brevis muscles.

How well did you know this?
1
Not at all
2
3
4
5
Perfectly
25
Q

A 67-year-old man comes to the office because of nerve deficit of the left lower extremity which occurred after undergoing a femoral-distal bypass 5 days ago. Physical examination shows numbness of the plantar surface of the foot and weakness in plantarflexion. Which of the following nerves is most likely injured in this patient?

A) Femoral
B) Obturator
C) Peroneal
D) Sural
E) Tibial

A

The correct response is Option E.

This patient appears to demonstrate symptoms of a tibial nerve injury. The tibial nerve is a branch of the sciatic nerve. It travels through the popliteal fossa and gives off branches to gastrocnemius, soleus, plantaris, and popliteus muscles. The tibial nerve travels in proximity to the posterior tibial artery. In the leg, it gives off branches to the flexor digitorum longus, tibialis posterior, and flexor hallucis longus. Distally in the foot, it branches to give rise to the medial and lateral plantar nerves, which provide sensation to the plantar surface of the foot. Injury to the tibial nerve results in deficits of plantarflexion, as well as anesthesia to the plantar surface of the foot.

The femoral nerve innervates muscles of the anterior thigh, including the quadriceps group, iliacus, and sartorius. Injury to the femoral nerve results in weakness of leg extension.

The obturator nerve provides innervation to the medial thigh muscles (adductor group), including adductor brevis, longus, and magnus, as well as the gracilis and obturator externus. The cutaneous branch provides sensation of the medial thigh. Injury to the obturator nerve results in weakness in thigh adduction, and sensory deficits in the medial thigh.

The peroneal nerve is divided into superficial and deep branches at the area of the fibular neck. The superficial peroneal nerve supplies the lateral compartment of the leg, giving motor branches to peroneus longus and brevis, as well as sensory contributions to the lateral aspect of the leg. Injury to the superficial peroneal nerve results in anesthesia of the lateral aspect of the leg and weakness in eversion and plantarflexion of the foot. The deep peroneal nerve travels in the anterior compartment of the leg, and gives branches to the tibialis anterior, extensor hallucis longus, and extensor digitorum longus and brevis, as well as peroneus tertius. The sensory distribution of the deep peroneal nerve is in the area of the first web space. Injury to the deep peroneal nerve causes weakness in dorsiflexion of the foot.

The sural nerve travels on the posterior aspect of the leg between the lateral malleolus and calcaneus. It provides sensation to the lateral aspect of the foot, and does not have a motor component. It is commonly sampled in nerve biopsy and used as a source of nerve graft. Injury or sacrifice of the sural nerve would result in numbness of the lateral foot.

How well did you know this?
1
Not at all
2
3
4
5
Perfectly
26
Q

A 52-year-old man presents with a chronic soft-tissue ulcer of the plantar surface of the first metatarsal head. Medical history includes type 2 diabetes mellitus. Examination shows the wound is not infected, and there is no evidence of peripheral vascular disease. Which of the following is the most appropriate initial treatment?

A) Achilles tendon lengthening
B) Creation of a custom-molded shoe insert
C) Hyperbaric oxygen therapy
D) Knee-high total contact casting
E) Metatarsal head resection

A

The correct response is Option D.

For the noninfected, nonischemic, neuropathic diabetic foot ulcer, pressure reduction through offloading measures is of critical importance. The International Working Group on the Diabetic Foot strongly recommends the use of a non-removable knee-high offloading device as first line treatment. This is supported by high-level quality evidence and multiple studies. These non-removable knee-high offloading devices include the use of total contact casts and non-removable knee-high walker devices. Removable offloading devices, such as custom molded and other therapeutic shoe inserts, as well as multiple therapeutic shoe designs, have consistently been shown to be less effective in healing chronic wounds than non-removable devices. This may be largely because of patient non-compliance. Surgical interventions that may decrease plantar pressures, such as Achilles tendon lengthening, metatarsal head resection, and metatarsal-phalangeal joint arthroplasty, may be of less utility and should only be considered when nonsurgical methods have failed. Hyperbaric oxygen therapy may play a significant role in Wagner grade III (bone involvement) or greater diabetic foot wounds in terms of increased healing and decreased amputation rate; however, evidence is lacking to suggest its routine use in soft-tissue-only diabetic foot ulcers.

How well did you know this?
1
Not at all
2
3
4
5
Perfectly
27
Q

A 54-year-old woman sustains an open fracture of the right ankle in a motorcycle collision. Flap coverage of the associated distal-third leg wound is planned. Which of the following is the most significant advantage of using a fasciocutaneous flap instead of a muscle flap?

A) Better fill of dead space
B) Higher flap survival rate
C) Improved clearance of osteomyelitis
D) Less donor site morbidity
E) Quicker dissection

A

The correct response is Option D.

Muscle flaps were “workhorses” for lower extremity reconstruction for years, but harvest of muscle always leaves some donor site functional morbidity because of loss of the muscle function. Survival rates, speed of dissection, and treatment of osteomyelitis are not significantly different between the flap types. Muscle flaps tend to fill dead space easier than fasciocutaneous flaps.

How well did you know this?
1
Not at all
2
3
4
5
Perfectly
28
Q

A 42-year-old man presents with an open tibia fracture sustained during a motor vehicle collision 4 hours ago. Physical examination shows a 3-cm puncture wound at the fracture site, no dirt or debris in the wound, and no exposed bone. X-ray studies show a transverse fracture of the tibia and fibula without comminution. Which of the following is the appropriate initial antibiotic coverage?

A) First generation cephalosporin
B) First generation cephalosporin, aminoglycoside, and penicillin
C) First generation cephalosporin and aminoglycoside
D) Third generation cephalosporin
E) Third generation cephalosporin, aminoglycoside, and penicillin

A

The correct response is Option A.

The Gustillo-Anderson classification system is used to grade open fractures based on the extent of bone and soft tissue injury, and the extent and nature of wound contamination. Aggressive debridement, administration of prophylactic antibiotics, application negative pressure dressing while the wound is open, and early definitive wound coverage (less than 5 days) reduces the infection risk. The open fracture described is a grade II injury and a first-generation cephalosporin alone provides appropriate antibiotic coverage. A concurrent vascular or neural injury or gross contamination could escalate this into a grade III injury, but there is no mention of these factors in the clinical scenario described.

How well did you know this?
1
Not at all
2
3
4
5
Perfectly
29
Q

A 12-year-old boy is referred to a multidisciplinary sarcoma treatment center because of a deep localized rhabdomyosarcoma of the right thigh. After neoadjuvant radiotherapy, radical resection with curative intent, including a 20-cm segmental intercalary resection of involved distal femoral diaphysis, is performed. Skin and major neurovascular structures will be spared. Postoperative chemotherapy is planned. Which of the following is the most appropriate method for management of the bony defect in this patient?

A) Distraction osteogenesis
B) Free fibula transfer with femoral allograft (Capanna technique)
C) Induced membrane (Masquelet) technique
D) Lower extremity rotationplasty (Van Ness procedure)
E) Pedicled medial femoral condyle flap

A

The correct response is Option B.

Rhabdomyosarcomas represent the most common soft-tissue sarcoma of childhood and are responsible for approximately half of all soft-tissue sarcomas in this age group. They are thought to originate from immature cells that are destined to form striated skeletal muscle, although they can arise anywhere in the body. With modern multimodal management, the cure rates for localized disease are generally greater than 70% overall. The primary goal of local tumor control in extremity rhabdomyosarcoma is limb-sparing complete resection where possible.

Vascularized bone grafting represents the gold standard for reconstructing segmental bone loss greater than 6 cm associated with a compromised local soft-tissue environment that occurs with radiotherapy and chemotherapy. For large weight-bearing intercalary reconstruction, significant literature supports the combination of a large structural allograft combined with vascularized fibula as described by Capanna in 1980. With this combination, the neoosteogenic properties of the free fibula are supplemented by the immediate structural support of the bulk allograft and provide a durable single-stage biological reconstruction.

Distraction osteogenesis is a technique of de novo bone formation that capitalizes on normal bone healing with gradual, surgically controlled distraction of adjacent osteotomy defects and has the advantage of simultaneously expanding surrounding soft-tissue envelopes. The technique requires viable bone in proximity to one another following a latency phase and is useful in limb lengthening and craniofacial procedures but has limited utility in long segmental tumor reconstruction.

The induced membrane technique proposed by Masquelet is a two-step procedure where a segment of bone loss is first filled with an acrylic spacer and later replaced by cancellous bone graft in the so-called self-induced reactive “periosteal” membrane. The technique requires two stages and is less favored in the setting of planned radiation or chemotherapy where experience has shown that vascularized flaps or supplemented vascularized allografts are beneficial. The medial femoral condyle flap has been used for small osteoperiosteal, corticoperiosteal, and osteocartilaginous flaps based off either the articular descending genicular or superomedial genicular arteries. It would be insufficient in size for a 20-cm-long bone defect.

The Van Ness rotationplasty is a type of autograft where functional limb below the knee is used to reconstruct more proximal defects. It can be a useful “spare part” reconstructive option in composite proximal extremity resections by repurposing a functional ankle joint more proximally in a rotated configuration for preserved gait advantage at the repurposed knee. A rotationplasty would not be indicated for intercalary resections sparing joint and metaphysis.

How well did you know this?
1
Not at all
2
3
4
5
Perfectly
30
Q

An otherwise healthy 62-year-old woman presents with mild edema, some hemosiderin deposition, and a clean, shallow, painful ulcer about 2 cm in size above the left medial malleolus. Medical history includes a left ankle fracture 15 years ago. She does not smoke cigarettes. She has a job which requires that she stand for 8-hour shifts. Distal pulses are present and ankle brachial index is .94. Which of the following is the most appropriate initial management?

A) Debride the wound and apply a split-thickness skin graft
B) Elevate and apply serial compression dressings (Unna boot)
C) Hyperbaric oxygen therapy (HBOT)
D) Optimize the wound bed with bilaminate neodermis (Integra)
E) Strip the greater saphenous vein and ligate the perforators

A

The correct response is Option B.

Venous insufficiency is staged using the CEAP (clinical, etiologic, anatomical, and pathophysiologic) classification. The patient presented in this scenario meets the criteria for C6 (Clinical 6) criteria with the presence of an active ulcer. Compression and keeping the wound clean are the initial, primary, and mainstay therapies for healing venous ulcers. The only option listed that provides compression and wound care is to clean the wound, elevate, and apply serial compression dressings (Unna boot). After a trial of compression and wound bed optimization, closure can be considered. The literature does not provide conclusive evidence that skin grafting is a superior or desired closure. There are studies that demonstrate the superiority of Apligraf in achieving wound closure. If the perforators are found to be the source of the issue, ligation may reduce the recurrence of ulcers in the area but studies comparing ligation and wound care do not show earlier closure of ulcers present. Hyperbaric oxygen therapy (HBOT) is not indicated in this situation.

How well did you know this?
1
Not at all
2
3
4
5
Perfectly
31
Q

Which of the following conditions is a relative CONTRAINDICATION for use of the flap in the image shown for reconstruction of an 8 x 10-cm anterior ankle wound?

A) Diabetes mellitus
B) Hypertension
C) Joint exposure with loss of the joint capsule
D) Occlusion of the peroneal artery
E) Underlying osteomyelitis

A

The correct response is Option D.

Hypertension does not preclude the use of any fasciocutaneous flaps in the lower extremity.

Diabetes mellitus can be associated with peripheral vascular disease, but by itself, would not prevent successful use of the reverse sural artery flap for foot or ankle reconstruction. Appropriate preoperative workup would include noninvasive ultrasound study of the lower extremity vasculature to prove the peroneal artery was patent.

Vascularized flaps, including the reverse sural artery flap, provide excellent coverage for foot/ankle wounds, including those with underlying osteomyelitis. Effective treatment would necessitate adequate debridement and antibiotic therapy as part of the reconstructive paradigm.

The distally based sural artery flap receives its blood supply from a few sources, the most robust of which are perforators from the peroneal artery. The most distal of these perforators arise between 4 and 7 cm proximal to the lateral malleolus. Additional perfusion arises from neurocutaneous perforators from the sural nerve and venocutaneous perforators from the lesser saphenous vein.

How well did you know this?
1
Not at all
2
3
4
5
Perfectly
32
Q

A 61-year-old man comes to the office for evaluation of a 3 x 3-cm calcaneal defect with exposed bone. Medial plantar flap reconstruction is planned. The principal blood supply to this flap arises from which of the following arteries?

A) Arcuate
B) Dorsalis pedis
C) Peroneal
D) Plantar arch
E) Posterior tibial

A

The correct response is Option E.

The primary blood supply to the medial plantar flap is the medial plantar artery, a terminal branch of the posterior tibial artery. The dorsalis pedis is the continuation of the anterior tibial artery and does not contribute to this flap. The peroneal artery is a proximal branch of the posterior tibial artery and descends in the deep posterior compartment posterior to the tibialis posterior and anterior to the flexor hallucis longus; it does not contribute to this flap. The arcuate artery is the terminal branch of the anterior tibial artery. The plantar arch runs on the plantar aspect of the foot at the level of the metatarsals; it is formed from a confluence of the lateral plantar artery and the deep plantar artery from the dorsalis pedis.

How well did you know this?
1
Not at all
2
3
4
5
Perfectly
33
Q

A 40-year-old man sustains an avulsion of the weight-bearing portion of the medial heel. Coverage with an instep flap is planned. Sensation to this flap is provided by which of the following?

A) Lateral plantar nerve from the deep peroneal nerve
B) Lateral plantar nerve from the superficial peroneal nerve
C) Lateral plantar nerve from the sural nerve
D) Medial plantar nerve from the deep peroneal nerve
E) Medial plantar nerve from the tibial nerve

A

The correct response is Option E.

The medial plantar artery flap, or instep flap, provides sensate, full-thickness glabrous skin and subcutaneous tissue that can be transferred as a pedicled or free flap. The tissue is well suited for weight-bearing areas of the foot but has also been used as a free tissue transfer for palmar defects. Because the instep donor site is non–weight-bearing, the donor site can be covered with a skin graft. The innervation of the medial instep flap comes from the medial plantar nerve, a branch of the tibial nerve.

How well did you know this?
1
Not at all
2
3
4
5
Perfectly
34
Q

A 65-year-old man is referred for evaluation of a 3 x 4-cm wound with exposed tendon over the distal anterior tibia after sustaining fracture to the lateral malleolus, which was successfully treated with cast immobilization. The wound had been managed with local wound care for the past several weeks. Physical examination shows a clean wound with some fibrinous exudate. Periosteum and peritenon are intact. Pulses cannot be palpated. Pencil Doppler signals in dorsalis pedis and posterior tibialis are noted. Which of the following studies is the most appropriate next step in management?

A) Ankle brachial index
B) CT angiography
C) MRA
D) MRI
E) Percutaneous angiography

A

The correct response is Option A.

This patient has a pressure sore from cast immobilization. He also has asymptomatic peripheral vascular disease, as is evidenced from his clinical examination. For the lower extremity to heal, adequate blood flow is required and this can be most effectively quantified with an ankle brachial index measurement. Ankle brachial index less than or equal to 0.9 establishes the presence of peripheral artery disease. Ankle brachial index between 0.5 and 0.79 yields wound healing issues and less than 0.5 results in rest pain and arterial insufficiency.

CT angiography, MRI, MRA, and percutaneous angiography can assist in delineating anatomy but they do not yield clinically helpful information about perfusion, prognosis, or stratification of peripheral artery disease.

How well did you know this?
1
Not at all
2
3
4
5
Perfectly
35
Q

A 50-year-old man comes to the office because of a persistent nonhealing wound 6 months after he underwent open reduction and internal fixation of an open ankle fracture. Examination shows palpable pedal pulse with retained protective sensation of the foot. Which of the following is the most appropriate initial step in management of this patient?

A) Application of collagenase ointment
B) Core needle bone culture
C) Coverage with a free flap
D) Operative debridement
E) Referral for hyperbaric oxygen therapy

A

The correct response is Option D.

The patient is at high risk for fracture nonunion and osteomyelitis. The best next course of management is operative debridement ideally along with the treating orthopedist to make judgments about bone viability and debridement and the risks and benefits of hardware removal. Enzymatic wound debridement would not address the concerns about the deeper wound issues. The role for hyperbaric oxygen in the scenario presented is not well established. Bone cultures at the time of operative debridement should be obtained; but, percutaneous core needle cultures alone would not likely be adequate to obtain best healing. Free flap coverage may be required but is not indicated at this time.

How well did you know this?
1
Not at all
2
3
4
5
Perfectly
36
Q

A 50-year-old man comes to the office because of a persistent nonhealing wound 6 months after he underwent open reduction and internal fixation of an open ankle fracture. Examination shows palpable pedal pulse with retained protective sensation of the foot. Which of the following is the most appropriate initial step in management of this patient?

A) Application of collagenase ointment
B) Core needle bone culture
C) Coverage with a free flap
D) Operative debridement
E) Referral for hyperbaric oxygen therapy

A

The correct response is Option D.

The patient is at high risk for fracture nonunion and osteomyelitis. The best next course of management is operative debridement ideally along with the treating orthopedist to make judgments about bone viability and debridement and the risks and benefits of hardware removal. Enzymatic wound debridement would not address the concerns about the deeper wound issues. The role for hyperbaric oxygen in the scenario presented is not well established. Bone cultures at the time of operative debridement should be obtained; but, percutaneous core needle cultures alone would not likely be adequate to obtain best healing. Free flap coverage may be required but is not indicated at this time.

How well did you know this?
1
Not at all
2
3
4
5
Perfectly
37
Q

A 40-year-old man sustains an avulsion of the weight-bearing portion of the medial heel. Coverage with an instep flap is planned. Sensation to this flap is provided by which of the following?

A) Lateral plantar nerve from the deep peroneal nerve
B) Lateral plantar nerve from the superficial peroneal nerve
C) Lateral plantar nerve from the sural nerve
D) Medial plantar nerve from the deep peroneal nerve
E) Medial plantar nerve from the tibial nerve

A

The correct response is Option E.

The medial plantar artery flap, or instep flap, provides sensate, full-thickness glabrous skin and subcutaneous tissue that can be transferred as a pedicled or free flap. The tissue is well suited for weight-bearing areas of the foot but has also been used as a free tissue transfer for palmar defects. Because the instep donor site is non–weight-bearing, the donor site can be covered with a skin graft. The innervation of the medial instep flap comes from the medial plantar nerve, a branch of the tibial nerve.

How well did you know this?
1
Not at all
2
3
4
5
Perfectly
38
Q

A 65-year-old man is referred for evaluation of a 3 x 4-cm wound with exposed tendon over the distal anterior tibia after sustaining fracture to the lateral malleolus, which was successfully treated with cast immobilization. The wound had been managed with local wound care for the past several weeks. Physical examination shows a clean wound with some fibrinous exudate. Periosteum and peritenon are intact. Pulses cannot be palpated. Pencil Doppler signals in dorsalis pedis and posterior tibialis are noted. Which of the following studies is the most appropriate next step in management?

A) Ankle brachial index
B) CT angiography
C) MRA
D) MRI
E) Percutaneous angiography

A

The correct response is Option A.

This patient has a pressure sore from cast immobilization. He also has asymptomatic peripheral vascular disease, as is evidenced from his clinical examination. For the lower extremity to heal, adequate blood flow is required and this can be most effectively quantified with an ankle brachial index measurement. Ankle brachial index less than or equal to 0.9 establishes the presence of peripheral artery disease. Ankle brachial index between 0.5 and 0.79 yields wound healing issues and less than 0.5 results in rest pain and arterial insufficiency.

CT angiography, MRI, MRA, and percutaneous angiography can assist in delineating anatomy but they do not yield clinically helpful information about perfusion, prognosis, or stratification of peripheral artery disease.

How well did you know this?
1
Not at all
2
3
4
5
Perfectly
39
Q

A 60-year-old man sustains a Gustilo type IIIB open fracture of the distal left tibia during a boating accident. There is severe contamination of the wound, and the patient undergoes multiple formal washouts in the operating room. There is no neurovascular compromise of the extremity. He undergoes external fixation to stabilize the limb. Which of the following is the most appropriate next step in treatment?

A) Coverage with a free tissue transfer
B) Negative pressure wound therapy until secondary healing is achieved
C) Pedicled gastrocnemius muscle and skin grafting
D) Primary bone allografting
E) Split-thickness skin grafting

A

The correct response is Option A.

The Gustilo classification describes open fractures of the tibia by the severity of the soft tissue injury overlying the fracture. In patients with IIIB injuries, there is extensive soft-tissue loss and periosteal stripping, but no vascular compromise requiring repair.

Gustilo classification:
Type I: The wound is less than 1 cm long. There is little soft-tissue damage and no sign of crush injury. There is no or minimal comminution of the fracture.
Type II: The laceration is more than 1 cm long but there is no extensive soft tissue damage, flap, or avulsion. There is slight to moderate crushing injury, moderate comminution of the fracture.
Type III: Extensive damage to the soft tissues, including muscle, skin, and neurovascular structures, and a high degree of contamination.
Type IIIA: Soft tissue coverage of the bone is adequate.
Type IIIB: Extensive injury to or loss of soft tissue, with periosteal stripping and exposure of bone, massive contamination, and severe comminution of the fracture from high-velocity trauma
Type IIIC: Any open fracture with a vascular injury requiring repair.

Free tissue transfer will bring healthy, nontraumatized tissue into the area to cover the exposed broken bone. Multiple recent studies have shown equivalence of muscle versus skin/fat/fascia flaps for coverage of the open fracture even in patients with osteomyelitis. Negative pressure wound therapy has proven to be an excellent adjunct in the management of patients with these injuries. Between washouts, negative pressure devices can help decrease edema and isolate the wound and bone from the outside world. In a patient with a IIIB injury, there is insufficient tissue available to cover the wound. Therefore, secondary intention would not close the wound.

Split-thickness skin grafting provides an epithelial barrier to help seal off a wound from outside contamination. Grafts require a viable wound bed to survive. There must be a pliable bed to help grafts resist minor trauma in the future. With the periosteal stripping in this type of injury, a graft would not survive. In addition, graft placed directly on bone with periosteum would be very vulnerable to breakdown from minor trauma.

Bone allografting can be used to bridge defects in many circumstances. In the patient described, the severe contamination of the initial injury would make bone allografting much less appealing than autografting. Because of contamination, any type of bone grafting may need to be delayed until after achieving stable soft tissue coverage of the fracture.

A pedicled gastrocnemius muscle flap provides excellent coverage for defects about the knee, including the proximal tibia. Although the free gastrocnemius muscle flap could be transferred to any location, the pedicled flap would not be able to reach the distal tibia.

How well did you know this?
1
Not at all
2
3
4
5
Perfectly
40
Q

A 26-year-old man comes to the office for evaluation after sustaining an open injury to the right knee during a motorcycle collision 2 weeks ago. Physical examination shows a 2-cm defect over the patella. A medial gastrocnemius flap is planned to close the defect. Which of the following is the dominant vascular supply to this muscle?

A) Anterior tibial
B) Inferior geniculate
C) Medial sural
D) Posterior tibial
E) Superior geniculate

A

The correct response is Option C.

The gastrocnemius flap is the primary flap used to cover soft-tissue defects of the upper third of the tibia and knee. The gastrocnemius muscle is a bipennate muscle located on the posterior surface of the lower leg. The muscle originates from the medial and lateral condyles of the femur and inserts into the Achilles tendon. The dominant blood supply of the muscle is the medial and lateral sural arteries, which are branches of the popliteal artery. Generally only one head of the gastrocnemius flap is harvested to cover soft-tissue defects. The muscle alone is generally taken and is covered with a split-thickness skin graft for lower extremity reconstructions. The geniculate arteries primarily supply the bone around the knee joint.

How well did you know this?
1
Not at all
2
3
4
5
Perfectly
41
Q

A 50-year-old woman with systemic lupus erythematosus is evaluated because of a nonhealing ulcer of the right lower extremity. It started as a small pustule 3 months ago and steadily worsened to an ulcerative lesion. Examination of a biopsy specimen ruled out malignancy. Cultures have been negative for more than 4 weeks. Debridement of the wound and skin grafting are attempted but result in loss of the graft and development of similar ulcerative areas at the donor site. Which of the following is the most appropriate next step in management?

A) Bilayer skin substitute
B) Fasciocutaneous flap
C) Hyperbaric oxygen therapy
D) Long-term antibiotic therapy
E) Systemic corticosteroid therapy

A

The correct response is Option E.

The most appropriate next therapy option for this patient is systemic corticosteroids. These ulcerative lesions are most likely pyoderma gangrenosum (PG), an ulcerative cutaneous condition of unknown etiology. This condition is most likely associated with other systemic diseases like inflammatory bowel disease, or immunologic diseases. This diagnosis is usually one of exclusion, and one must have a high index of suspicion for ulcerative wounds that are persistent despite adequate workup and treatment. One must be especially aware of PG’s association with a condition known as pathergy. This is a phenomenon in which surgical manipulation of the area or distant sites may trigger worsening of the ulcerative condition and/or development of the condition in an area of skin trauma. First-line therapy for PG involves the use of prednisone. Other anti-inflammatory agents, including immunosuppressive agents, and biologic agents have also been used. The prognosis is generally good; however, the disease can recur and residual scarring is common. Because of these factors, the other options are not the most appropriate next steps in the treatment of this patient.

How well did you know this?
1
Not at all
2
3
4
5
Perfectly
42
Q

A 55-year-old man comes to the office because of an exposed knee prosthesis. Repair with a gastrocnemius flap is planned, using the entire muscle for reconstruction of the anterior knee defect and hardware coverage. The biomechanical consequence of using this flap is most likely to be observed by which of the following motions?

A) Dorsiflexion
B) Foot eversion
C) Foot inversion
D) Leg extension
E) Plantar flexion

A

The correct response is Option E.

The most appropriate answer is plantar flexion. The gastrocnemius muscle originates as two heads off the femur. The medial head comes off the medial condyle of the femur, just above the condyle and the lateral head comes off just above the lateral condyle. The muscle inserts onto the posterior calcaneus via the calcaneal tendon. This is the common tendon shared with the soleus muscle. Because of this fact, both heads can be harvested and the patient still maintains 75% of plantar flexion strength. The function of the gastrocnemius muscle is to plantar flex the foot and also flex the leg at the knee. Plantar flexion is the only biomechanical consequence listed above, although minimal. The blood supplies to the gastrocnemius muscle are from the sural branches of the popliteal artery and are independent. The medial head is the larger of the two and will have a larger arc of rotation. The innervation is via separate branches to each head off the tibial nerve.

How well did you know this?
1
Not at all
2
3
4
5
Perfectly
43
Q

A 68-year-old man presents 3 months after undergoing reconstruction of a large mandibular defect following tumor resection with a right fibula osteocutaneous flap. Postoperatively, immediate footdrop is noted. Which of the following is the most appropriate next step in management?

A) Clinical observation, conservative management, and re-evaluation in 3 months
B) Exploration of the peroneal nerve, neurolysis, and primary repair if transected
C) Exploration of the sural nerve, neurolysis, and primary repair if transected
D) Exploration of the tibial nerve, neurolysis, and primary repair if transected

A

The correct response is Option B.

It is recommended that the proximal 4 to 8 cm of the fibula be preserved in order to prevent knee instability and to avoid injury to the peroneal nerve. In large resections, more fibula length is required and the fibular head is often useful in the reconstruction. The common peroneal nerve is formed by the lateral division of the sciatic nerve. The peroneal nerve wraps around the lateral surface of the biceps femoris tendon and fibular head and courses into the anterolateral portion of the leg. The common peroneal nerve trifurcates into the superficial peroneal nerve, the deep peroneal nerve, and the recurrent articular branch. The deep peroneal nerve innervates the anterior compartment muscles of the leg, and provides ankle dorsiflexion. Injury to the common or deep peroneal nerve can result in footdrop or weakened dorsiflexion. Given that this patient had a large resection, the footdrop is indicative of a peroneal nerve injury. Exploration is warranted. The tibial nerve is a branch of the sciatic nerve and runs in the popliteal fossa to pass below the arch of the soleus muscle. The sural nerve is a sensory nerve in the calf. Injury to the tibial or sural nerve would not cause a footdrop.

How well did you know this?
1
Not at all
2
3
4
5
Perfectly
44
Q

A 40-year-old man is brought to the emergency department because of a grade IIIB open fracture of the right distal aspect of the tibia and fibula sustained during a motorcycle collision. The plastic surgeon is consulted after initial debridement and external fixation of the fracture. Examination shows a 10-cm open wound of the right medial ankle with complete transection of the tibial nerve. The tibia fracture is comminuted but without marked bone loss. The foot is well perfused, but single-vessel run-off through the anterior tibial artery is noted. Which of the following is the most appropriate management of this patient’s condition?

A) Debridement and free muscle flap without nerve repair
B) Debridement, tibial nerve repair, and coverage with a bilaminar acellular dermal regeneration template
C) Debridement, tibial nerve repair, and coverage with a free fasciocutaneous flap
D) Debridement, tibial nerve repair, and coverage with a pedicled reverse sural fasciocutaneous flap
E) Primary below-knee amputation

A

The correct response is Option C.

The answer is debridement with repair of the tibial nerve and coverage with an anterolateral thigh (ALT) free flap. This patient is presenting with a grade IIIB open fracture of the distal tibia and fibula with an open medial wound. Traditionally, lower extremity injuries with an insensate plantar foot were considered unsalvageable. However, more recent data have demonstrated that an insensate foot by itself should not be considered a contraindication to limb salvage if repair is otherwise possible. Studies have shown equivalent long-term outcomes with limb salvage and primary amputation and that half of the patients with tibial nerve injuries will regain plantar sensation within two years post-injury. This vignette describes a situation in which one could reasonably expect a successful outcome.

The size and location of the wound are most amenable to coverage with a microvascular free flap. In this instance, a fasciocutaneous flap will provide adequate soft-tissue coverage with minimal donor morbidity and potentially better long-term outcome if secondary bone grafting or hardware revision is required.

The reverse sural fasciocutaneous is a versatile flap that can be used for reconstruction of many different wounds of the distal lower extremity. The flap is a neurocutaneous flap supplied by the vascular axis of the sural nerve as well as distal peroneal artery perforators. In this case, the peroneal artery was damaged from the trauma and the pedicle of the flap is in the zone of injury, making it an inappropriate choice for reconstruction when better options are available.

The use of biologic materials such as bilaminar acellular dermal regeneration templates have been used successfully to cover wounds with exposed vital structures such as bone, tendon, and nerve. However, its use is best suited for smaller wounds or in patients in whom microvascular reconstruction is contraindicated or not desired. This patient required soft-tissue coverage over a major nerve repair and a comminuted fracture with internal hardware. A microvascular free flap would be the method of choice in this patient when feasible.

As stated previously, up to fifty percent of patients with tibial nerve injuries will recover plantar sensation after nerve repair. Nerve repair should be attempted to improve the overall outcome.

How well did you know this?
1
Not at all
2
3
4
5
Perfectly
45
Q

A 49-year-old man comes to the office because of dull, aching pain over the dorsum of the foot. Nerve study shows no abnormalities of the nerves around the knee but chronic denervation in the extensor digitorum brevis at the dorsum of the foot. Which of the following nerves is most likely to be entrapped?

A) Anterior tibial
B) Deep peroneal
C) Lesser saphenous
D) Medial plantar
E) Medial sural

A

The correct response is Option B.

The deep peroneal nerve emerges from the leg anterior compartment musculature, from beneath the extensor retinaculum of the ankle. The nerve gives off a motor branch to the extensor digitorum brevis, and then terminates into the first web space of the foot, after running beneath the tendon of the extensor hallucis brevis. Entrapment of this nerve, as it exits the extensor retinaculum of the ankle, can manifest itself as pain, weakness, or numbness or tingling over the dorsum of the foot. The saphenous nerve innervates the cutaneous region over the anterior-medial aspect of the distal leg and ankle. The medial sural nerve innervates the cutaneous region over the posterior lateral aspect of the lower leg. The tibial nerve innervates the plantar surface of the foot. It divides into the medial and lateral plantar nerve over the plantar aspect of the foot. The medial and lateral plantar nerves innervate the cutaneous aspect of the plantar foot and the intrinsic musculature of the foot.

How well did you know this?
1
Not at all
2
3
4
5
Perfectly
46
Q

A 62-year-old man comes to the office because of an open ankle fracture with exposed hardware. Use of a sural artery flap for reconstruction is planned. Which of the following veins must be harvested within the flap?

A) Anterior tibial
B) Lesser saphenous
C) Peroneal
D) Popliteal
E) Posterior tibial

A

The correct response is Option B.

The sural artery flap is a cutaneous flap located on the posterior aspect of the lower leg. The flap is based on the arteries that accompany the lesser saphenous vein and sural nerve; the vein and nerve must be included in the flap. The axial pattern flap can cover defects around the knee and upper third of the leg. The reverse flow flap was first introduced by Masquelet in 1992 and is a workhorse flap for pedicle reconstruction of the lower third defects in the leg. The anterior tibial, posterior tibial, popliteal, and peroneal veins do not contribute to the vascular anatomy of this flap.

How well did you know this?
1
Not at all
2
3
4
5
Perfectly
47
Q

A 30-year-old man is evaluated after sustaining multiple gunshot wounds to the right leg and thigh. X-ray study shows no retained foreign bodies and no fractures. On physical examination, the patient’s foot is warm, with palpable pulses at the ankle. He is able to extend the toes, dorsiflex the ankle, and evert the ankle. He is unable to flex his toes. He has normal sensation to the dorsum of his foot and medial-most part of the instep and lateral-most midfoot/hindfoot. Which of the following nerves is most likely injured in this patient?

A) Common peroneal nerve
B) Femoral nerve
C) Saphenous nerve
D) Sural nerve
E) Tibial nerve

A

The correct response is Option E.

Gunshot wounds can create a range of nerve injuries from contusion to transection. Electrodiagnostic testing can be very helpful in later diagnosis and intraoperatively during nerve reconstruction but will not demonstrate changes in the nerve on the day of injury. EMG/nerve conduction testing will not demonstrate changes in findings until 2 to 6 weeks after injury. An accurate sensory and motor examination is the best initial step to identify abnormalities that can be tracked over time.

The common peroneal nerve provides motor axons to the anterior and lateral compartment muscles. It also provides sensory axons to the dorsal foot, primarily via the terminal branches of the superficial peroneal nerve. Its only branch above the knee is to the lateral knee joint capsule.

The saphenous nerve receives the terminal branches of the femoral nerve. It provides sensation to the medial-most plantar surface of the instep.

The femoral nerve provides sensation to the thigh via cutaneous nerve branches as well as motor axons to the quadriceps muscle. Its terminal sensory fibers reach the foot via the saphenous nerve.

The sural nerve is a terminal branch of the tibial nerve. It provides motor axons to the gastrocnemius muscle and sensory fibers to the lateral-most forefoot and midfoot dorsally and lateral-most midfoot and hindfoot plantarly.

The tibial nerve provides sensation to the majority of the plantar surface of the foot via the medial and lateral plantar nerves. It also provides motor axons to the muscles of the deep posterior compartment, including the toe flexors. For the patient in this scenario, the tibial nerve is injured distal to the takeoff of the sural nerve.

How well did you know this?
1
Not at all
2
3
4
5
Perfectly
48
Q

A 30-year-old man comes to the office because of stage IV heel pressure ulcer of the right foot. Reconstruction with a medial plantar artery flap is performed. The pedicle for this flap derives from which of the following arteries in the lower extremity?

A) Anterior tibial
B) Dorsalis pedis
C) Lateral plantar
D) Peroneal
E) Posterior tibial

A

The correct response is Option E.

The medial instep flap (or medial plantar artery flap) is an ideal choice for coverage of a heel defect in a patient with adequate peripheral vasculature. This flap is based on the medial plantar branch of the posterior tibial artery. This vessel lies between the abductor hallucis and flexor digitorum brevis muscles.

The lateral plantar artery supplies the lateral aspect of the sole and digits but does not supply the medial instep. The anterior tibial artery and dorsalis pedis supply the dorsum of the foot and digits and are not involved in this flap. The peroneal artery is used in a fibular flap but not in the foot.

How well did you know this?
1
Not at all
2
3
4
5
Perfectly
49
Q

A 60-year-old man with type 2 diabetes mellitus comes to the office because of a diabetic ulcer on the sole of the right foot. Treatment of the ulcer with a medial plantar artery flap is planned. Against which of the following muscles is the arterial perforator located?

A) Adductor hallucis
B) Flexor hallucis
C) Lumbrical
D) Plantar interosseous
E) Quadratus plantae

A

The correct response is Option B.

The medial plantar artery flap is elevated starting at the plantar aspect, deep to the muscular fascia. The perforator is identified between the flexor hallucis and abductor hallucis muscles. The perforator is then dissected toward its origin on the medial plantar artery in the intermuscular space.

How well did you know this?
1
Not at all
2
3
4
5
Perfectly
50
Q

A 19-year-old man is brought to the emergency department after being thrown from his motorcycle. The trauma team has ruled out intracranial, thoracic, abdominal, and spinal injury. A comminuted tibia fracture is visible through a 7-cm full-thickness soft-tissue avulsion of the lower one third of the leg. Which of the following is the most appropriate next step in management?

A) Intraoperative debridement and washout of the wound, external fixation, and immediate cross-leg flap
B) Intraoperative debridement and washout of the wound, external fixation, burring of the tibia, and formation of granulation tissue over the next several weeks
C) Intraoperative debridement and washout of the wound, placement of external fixator, serial debridement, and free tissue transfer within 1 week of injury
D) Irrigation of the wound, stabilization of reduction with a cast, and application of suction wound dressing
E) Serial debridement of the wound and coverage with a gastrocnemius muscle flap

A

The correct response is Option C.

Lower extremity open fractures are described using the Gustilo classification. The patient in this scenario has a Gustilo IIIB: extensive soft tissue avulsion or degloving, from high velocity injury and gross contamination. The best treatment for such injuries is intraoperative debridement and washout with early or immediate fracture stabilization, often with an external fixator. Immediate soft-tissue reconstruction is not done due to the high-energy mechanism and gross contamination. This mandates repeat evaluation to assure all nonviable tissue and foreign material are removed prior to reconstruction. Definitive and stable soft-tissue reconstruction should be done as soon as possible and is classically thought to be best when provided within 72 hours of injury. Soft-tissue reconstruction done as quickly as possible reduces the risk of nonunion and osteomyelitis. In the proximal third of the leg, the gastrocnemius flap is indicated, while the soleus flap is for the middle third. Most often, free tissue transfer is the best option for the distal third wounds. Cross-leg flap is seldom used because of the prolonged immobilization that is required. Delayed reconstruction beyond the one week window is sometimes necessary because of other confounding factors in a multiple-trauma patient. In such situations, preventing desiccation of the bone is necessary, for which negative pressure wound therapy is useful. Soft tissue reconstruction is then accomplished when feasible with preference for flap reconstruction.

How well did you know this?
1
Not at all
2
3
4
5
Perfectly
51
Q

A 65-year-old man comes to the office because of an infected wound to the left plantar region. Medical history includes type 2 diabetes mellitus. Dorsalis pedis and posterior tibial pulses are not palpable but are located with a handheld Doppler probe. Ankle brachial index cannot be obtained because of noncompressible vessels in the left lower extremity, below the knee. Which of the following is the most appropriate next step in evaluating the arterial perfusion of this patient’s foot?

A) Computed tomography arteriography
B) Magnetic resonance arteriography
C) Percutaneous arteriography
D) Repeat ankle brachial index
E) Toe brachial index

A

The correct response is Option E.

The most appropriate next step when evaluating the arterial perfusion of this diabetic patient’s foot is obtaining a toe-brachial index (TBI). TBI is calculated by dividing the great toe systolic pressure by the brachial systolic pressure. An index of >0.7 is considered normal.

Patients with diabetes mellitus have a higher incidence of peripheral arterial disease than the non-diabetic population. Atherosclerotic lesions in diabetic patients tend to favor the arteries below the knee, which also commonly display medial calcinosis, causing stiffening of the arterial walls, poor compressibility, and an unreliably high ankle-brachial index (ABI). Interestingly, the small vessels of the great toes are usually spared of disease, therefore the diagnostic advantage of TBIs. These features should be taken into consideration whenever assessing arterial blood flow to the distal lower extremity of a diabetic patient.

Percutaneous arteriography is an invasive procedure and should be reserved for when surgical or endovascular therapeutic interventions are anticipated. Computed tomography arteriography and magnetic resonance arteriography may also be used in the diagnosis of peripheral artery disease in the lower extremities, but a normal TBI would most likely preclude their need. Repeating the ABI would most likely render a similar result, as the inability to compress the arteries in the leg is due to stiffened vessel walls.

How well did you know this?
1
Not at all
2
3
4
5
Perfectly
52
Q

A 25-year-old surfer who sustained a shark bite to the left thigh is brought to the emergency department. The patient is hemodynamically stable. Physical examination shows a bleeding mid-thigh wound. The left foot is pale and cool; sensibility in the foot is decreased. The ankle pulses are absent. On surgical exploration, a 6-cm injury to the superficial femoral artery is identified. After local debridement, which of the following is the most appropriate next step in management of the artery?

A) End-to-end anastomosis
B) End-to-side anastomosis
C) Interposition prosthetic grafting
D) Interposition vein grafting

A

The correct response is Option D.

The patient has a major vascular injury that is greater than 5.5 cm in length. A shark bite is considered contaminated and requires debridement. Because of the length of arterial injury, vein grafting is the most appropriate management option. End-to-end and end-to-side anastomoses are incorrect because the arterial defect is too long. The use of prosthetic material is incorrect because this is a contaminated wound, increasing the risk for infection.

How well did you know this?
1
Not at all
2
3
4
5
Perfectly
53
Q

Which of the following compartment pressure measurements is the minimum threshold that is most consistent with compartment syndrome?

A) 10 mmHg
B) 20 mmHg
C) 30 mmHg
D) 40 mmHg
E) 50 mmHg

A

The correct response is Option C.

The absolute minimum compartment pressure measurements ranging from 25 to 50 mmHg are quoted as absolute indications for fasciotomy. The most frequently quoted absolute measurement is 30 mmHg.

How well did you know this?
1
Not at all
2
3
4
5
Perfectly
54
Q

A 28-year-old man is flown by helicopter to the emergency department after sustaining a deep, isolated, lateral abrasion to the right lower leg in a motorcycle collision. On physical examination, he has a segmental injury to the common peroneal nerve. Repair with a sural nerve autograft is planned. Which of the following is the maximum length at which any functional recovery is expected?

A) 3 cm
B) 6 cm
C) 9 cm
D) 12 cm
E) 15 cm

A

The correct response is Option D.

Nerve repair outcomes are related to mechanism of injury, need for a graft and graft length, and timing of surgery relative to injury. Although results vary, good results are typical for grafts measuring less than 6 cm, and may be possible in approximately 25% of patients with grafts measuring 6 to 12 cm. Almost no studies report an M4 motor recovery or better when a graft greater than 12 cm is used.

How well did you know this?
1
Not at all
2
3
4
5
Perfectly
55
Q

A 35-year-old man who was involved in a motorcycle accident sustains fractures to the right tibia and fibula. On physical examination, he has numbness in the dorsum of the right foot and inability to dorsiflex the foot. Vascular status of the right lower extremity is normal. Which of the following nerves has most likely been injured?

A) Calcaneal
B) Common peroneal
C) Lateral plantar
D) Lateral sural cutaneous
E) Posterior tibial

A

The correct response is Option B.

The common peroneal nerve derives from the dorsal branches of the fourth and fifth lumbar and first and second sacral nerves. The common peroneal nerve lies between the biceps femoris and lateral head of the gastrocnemius muscle; it continues around the neck of the fibula between the peroneus longus muscle and the fibula, and then branches into the superficial fibular and deep fibular nerves. The common peroneal nerve innervates the peroneus longus, peroneus brevis, and biceps femoris muscle. Injury to this nerve results in a foot drop and sensory loss to the dorsal surface of the foot.

The lateral sural nerve is a cutaneous nerve arising from the common fibular nerve. It supplies sensation to the posterior and lateral surfaces of the leg. This nerve does not supply motor innervation.

The posterior tibial nerve, also known as the tibial nerve, is derived from L4, L5, S1, S2, and S3. It is a branch of the sciatic nerve. The nerve gives branches to the gastrocnemius, popliteus, and soleus muscles. Below the soleus muscle it supplies the tibialis posterior, the flexor digitorum longus, and the flexor hallucis longus muscles.

The lateral plantar nerve is a branch of the tibial nerve. It supplies the quadratus plantae and the abductor digiti minimi muscles. Its sensory component supplies the skin of the fifth toe and the lateral half of the fourth toe.

The lateral calcaneal nerve is a branch of the sural nerve supplying cutaneous sensation to the lateral aspect of the heel skin. This nerve is a cutaneous nerve with no muscle innervation.

How well did you know this?
1
Not at all
2
3
4
5
Perfectly
56
Q

A 35-year-old man is brought to the emergency department because of an injury to the left lower leg after being involved in a motorcycle collision. X-ray studies confirm a Gustilo IIIB tibia-fibula fracture. After debridement, there is a bone defect measuring 12 cm in the mid shaft of the tibia. Which of the following is the most appropriate technique to restore the bone defect?

A) Autogenous bone grafting
B) Cadaveric bone grafting
C) Coverage with a fibular free flap
D) Osteodistraction

A

The correct response is Option C.

Although various techniques have been used successfully to reconstruct large bony defects of the lower extremity, the most reliable technique for such a large bone gap is the fibular free flap reconstruction. The Ilizarov osteodistraction technique can be used for large defects, but would necessitate a very long period of immobilization and fixation. Neither autogenous nor cadaveric bone graft would be as reliable as vascularized bone.

How well did you know this?
1
Not at all
2
3
4
5
Perfectly
57
Q

A 27-year-old man is brought to the emergency department because of compartment syndrome of the lower left leg. Release of the deep posterior compartment includes decompression of which of the following muscles?

A) Flexor hallucis longus
B) Peroneus brevis
C) Plantaris
D) Soleus
E) Tibialis anterior

A

The correct response is Option A.

The flexor hallucis longus flexes the great toe and is located in the deep posterior compartment. The soleus and plantaris muscles are located in the superficial posterior compartment, just deep to the gastrocnemius muscles. The peroneus brevis is located in the lateral compartment. The tibialis anterior is located in the anterior compartment.

How well did you know this?
1
Not at all
2
3
4
5
Perfectly
58
Q

A 24-year-old man is brought by ambulance to the emergency department after sustaining a tibial fracture in a motorcycle collision. On physical examination, there is an open, segmental fracture with a 4-cm wound and a large avulsion flap, but adequate soft-tissue coverage. Which of the following Gustilo-Anderson fracture classifications best describes this injury?

A) Type I
B) Type II
C) Type IIIA
D) Type IIIB
E) Type IIIC

A

The correct response is Option C.

How well did you know this?
1
Not at all
2
3
4
5
Perfectly
59
Q

A 37-year-old postal worker is brought to the emergency department 2 hours after he sustained a crush injury to his right leg when his truck rolled downhill and pinned him to a wall. He reports progressive, intense burning pain of the right leg. On examination, the leg appears swollen and there is a tense woody feeling anterolaterally associated with severe tenderness on passive range of motion of the ankle. There is decreased sensation between the first and second toes. Palpable dorsalis pedis and posterior tibial pulses are noted. The difference between diastolic blood pressure and the anterior compartment pressure (delta pressure) is 15 mmHg. X-ray studies show no fracture. Which of the following is the most appropriate next step?

A) Perform emergency fasciotomy
B) Recheck anterior compartment pressure in 1 hour
C) Test electromyographic activity of the anterior tibialis
D) Test serum creatinine kinase activity
E) Wean analgesics to reduce variability in serial review

A

The correct response is Option A.

This patient is manifesting signs and symptoms of progressive acute compartment syndrome (ACS). ACS is a surgical emergency, and the next step in management is acute fasciotomy to fully decompress all involved compartments. Delays in fasciotomy increase unrecoverable tissue injury leading to permanent functional loss and morbidity such as muscle contractures, sensory deficits, paralysis, and infection.

Compartment syndrome occurs when increased pressures within unyielding fascial compartments lead to progressive cellular anoxic injury to tissues within the compartment. Symptoms of ACS include pain out of proportion to injury and deep aching or burning pain. Complaints of paresthesia suggest progressive ischemic nerve dysfunction. Signs of ACS include pain with passive ranging of muscles in the affected compartment, a tense compartment with a firm and indurated “woody” feeling, and diminished sensation.

Most cases of ACS occur with long-bone fractures, and risk increases with comminuted fractures. Other common forms of trauma including crush injury, severe thermal burns, constrictive bandages, penetrating trauma, vascular injuries, and ischemia-reperfusion injuries can lead to ACS. The anterior compartment of the leg is the most common site for ACS. Early signs of ACS affecting this compartment include loss of sensation in the distribution of the deep peroneal nerve, and weakness of dorsiflexion.

Normal compartment pressures fall between 0 and 8 mmHg. Pain develops as compartment pressures increase, and tissue ischemia occurs as compartment pressures approach diastolic pressure. Differences between diastolic blood pressure and measured compartment pressure (delta pressure) less than 20 to 30 mmHg indicate a need for fasciotomy.

As ACS evolves, muscle breakdown can lead to elevations in serum creatine kinase and myoglobinuria. Sensory deficits typically preclude motor deficits and EMG abnormalities progressing to total paralysis will occur. These are relatively late findings in ACS; they should not delay fasciotomy once progressive ACS is diagnosed.

There is no diagnostic role for withholding analgesics in extremity trauma.

How well did you know this?
1
Not at all
2
3
4
5
Perfectly
60
Q

A 28-year-old man presents 8 days after open reduction and internal fixation of an unstable distal tibia fracture. Postoperatively, the incision has dehisced. Examination shows a 5 × 2-cm open wound with marginal skin necrosis and exposed hardware. Which of the following is the most appropriate next step in soft-tissue coverage?

A) Debridement with application of skin substitute
B) Hardware removal, casting, and wound care
C) Hyperbaric oxygen and wound care
D) Operative debridement and placement of a VAC for a bridge to skin grafting
E) Operative debridement and vascularized reconstruction with a flap

A

The correct response is Option E.

The best treatment that would allow salvage of the fracture fixation is operative debridement and vascularized flap reconstruction. Stable fixation has been achieved and subacute wound dehiscence has occurred because of ischemia or devitalization of the overlying soft tissue. This is due to the forces of the original trauma as well as potential further traumatic insult of the tissue during surgical repair. In the absence of infection, immediate soft-tissue reconstruction will provide stable vascularized soft-tissue coverage of the fracture site and the hardware. As such, vascularized flap reconstruction is appropriate. In the distal leg, this often requires free tissue transfer, but depending on the location and size of the defect, soleus flap or perforator propeller flaps can be used.

Gustilo provided a classification of open fractures of the leg in which the fracture site was exposed through a disruption of soft-tissue integrity:

Determination of the type of flap reconstruction required requires assessment of not only the location and size of the defect, but also the zone of injury. Greater degrees of force are associated with the increasing Gustilo classification such that type III fractures often require free tissue transfer because of concomitant damage of the regional and local tissues.

Operative debridement and placement of a VAC for a bridge to skin grafting is a potential treatment for an open wound with exposed bone without hardware. The period of time in which the wound remains open and granulates during this process provides a very high risk for hardware infection, nonunion, and osteomyelitis.

Debridement with skin advancement and closure is likely to fail because of the difficulty in providing appropriate tension-free advancement flaps in the leg, combined with the need to accommodate for tissue loss from the debridement and ischemia of the advancing skin edges. The reliability of this treatment is poor and would have high risk for failure and subsequent hardware infection and nonunion or osteomyelitis.

Hardware removal, casting, and wound care is not indicated since the fracture repair is intact and no signs of infection are present. However, if hardware removal were required because of overt infection, the most appropriate treatment would be placement of an external fixator and soft-tissue flap reconstruction.

Hyperbaric oxygen and wound care is not the best option in this acute situation in which prompt soft-tissue reconstruction and vascularized coverage of the fracture site are required to salvage the existent fixation and avoid mal/nonunion or osteomyelitis.

How well did you know this?
1
Not at all
2
3
4
5
Perfectly
61
Q

An otherwise healthy 65-year-old man is evaluated because of a 2-month history of a nonhealing wound to the back of the left heel. He has a history of smoking 50 packs of cigarettes yearly but quit 1 year ago. Physical examination shows a clean wound with exposed bone and palpable distal pulses in the lower extremities. Coverage with a distally based fasciocutaneous sural flap is planned. Because of the patient’s history of smoking, a “delay” procedure is performed first. Division of which of the following is required for this procedure?

A) Distal greater saphenous vein
B) Distal lesser saphenous vein
C) Perforator 5 cm proximal to the lateral malleolus
D) Proximal greater saphenous vein
E) Proximal lesser saphenous vein

A

The correct response is Option E.

The surgical step required as part of the “delay” procedure in a distally based sural flap is division of the proximal lesser saphenous vein. The distally based sural flap is a neurofasciocutaneous flap used to reconstruct ankle, heel, and foot defects. The classically described and possibly most important arterial supply to the distally based sural flap is provided by septocutaneous perforators arising from the peroneal artery. The most distal of these is located 4 to 7 cm proximal to the lateral malleolus. However, there are at least three other sources described: fasciocutaneous perforators from the posterior tibial artery, venocutaneous perforators from the lesser saphenous vein, and neurocutaneous perforators from the sural nerve. The skin and fascia of the flap are drained primarily by the lesser saphenous vein. The lesser saphenous vein contains numerous valves that prevent retrograde blood flow. There are, however, one or more smaller collateral veins that run parallel to the lesser saphenous vein. These veins have anastomotic connections to the lesser saphenous vein, which can allow blood to bypass the valves of the lesser saphenous vein and flow in a retrograde fashion.

In attempts to redirect blood flow and decrease the risk of flap necrosis and other complications, several authors have described sural flap delay procedures. Two distinct delay procedures have been described. In one, the flap is first elevated without completely incising the proximal edge of the skin island. A powder-free glove is then placed between the elevated fascia and the gastrocnemius muscle, and the skin is closed. Two weeks later, the flap is completely elevated and transferred into the defect site. This procedure has the goal of redirecting blood flow in a longitudinal direction before complete elevation of the flap. In the other technique, the flap is raised in its entirety and then sutured back into its donor site. The flap is then transferred into its recipient site as a second procedure. This technique allows the flap to become viable on its distal vascular pedicle before causing the additional trauma of transferring the flap, which can potentially compromise that pedicle.

Division of the greater saphenous vein is not indicated because it is not in the vicinity of the flap. Similarly, division of the perforator 5 cm proximal to the lateral malleolus is not appropriate because this is the major pedicle supplying the flap.

How well did you know this?
1
Not at all
2
3
4
5
Perfectly
62
Q

A 56-year-old man who is an active smoker sustains a degloving injury of the left foot from a motorcycle collision. The heel was avulsed from the calcaneus by a deep posterior laceration but has normal capillary refill. No tissue is missing, but the wound is heavily contaminated and the calcaneus has an abrasion that is imbedded with grit. After irrigation and debridement in the operating room, which of the following is the most appropriate next step in management of this wound?

A) Coverage with a free gracilis muscle flap
B) Healing by secondary intention
C) Layered closure over a drain
D) Negative pressure wound therapy and skin grafting
E) Serial debridement and delayed closure

A

The correct response is Option E.

The most appropriate management of this wound is serial debridement and delayed closure. With such a high level of contamination of both the soft and hard tissues, layered closure after the initial debridement will very likely lead to infection, especially in a patient with a history of smoking. It would be a mistake to perform a free tissue transfer in a highly contaminated wound. Furthermore, there is no missing or ischemic tissue. Negative pressure wound therapy followed by skin grafting would not be appropriate for a deep wound with bone exposure when local tissues are available for closure; this would be more appropriate for a superficial wound with missing skin. Healing by secondary intention is an option; however, serial debridement and delayed closure will take less time, is less painful, and avoids scar formation in the heel.

How well did you know this?
1
Not at all
2
3
4
5
Perfectly
63
Q

A thin 40-year-old woman has an 8 × 5-cm skin defect in the distal third of the anterior leg extending to the dorsum of the foot, with tibia denuded of periosteum and exposed tendon, after undergoing stabilization of the fracture with internal hardware 3 days ago. The distal posterior tibial artery was ligated before surgery at the distal third of the leg. There are no signs of infection or osteomyelitis. Which of the following is the most appropriate method of reconstruction?

A) Application of bilaminate neodermis (Integra) and negative pressure wound therapy
B) Coverage with a dorsalis pedis flap
C) Coverage with a free anterior lateral thigh (ALT) flap
D) Coverage with a free tranverse rectus abdominus myocutaneous (TRAM) flap
E) Coverage with a reverse sural flap

A

The correct response is Option C.

A free anterior lateral thigh flap is large enough to close the defect, can be thinned for aesthetics and shoe wear, and may allow for primary closure of the donor site. Although free tranverse rectus abdominus myocutaneous (TRAM) flap coverage is a possibility, the potential complications of taking muscle and unpredictable control of the final contour make them less ideal options. The reverse sural flap is not an option because of the ligation of the posterior tibial artery. In addition to having severe donor site morbidity, the dorsalis pedis would remove the remaining blood supply to the foot. The vascular nature of the defect’s wound bed makes bilaminate neodermis (Integra) and negative pressure wound therapy a less optimal choice.

How well did you know this?
1
Not at all
2
3
4
5
Perfectly
64
Q

A 15-year-old girl sustained an isolated open tibial fracture in a motor vehicle collision. At the proximal third of the tibia, 15 cm of anterior soft-tissue loss is noted. Despite fracture reduction, the foot is warm but pulseless without dopplerable signals. The patient is otherwise stable. Which of the following is the most appropriate next step in management?

A) Below-knee amputation
B) CT angiography
C) Four-compartment fasciotomy
D) Internal fixation and soft-tissue coverage
E) Surgical exploration of the popliteal artery

A

The correct response is Option B.

Lower extremity fractures with combined soft-tissue and neurovascular trauma have high rates of complications, and a percentage of these injuries lead to amputation. Risk factors for amputation include Gustilo IIIC injuries, sciatic or tibial nerve injuries, prolonged ischemia (more than 4 to 6 hours), significant soft-tissue injury, significant wound contamination, multiple injured extremities, advanced age, lower versus upper extremity trauma, and futile attempt at revascularization. While tibial nerve injury is a risk factor and relative indication for amputation, it is never an absolute indication for amputation.

Hard signs for vascular injury include: active hemorrhage, expanding hematoma, bruit or thrill, absent distal pulses, and distal ischemic signs and symptoms (five P’s). In the face of these hard signs, imaging such as CT angiography should be used to evaluate for vascular injury. With that said, most hard signs can be explained by soft-tissue or bone bleeding, traction of intact arteries due to unreduced fractures, or compartment syndrome.

Early soft-tissue coverage is associated with a lower complication rate. The goal is to close wounds within 7 to 10 days to decrease the risk for infection, osteomyelitis, nonunion, and further tissue loss.

It is best to get wound control prior to bone grafting, avoiding the risk of losing valuable limited bone; therefore, bone grafting is generally postponed until 8 to 10 weeks after soft-tissue wound coverage.

How well did you know this?
1
Not at all
2
3
4
5
Perfectly
65
Q

A 20-year-old man has purulent breakdown 5 months after sustaining a Gustilo type IIIB open fracture treated with intramedullary rod placement and skin grafting over a medial gastrocnemius flap. A postoperative x-ray study and current photograph are shown. The intramedullary rod is removed and an external fixator is placed. There is 1.5 cm of bone without periosteum surrounding the fracture exposed in the wound. Which of the following is the most appropriate next step in wound reconstruction?

A) Full-thickness skin grafting with a bolster dressing
B) Reconstruction with an anterior tibial artery perforator flap
C) Reconstruction with a lateral gastrocnemius muscle flap and skin grafting
D) Reconstruction with a pedicled descending medial genicular artery flap
E) Split-thickness skin grafting with negative pressure wound therapy

A

The correct response is Option B.

Perforator flap reconstruction, whether free or pedicled, has become increasingly popular over the past decade. Perforator flap use allows for the creation of an axial pattern flap without the sacrifice of a major artery and can often be done for areas once considered to require free flaps for coverage. Prior transfer of a medial gastrocnemius flap might disrupt perforators from the posterior tibial artery to the medial leg skin, but would not have disturbed anterior tibial artery perforators through the skin of the anterolateral leg.

Skin grafting, whether split- or full-thickness, would not be successful on fractured bone without periosteum, regardless of the type of dressing used.

The lateral gastrocnemius muscle is smaller and cannot reach as far as the medial gastrocnemius. It would not be able to reach the mid-shaft tibia defect shown in this patient.

The descending medial genicular artery is the pedicle of the medial femoral condyle flap. It is normally used as a bone graft donor, although an overlying skin paddle can be harvested with it. When used in a pedicled fashion, it can be transposed proximally onto the thigh, but not distally onto the leg.

How well did you know this?
1
Not at all
2
3
4
5
Perfectly
66
Q

A 35-year-old man is referred to the office after undergoing prolonged failed attempts at local wound care of an exposed Achilles tendon. Physical examination shows that the tendon is beginning to desiccate. Coverage with a flap is performed, as shown. The blood supply to this flap is derived from which of the following arteries?

A) Anterior tibial
B) Geniculate
C) Peroneal
D) Popliteal
E) Superficial femoral

A

The correct response is Option C.

The flap used to cover the Achilles tendon in the patient described is the reverse sural fasciocutaneous flap. It is based on perforators from the peroneal artery through a network of small vessels. The general axis of the flap follows the sural nerve from behind the lateral malleolus to the mid portion of the gastrocnemius muscle bellies. Sural nerve injury results in loss of lateral foot sensation. This is often of no functional consequence because its harvest does not result in loss of a major neurovascular structure.

The other vessels noted do not supply inflow to the flap.

How well did you know this?
1
Not at all
2
3
4
5
Perfectly
67
Q

A 66-year-old man comes for evaluation because of a chronic left lower extremity wound. He reports that the wound has been present for the past 15 years. Physical examination shows an 8 × 10-cm wound on the lateral aspect of the left calf. Which of the following is the most appropriate next step in diagnosis?

A) Angiography
B) Biopsy
C) Bone scan
D) CT scan
E) MRI

A

The correct response is Option B.

The most important next step in establishing a diagnosis in this patient is an excisional biopsy. The clinical appearance and duration of the chronic wound is highly suggestive of a Marjolin ulcer in the setting of a chronic venous stasis ulcer. Marjolin ulcers are defined as malignant generation in the presence of a burn wound or other chronic inflammatory conditions. The most common etiology is a burn wound; however, malignancies have been found in chronic wounds secondary to pressure ulceration, trauma, venous stasis, and others. The most common pathologic diagnosis is well-differentiated squamous cell carcinoma (SCC), but basal cell carcinoma and various other sarcomas have been reported in the literature. Marjolin ulcers are thought to be aggressive forms of SCC with metastatic potential related to tumor grade. The incidence of metastasis is 10% for Grade 1 lesions, 59% for Grade 2, and 86% for Grade 3.

Diagnosis is based on clinical appearance, history, and most importantly biopsy. Patients should also receive a thorough regional lymph node exam, CT scan or MRI, and routine laboratory analysis. Sentinal lymph node biopsy may be indicated. Treatment is generally wide excision of the chronic wound and rapid coverage with skin grafts or tissue flaps. Depending on tumor characteristics, adjuvant radiation therapy may be recommended.

Angiography will evaluate the arterial flow of the extremity and may be important in planning coverage of the wound, but it is not required to rule out a malignancy. MRI, CT scan, and x-ray studies can also be helpful as adjunctive diagnostic techniques to evaluate the extent of the tumor and presence of metastasis, but they are not used for diagnosis alone.

68
Q

An otherwise healthy 30-year-old man is evaluated because of left foot drop after posterior knee dislocation. The common peroneal nerve is explored at the level of the knee shortly after the time of injury and found to be in continuity. The patient does not return for follow-up examination within the next year. Eighteen months after the injury, the patient returns for follow-up examination and shows no improvement of the foot drop. Passive range of motion of the ankle is full. Which of the following transfers is most likely to correct this patient’s foot drop deformity?

A) Peroneus brevis tendon with graft to calcaneus bone
B) Peroneus longus tendon to calcaneus bone
C) Peroneus longus tendon to talus bone
D) Tibialis anterior nerve to tibialis posterior nerve
E) Tibialis posterior tendon to tibialis anterior tendon

A

The correct response is Option E.

Common peroneal nerve injury is common after posterior knee dislocation, usually occurring at or near where the nerve crosses the fibula neck. After surgical exploration to confirm the peroneal nerve is intact, initial management consists of supportive care with an ankle brace to correct foot drop. Patients are observed clinically for recovery, often with serial electromyography and nerve conduction studies.

Tibialis posterior to anterior transfer will restore the patient’s ability to dorsiflex the ankle. The donor muscle is innervated by the tibial nerve, which is not commonly injured in a posterior knee dislocation. The tendon is dis-inserted from the tarsus and brought out through the medial leg.

It is transferred through the interosseous membrane to the tibialis anterior tendon.

By 18 months after injury, the motor end plates to the tibialis anterior have degenerated, making a nerve transfer no longer an option. Earlier after injury, a nerve transfer from the tibialis posterior to the tibialis anterior might be a feasible option.

Peroneus longus and brevis would also be paralyzed in the setting of a common peroneal nerve injury such as this patient’s. In certain situations, such as with some patients with leprosy, the deep peroneal nerve is not paralyzed. For these patients, the peroneus longus can be used as a tendon transfer, but it is normally used to provide toe extension in conjunction with a tibialis posterior transfer to provide ankle dorsiflexion.

69
Q

A 45-year-old man is brought to the emergency department after sustaining a fracture of the neck of the fibula after being struck by a baseball. Physical examination shows major nerve deficit. Which of the following actions will the patient be unable to perform?

A) Dorsiflex the foot
B) Extend the leg
C) Flex the leg
D) Invert the foot
E) Plantar flex the foot

A

The correct response is Option A.

The most commonly injured nerve in the leg is the common peroneal nerve because of its superficial location as it courses around the neck of the fibula. The common peroneal nerve then divides into the superficial and deep branches. The superficial branch will evert the foot by innervating the lateral compartment of the leg, while the deep branch will dorsiflex the foot by innervating the anterior compartment. The superficial branch also provides sensation for the anterior and lateral sides of the leg and the majority of the dorsum of the foot and toes, including the medial side of the big toe. Paralysis of the common peroneal nerve would lead to foot drop and foot inversion, abnormal “steppage” gait, and loss of sensation.

70
Q

A 47-year-old woman is referred by orthopedic surgery for evaluation and discussion of soft-tissue reconstruction at the time of nonvascularized allograft reconstruction of recurrent Achilles tendon rupture. The patient has a history of congenital clubfoot and multiple previous Achilles tendon repairs. Physical examination shows atrophied skin and multiple longitudinal scars along both the medial and lateral distal posterior calf. Which of the following is the most appropriate management?

A) Cross-leg fasciocutaneous flap
B) Fasciocutaneous free flap
C) Reverse sural artery flap
D) Soleus muscle flap
E) Tissue expansion

A

The correct response is Option B.

On the basis of the scenario described, fasciocutaneous free flap is the most appropriate management option.

The soleus muscle flap is appropriate for defects of the middle third of the leg but lacks adequate reach for soft-tissue coverage of the distal third of the leg.

Tissue expansion has been described for soft-tissue reconstruction of congenital talipes equinovarus but is usually reserved for children and in the setting of primary correction. When comparing tissue expansion in the limb versus non-limb sites, the incidence of complications associated with tissue expansion is significantly higher in the limb. Because a nonvascularized allograft is to be used, and the patient has a contracted and scarred soft-tissue envelope, tissue expansion would be associated with higher risk of expansion failure and complications when compared with free tissue transfer soft-tissue reconstruction.

Cross-leg flaps are rarely used because of the availability of free tissue transfer. This flap is more appropriate in children than elderly patients, in whom stiffness is a factor.

A reverse sural artery flap is not appropriate given the patient’s multiple past surgeries and local scars.

71
Q

A 29-year-old man undergoes open reduction and internal fixation of an open fracture of the proximal right tibia. There is no tissue loss, and there is little wound contamination. The wound is closed with 2-cm raised flaps. Reconstruction of the popliteal artery is required. Which of the following Gustilo fracture classifications is most likely in this patient?

A) I
B) II
C) IIIA
D) IIIB
E) IIIC

A

The correct response is Option E.

Gustilo initially classified long-bone fractures into three types in order to establish a treatment algorithm. Essentially, this classification subdivided fractures according to the energy of the initial trauma that resulted in significant soft-tissue injury, periosteal stripping, and fracture comminution in the worst subtype. Debridement, antibiotics, and primary or delayed wound closure were advocated dependent on fracture severity. Type III fractures were subsequently subdivided into A, B, and C subtypes. Subtypes were stratified according to potential for complications such as infection, osteomyelitis, non-union, and amputation rates. Type IIIC had open fracture with arterial injury requiring repair (the case in this patient, even though there appears to be adequate soft-tissue coverage).

Although fracture fixation methods have substantially improved since the original publications of Gustilo, the ability to transport bone into segmental traumatic defects has also since developed, and free flaps have extended our ability to cover large wounds. This classification system has stood the test of time and still forms the basis of prognosticating and determining the optimum treatment algorithm.

72
Q

A 57-year-old man comes to the office 4 weeks after undergoing a free osseocutaneous fibula flap. He says he has pain with walking. A photograph is shown. X-ray studies show 6 cm of fibular bone remains proximally and distally. Sensation of the right foot shows no abnormalities; pain is noted on plantar flexion. Which of the following is the most appropriate next step in management?

A) Cast immobilization of the lower extremity (above the knee)
B) Cast immobilization of the lower extremity (below the knee)
C) Operative exploration and bone grafting
D) Operative exploration and nerve grafting
E) Reassurance that the pain is self-limiting

A

The correct response is Option E.

Vascularized bone flap is typically needed for defects >6 cm regardless of location in the body. The fibula is a common donor for vascularized bone. Understanding the postoperative course and complications is needed both in terms of discussions with the patient preoperatively and management of the patient’s condition after surgery. Common sequelae of fibula harvest include pain in the leg (especially when walking). Four weeks is relatively early in the postoperative course and reassurance should be given.

Risks of fibula harvest include damage to the peroneal nerve (increased when <6 cm of bone is left behind or when the head of the fibula is included in the harvest); destabilization of the ankle (increased when <6 cm of bone is left behind); and damage to the posterior tibial nerve.

A free-fibular flap design with hash marks left intact is shown.

73
Q

A 32-year-old woman comes to the office because the toes of the right foot “drag” when she walks. She underwent vein stripping of the right leg and ligation of the lesser saphenous vein 4 weeks ago. Physical examination shows absent dorsiflexion and eversion of the ankle. Electromyography findings show:

Which of the following is the most likely site of nerve injury in this patient?

A) Common peroneal nerve at the knee
B) Superficial peroneal nerve at the knee
C) Sural nerve at the knee
D) Tibial nerve at the knee
E) Tibial nerve at the mid calf

A

The correct response is Option A.

The most likely site of injury would be the common peroneal nerve at the knee. Injuries to the common peroneal nerve are well documented in both traumatic (knee dislocation) and iatrogenic settings. Patients have footdrop and numbness over the first dorsal web space of the foot. History and clinical examination are the mainstays for diagnosis, but electromyography can be helpful in less-clear circumstances. The absence of recruitment of the lateral compartment muscles (peroneals) and the anterior compartment muscles (tibialis anterior, extensor hallucis longus) strongly suggest common peroneal involvement. The presence of recruitment of the biceps femoris and the tibialis posterior rules out tibial nerve involvement. An isolated superficial peroneal nerve injury would spare the anterior compartment muscles.

The sural nerve is a sensory nerve and provides no motor function.

74
Q

A 24-year-old man is brought to the emergency department 2 hours after sustaining injuries to the left lower extremity when he was hit by a motor vehicle. Physical examination shows avulsion of the soft tissue of the posterior thigh. A fracture of the femur is stabilized by an intramedullary rod; the sciatic nerve is noted to be intact but ecchymotic at the level of the mid posterior thigh. Soft tissue is available for coverage. Which of the following is the most appropriate management?

A) Acute resection of the ecchymotic nerve and repair with a nerve graft
B) Delayed resection of the ecchymotic nerve at 10 days and repair with a nerve graft
C) Electromyography after 3 weeks and repair with a nerve graft if fibrillations occur
D) Serial electrodiagnostic studies after 3 weeks and again after 3 months with repair if no improvement

A

The correct response is Option D.

In cases in which the nerve has undergone a significant crush component, it is important to get a sense of nerve viability and recovery. The nerve conduction study at 3 weeks largely serves as a baseline study as it rarely provides more information than physical examination other than the presence of fibrillations, which indicates at least some axonal loss. The nerve conduction study at 3 weeks is not a reliable indicator of possible nerve recovery; therefore, resection and reconstruction are not advisable at this time point. The 3-month nerve conduction study is able to pick up subtle signs of recovery that may not be evident on physical examination. If at the 3-month mark there are no signs of recovery on physical examination or nerve conduction study, repair is indicated. Acute resection of a possibly viable nerve is not indicated. If the nerve were noted to be transected at the time of initial exploratory surgery, the viability of the nerve ends would not be stable until 7 to 10 days post trauma, making this a good time for definitive repair. In the scenario described, the nerve is in continuity and viability cannot be ascertained intraoperatively at 10 days; therefore, resection and reconstruction are not indicated.

75
Q

A 35-year-old man is brought to the emergency department 2 hours after sustaining a severe crush injury to the right distal thigh in a motor vehicle collision. Physical examination shows an open fracture of the femur; the leg and foot are pale and cool. There are no palpable popliteal, dorsalis pedis, or posterior tibialis pulses. Closed reduction does not restore perfusion. Which of the following is the most appropriate next step?

A) Arterial repair with a polytetrafluoroethylene graft
B) Arterial repair with reverse saphenous vein graft
C) CT angiography
D) Intramedullary fixation of the femoral fracture
E) Placement of a temporary vascular shunt

A

The correct response is Option E.

Gustilo Type IIIC fractures involve arterial injury requiring repair irrespective of the degree of soft tissue and often represent significant limb-threatening injuries. Early recognition and management of lower extremity vascular injury is crucial to limb salvage. CT angiography is of little benefit in the presence of hard signs of vascular injury and can delay operative intervention as well as increase limb ischemia time.

The combination of vascular and orthopedic injuries requiring repair is rare, with a reported incidence as low as 1.5%. Data exist from both wartime and civilian groups evaluating the sequence of management of such injuries. The recommended algorithm suggests improved ischemia times and favorable limb salvage rates with temporary vascular repair, using shunts as the initial adjunct to restore perfusion followed by debridement and fracture fixation.

Definitive vascular repair should follow debridement and fracture fixation. Both synthetic polytetrafluoroethylene and autologous (reverse saphenous vein) interposition grafts are reported to be used in traumatic reconstruction, although autologous tissue is often preferred in the setting of gross contamination.

76
Q

A 25-year-old man is brought to the emergency department after he sustained a knife wound to the right lower extremity. Examination shows numbness of the lateral aspect of the leg and weakness in plantar flexion and eversion of the foot. Which of the following nerves was most likely injured in this patient?

A) Femoral
B) Obturator
C) Peroneal
D) Sural
E) Tibial

A

The correct response is Option C.

The patient appears to demonstrate symptoms of a superficial peroneal nerve injury. The superficial peroneal nerve arises from the common peroneal nerve at the fibular neck. It supplies the lateral compartment of the leg, giving motor branches to peroneus longus and brevis, as well as sensory contribution to the lateral aspect of the leg. Injury to the superficial peroneal nerve results in anesthesia of the lateral aspect of the leg and weakness in eversion and plantar flexion of the foot.

The deep peroneal nerve arises from the common peroneal nerve at the fibular neck. It travels in the anterior compartment of the leg and gives branches to the tibialis anterior, extensor hallucis longus, and extensor digitorum longus and brevis, as well as peroneus tertius. The sensory distribution is in the area of the first web space. Injury to the deep peroneal nerve causes weakness in dorsiflexion of the foot.

The femoral nerve innervates muscles of the anterior thigh, including the quadriceps group, iliacus, and sartorius. Injury to the femoral nerve results in weakness of leg extension.

The obturator nerve provides innervation to the medial thigh muscles (adductor group), including adductor brevis, longus, and magnus, as well as the gracilis and obturator externus. The cutaneous branch provides sensation of the medial thigh. Injury to the obturator nerve results in weakness in thigh adduction and sensory deficits in the medial thigh.

The sural nerve travels on the posterior aspect of the leg between the lateral malleolus and calcaneus. It provides sensation to the lateral aspect of the foot and does not have a motor component. It is commonly sampled in nerve biopsy and used as a source of nerve graft.

Injury or sacrifice of the sural nerve would result in numbness of the lateral foot.

The tibial nerve is a branch of the sciatic nerve. It travels through the popliteal fossa and gives off branches to gastrocnemius, soleus, plantaris, and popliteus muscles. The tibial nerve travels in proximity to the posterior tibial artery. In the leg, it gives off branches to the flexor digitorum longus, tibialis posterior, and flexor hallucis longus. Distally in the foot, it branches to give rise to the medial and lateral plantar nerves, which provide sensation to the plantar surface of the foot. Injury to the tibial nerve results in deficits of plantar flexion, as well as anesthesia to the plantar surface of the foot.

77
Q

A 24-year-old man comes to the office 3 months after closed reduction of a right knee dislocation. His knee is stable, but he still depends on an ankle/foot orthosis for ambulation. Physical examination shows decreased light-touch sensation along the dorsolateral aspect of the foot. Ankle eversion is absent. Sensation and motor function are otherwise intact. Nerve conduction testing is most likely to demonstrate a block in which of the following nerves?

A) Common peroneal
B) Lateral plantar
C) Medial plantar
D) Posterior tibial
E) Superficial peroneal

A

The correct response is Option E.

Common peroneal nerve injuries involving motor function loss have been reported in up to 50% of knee dislocations. If isolated sensory disturbances are also included, the incidence of nerve injury approaches 75%. If no recovery is noted by 3 to 6 months following injury, then surgical treatment is warranted. Physical exam primarily determines the nerve to be explored, neurolysed, and possibly grafted, but nerve conduction studies can be useful pre- and intraoperatively.

The common peroneal nerve divides into three branches at the knee, an articular branch that innervates the joint capsule and lateral collateral ligament of the knee, the superficial, and deep branches. The superficial branch innervates the muscles of the lateral compartment of the leg and provides sensation to the lateral calf and dorsal foot. The deep branch innervates the anterior compartment and provides sensation to the first web space of the foot. The scenario given above is most consistent with compromise of the superficial peroneal nerve. If dorsiflexion of the ankle and toe extension had also been lost, then common peroneal nerve injury would have been suggested.

The posterior tibial nerve proper innervates the muscles of the posterior calf, mediating ankle plantar flexion and toe flexion. The medial and lateral plantar nerves are terminal branches of the posterior tibial nerve. They provide motor innervation to the deep plantar muscles of the foot and sensation to the plantar surface of the foot.

78
Q

A 25-year-old construction worker has a 4-cm-diameter posterior calcaneal ulcer with exposed bone on removal of a short-leg cast applied 6 weeks ago for an ankle fracture. Coverage with a lateral calcaneal artery flap is planned. The lateral calcaneal artery is usually the terminal branch of which of the following arteries?

A) Anterior tibial
B) Dorsalis pedis
C) Lateral malleolar
D) Peroneal

A

The correct response is Option D.

Anatomic dissections by Drs. Grabb and Argenta found that the lateral calcaneal artery is usually the terminal branch of the peroneal artery but occasionally may arise from the posterior tibial artery. The branches of the peroneal include the nutrient artery which supplies the fibula, the perforating branch which gives branches to the tarsus, the communicating branch and the lateral calcaneal.

79
Q

During harvest of a plantaris tendon graft, which of the following structures is at greatest risk for injury?

A) Dorsalis pedis artery
B) Extensor hallucis brevis muscle
C) Medial plantar artery
D) Sural nerve
E) Tibial nerve

A

The correct response is Option E.

When multiple tendon grafts are needed or when it is necessary to harvest grafts long enough to reach from the forearm to the fingertip, lower extremity tendon graft harvest is necessary. The plantaris tendon is a good source of tendon graft and is present in about 80% of limbs.

The graft is harvested through a vertical incision just anterior to the medial aspect of the Achilles tendon. Then the graft is followed proximally using either a tendon stripper or with further incisions. As such, dissection of the plantaris tendon begins behind the medial malleolus and close to the tibial nerve. The sural nerve lies about the lateral malleolus and thus is not likely to be inadvertently injured during plantaris harvest. The medial plantar artery is on the sole of the foot and would be distal to the field of dissection. The extensor hallucis brevis muscle is a small muscle that lies over the dorsum of the foot and thus, like the dorsalis pedis artery, would not be injured in the dissection.

80
Q

The primary blood supply to a free anterolateral thigh fasciocutaneous flap arises from vessels that perforate which of the following muscles?

A) Gluteus maximus
B) Rectus femoris
C) Sartorius
D) Tensor fascia lata
E) Vastus lateralis

A

The correct response is Option E.

The anterolateral thigh (ALT) flap has proven to be one of the most versatile free tissue transfers in reconstructive surgery. Based on perforators from the descending branch of the lateral circumflex femoral artery that traverse the vastus lateralis (VL) (80%) or the septum between the rectus femoris and VL (18-20%), this flap can be fashioned as large as 10 cm wide by 25 cm long. Occasionally, no large perforator will be identified during dissection; in this circumstance, the flap may be carried on multiple perforators along with the vastus lateralis muscle.

Branches of the lateral circumflex femoral artery also supply the sartorius (partial, as the supply is segmental), rectus femoris (descending branch), and tensor fascia lata (ascending branch). Vascular supply to the gluteus maximus arises from the superior and inferior gluteal arteries.

81
Q

A 30-year-old man is scheduled to undergo great toe-to-thumb transfer 7 months after traumatic amputation of the dominant thumb. During dissection of the toe, the first dorsal metatarsal artery is most likely to be found branching from which of the following vessels?

A) Deep plantar
B) Dorsalis pedis
C) Plantar arterial arch
D) Posterior tibial
E) Proper digital

A

The correct response is Option B.

The origin and course of the first dorsal metatarsal artery (FDMA) are key to dissecting the first or second toe and the variety of available toe flaps. This anatomy is quite variable. In two thirds of cases, this artery emanates from the dorsalis pedis artery as its distal continuation. This course can then be superficial, within, or deep to the interosseous muscle. However, in one third of patients, the metatarsal artery may arise from the deep plantar artery that communicates with the plantar arch or actually from the plantar arch itself, in which case the FDMA may be vestigial. In the latter two situations, the metatarsal artery passes plantar to the deep transverse metatarsal ligament. The proper digital arteries are the distal continuations of the FDMA. The posterior tibial artery runs longitudinally in a superficial plane to the forefoot on the plantar surface. It is the larger lateral plantar artery that travels deeply to become the plantar arterial arch.

82
Q

An otherwise healthy 47-year-old man is transferred to the hospital because of an infection of the leg. He sustained the initial injury in a fall 6 weeks ago that was treated with internal fixation. The infection is now under control, and the internal hardware has been removed. Examination shows a 9-cm bony defect of the lower extremity. Neurovascular status of the foot is normal. Angiography of both lower extremities shows no abnormalities. A photograph and x-ray study are shown. Which of the following considerations favors vascularized bone grafting in this patient?

A) Length of time since the initial injury
B) Mechanism of the injury
C) Method of injury stabilization
D) Patient age
E) Size of the bony defect

A

The correct response is Option E.

The injury described is a Gustilo IIIB lower extremity wound complicated by infection. The sequence of reconstruction is often bony stabilization and debridement until bacterial balance. Bony deficits can be reconstructed in a variety of ways, including non-vascularized grafts, vascularized grafts (free of pedicle), and bone transport.

Generally, for defects greater than 6 to 8 cm, vascularized bone grafting is indicated.

Other indications for vascularized bone grafting are the presence of infection and prior failure of conventional (non-vascularized) grafting.

Age of the patient is not a contraindication to reconstruction per se, as long as he or she is medically stable to undergo a prolonged operation.

Gustilo Classification
I: open fracture; clean; wound less than 1 cm
II: open fracture; wound greater than 1 cm
IIIA: open fracture; extensive soft-tissue injury but adequate tissue for coverage
IIIB: open fracture; extensive soft-tissue injury but inadequate tissue for coverage
IIIC: any of the above with a vascular (arterial) injury

83
Q

A 37-year-old man comes to the office because of wound breakdown 2 weeks after he sustained a calcaneal fracture. Orthopaedic stabilization was performed in the emergency department at the time of the injury. Examination today shows a 3 × 4-cm wound over the lateral calcaneus. Coverage with a propeller fasciocutaneous flap from the lateral leg is planned. Which of the following blood vessels supplies the perforators of this flap?

A) Anterior tibial artery
B) Lateral plantar artery
C) Lateral sural artery
D) Peroneal artery
E) Posterior tibial artery

A

The correct response is Option D.

Propeller flaps are a useful method of lower extremity reconstruction. These flaps were initially popularized by Teo and have been utilized for a variety of defects in the lower leg and foot. The propeller flap is based on perforating blood vessels from the peroneal artery to reconstruct lateral defects and perforators from the posterior tibial artery to reconstruct defects on the medial aspect of the leg wound. The propeller flaps can often replace the need for a sural artery, neurocutaneous artery flap or a free tissue transfer. Laterally based plantar flaps may be used to cover small defects on the weight-bearing surface of the foot but are not reliable for larger or lateral defects. The posterior and anterior tibial arteries do not have perforator in the desired location for a distally based propeller flap. The lateral sural vessels are too proximal for heel coverage.

84
Q

A 55-year-old man is evaluated in the emergency department for foot salvage after he was involved in a motorcycle collision. Flow is restored after 6 hours from the time of injury. On examination, the foot is cold, and no plantar sensation is noted. The posterior tibial nerve is disrupted. A temporary external fixator is placed. A photograph and an x-ray study are shown. Which of the following is the most appropriate classification of this injury and recommendation for management?

A) Gustilo type IIIB; amputation
B) Gustilo type IIIB; reconstruction
C) Gustilo type IIIC; amputation
D) Gustilo type IIIC; reconstruction

A

The correct response is Option C.

The patient described has a Gustilo IIIC injury. Based on the best available data, he should undergo amputation.

Ultimately, the choice to reconstruct versus amputate is a gestalt of the situation and the patient, as well as the capabilities of the hospital and the care team. In this case, the factors influencing the decision would be the warm ischemia time of 6 hours and severed posterior tibial nerve, as well as the extensive soft-tissue injury.

Some of the newer data suggests that absence of plantar sensation is no longer criteria for amputation in and of itself. However, an anatomically disrupted nerve in an adult strongly favors amputation.

In addition, there is evidence supporting the notion that limb salvage might involve less cost in the long term versus reconstruction.

Gustilo Classification
I: open fracture; clean; wound less than 1 cm
II: open fracture; wound greater than 1 cm
IIIA: open fracture; extensive soft-tissue injury but adequate tissue for coverage
IIIB: open fracture; extensive soft-tissue injury but inadequate tissue for coverage
IIIC: any of the above with a vascular (arterial) injury

85
Q

A 48-year-old man comes to the office because of drainage from the distal aspect of the wound 2 weeks after he underwent soft-tissue coverage with a free latissimus dorsi muscle flap for a degloving injury of the left lower extremity sustained in a motorcycle collision. Examination in the emergency department showed a Gustilo type IIIB tibia/fibula fracture and 6 cm of tibial loss. Debridement of the bone and stabilization with an external fixator were performed at the time of the injury. The patient underwent soft-tissue coverage 8 days after the injury. Examination today shows purulent drainage at the lower portion of the flap. Which of the following is the most likely cause of this complication?

A) Age of patient
B) Delay in soft-tissue coverage
C) Distal flap necrosis
D) Inadequate debridement

A

The correct response is Option D.

Posttraumatic reconstruction of the lower extremities with significant soft-tissue defects that expose bone, joints and tendons generally require free tissue transfer. The Gustilo-Anderson fracture classification system is widely used to describe the injury when associated with a long bone fracture. Gustilo type IIIB fractures are associated with periosteal stripping and exposure of bone with contamination. In 1986, Godina emphasized coverage of these soft-tissue defects within the first 72 hours of injury. Given the nature of these poly-trauma injuries, coverage of these wounds is not always possible in the given time frame. Staged debridement and negative pressure therapy have become common in the management of these injuries. Inadequate bone or soft-tissue debridement prior to reconstruction is a common cause of failure. The wound has to be free of all contaminants prior to reconstruction; thus, it is unlikely that an 8-day delay is the cause of breakdown. Distal flap necrosis would show more superficial breakdown and, in this case, is not the cause for a deep space infection. The patient’s age does not have any bearing on this complication.

86
Q

A 25-year-old man is brought to the emergency department after he sustained a knife wound to the right lower extremity. Examination shows numbness of the lateral aspect of the leg and weakness in plantar flexion and eversion of the foot. Which of the following nerves was most likely injured in this patient?

A) Femoral
B) Obturator
C) Peroneal
D) Sural
E) Tibial

A

The correct response is Option C.

The patient appears to demonstrate symptoms of a superficial peroneal nerve injury. The superficial peroneal nerve arises from the common peroneal nerve at the fibular neck. It supplies the lateral compartment of the leg, giving motor branches to peroneus longus and brevis, as well as sensory contribution to the lateral aspect of the leg. Injury to the superficial peroneal nerve results in anesthesia of the lateral aspect of the leg and weakness in eversion and plantar flexion of the foot.

The deep peroneal nerve arises from the common peroneal nerve at the fibular neck. It travels in the anterior compartment of the leg and gives branches to the tibialis anterior, extensor hallucis longus, and extensor digitorum longus and brevis, as well as peroneus tertius. The sensory distribution is in the area of the first web space. Injury to the deep peroneal nerve causes weakness in dorsiflexion of the foot.

The femoral nerve innervates muscles of the anterior thigh, including the quadriceps group, iliacus, and sartorius. Injury to the femoral nerve results in weakness of leg extension.

The obturator nerve provides innervation to the medial thigh muscles (adductor group), including adductor brevis, longus, and magnus, as well as the gracilis and obturator externus. The cutaneous branch provides sensation of the medial thigh. Injury to the obturator nerve results in weakness in thigh adduction and sensory deficits in the medial thigh.

The sural nerve travels on the posterior aspect of the leg between the lateral malleolus and calcaneus. It provides sensation to the lateral aspect of the foot and does not have a motor component. It is commonly sampled in nerve biopsy and used as a source of nerve graft.

Injury or sacrifice of the sural nerve would result in numbness of the lateral foot.

The tibial nerve is a branch of the sciatic nerve. It travels through the popliteal fossa and gives off branches to gastrocnemius, soleus, plantaris, and popliteus muscles. The tibial nerve travels in proximity to the posterior tibial artery. In the leg, it gives off branches to the flexor digitorum longus, tibialis posterior, and flexor hallucis longus. Distally in the foot, it branches to give rise to the medial and lateral plantar nerves, which provide sensation to the plantar surface of the foot. Injury to the tibial nerve results in deficits of plantar flexion, as well as anesthesia to the plantar surface of the foot.

87
Q

A 24-year-old man comes to the office 3 months after closed reduction of a right knee dislocation. His knee is stable, but he still depends on an ankle/foot orthosis for ambulation. Physical examination shows decreased light-touch sensation along the dorsolateral aspect of the foot. Ankle eversion is absent. Sensation and motor function are otherwise intact. Nerve conduction testing is most likely to demonstrate a block in which of the following nerves?

A) Common peroneal
B) Lateral plantar
C) Medial plantar
D) Posterior tibial
E) Superficial peroneal

A

The correct response is Option E.

Common peroneal nerve injuries involving motor function loss have been reported in up to 50% of knee dislocations. If isolated sensory disturbances are also included, the incidence of nerve injury approaches 75%. If no recovery is noted by 3 to 6 months following injury, then surgical treatment is warranted. Physical exam primarily determines the nerve to be explored, neurolysed, and possibly grafted, but nerve conduction studies can be useful pre- and intraoperatively.

The common peroneal nerve divides into three branches at the knee, an articular branch that innervates the joint capsule and lateral collateral ligament of the knee, the superficial, and deep branches. The superficial branch innervates the muscles of the lateral compartment of the leg and provides sensation to the lateral calf and dorsal foot. The deep branch innervates the anterior compartment and provides sensation to the first web space of the foot. The scenario given above is most consistent with compromise of the superficial peroneal nerve. If dorsiflexion of the ankle and toe extension had also been lost, then common peroneal nerve injury would have been suggested.

The posterior tibial nerve proper innervates the muscles of the posterior calf, mediating ankle plantar flexion and toe flexion. The medial and lateral plantar nerves are terminal branches of the posterior tibial nerve. They provide motor innervation to the deep plantar muscles of the foot and sensation to the plantar surface of the foot.

88
Q

A 25-year-old construction worker has a 4-cm-diameter posterior calcaneal ulcer with exposed bone on removal of a short-leg cast applied 6 weeks ago for an ankle fracture. Coverage with a lateral calcaneal artery flap is planned. The lateral calcaneal artery is usually the terminal branch of which of the following arteries?

A) Anterior tibial
B) Dorsalis pedis
C) Lateral malleolar
D) Peroneal

A

The correct response is Option D.

Anatomic dissections by Drs. Grabb and Argenta found that the lateral calcaneal artery is usually the terminal branch of the peroneal artery but occasionally may arise from the posterior tibial artery. The branches of the peroneal include the nutrient artery which supplies the fibula, the perforating branch which gives branches to the tarsus, the communicating branch and the lateral calcaneal.

89
Q

During harvest of a plantaris tendon graft, which of the following structures is at greatest risk for injury?

A) Dorsalis pedis artery
B) Extensor hallucis brevis muscle
C) Medial plantar artery
D) Sural nerve
E) Tibial nerve

A

The correct response is Option E.

When multiple tendon grafts are needed or when it is necessary to harvest grafts long enough to reach from the forearm to the fingertip, lower extremity tendon graft harvest is necessary. The plantaris tendon is a good source of tendon graft and is present in about 80% of limbs.

The graft is harvested through a vertical incision just anterior to the medial aspect of the Achilles tendon. Then the graft is followed proximally using either a tendon stripper or with further incisions. As such, dissection of the plantaris tendon begins behind the medial malleolus and close to the tibial nerve. The sural nerve lies about the lateral malleolus and thus is not likely to be inadvertently injured during plantaris harvest. The medial plantar artery is on the sole of the foot and would be distal to the field of dissection. The extensor hallucis brevis muscle is a small muscle that lies over the dorsum of the foot and thus, like the dorsalis pedis artery, would not be injured in the dissection.

90
Q

The primary blood supply to a free anterolateral thigh fasciocutaneous flap arises from vessels that perforate which of the following muscles?

A) Gluteus maximus
B) Rectus femoris
C) Sartorius
D) Tensor fascia lata
E) Vastus lateralis

A

The correct response is Option E.

The anterolateral thigh (ALT) flap has proven to be one of the most versatile free tissue transfers in reconstructive surgery. Based on perforators from the descending branch of the lateral circumflex femoral artery that traverse the vastus lateralis (VL) (80%) or the septum between the rectus femoris and VL (18-20%), this flap can be fashioned as large as 10 cm wide by 25 cm long. Occasionally, no large perforator will be identified during dissection; in this circumstance, the flap may be carried on multiple perforators along with the vastus lateralis muscle.

Branches of the lateral circumflex femoral artery also supply the sartorius (partial, as the supply is segmental), rectus femoris (descending branch), and tensor fascia lata (ascending branch). Vascular supply to the gluteus maximus arises from the superior and inferior gluteal arteries.

91
Q

A 30-year-old man is scheduled to undergo great toe-to-thumb transfer 7 months after traumatic amputation of the dominant thumb. During dissection of the toe, the first dorsal metatarsal artery is most likely to be found branching from which of the following vessels?

A) Deep plantar
B) Dorsalis pedis
C) Plantar arterial arch
D) Posterior tibial
E) Proper digital

A

The correct response is Option B.

The origin and course of the first dorsal metatarsal artery (FDMA) are key to dissecting the first or second toe and the variety of available toe flaps. This anatomy is quite variable. In two thirds of cases, this artery emanates from the dorsalis pedis artery as its distal continuation. This course can then be superficial, within, or deep to the interosseous muscle. However, in one third of patients, the metatarsal artery may arise from the deep plantar artery that communicates with the plantar arch or actually from the plantar arch itself, in which case the FDMA may be vestigial. In the latter two situations, the metatarsal artery passes plantar to the deep transverse metatarsal ligament. The proper digital arteries are the distal continuations of the FDMA. The posterior tibial artery runs longitudinally in a superficial plane to the forefoot on the plantar surface. It is the larger lateral plantar artery that travels deeply to become the plantar arterial arch.

92
Q

An otherwise healthy 47-year-old man is transferred to the hospital because of an infection of the leg. He sustained the initial injury in a fall 6 weeks ago that was treated with internal fixation. The infection is now under control, and the internal hardware has been removed. Examination shows a 9-cm bony defect of the lower extremity. Neurovascular status of the foot is normal. Angiography of both lower extremities shows no abnormalities. A photograph and x-ray study are shown. Which of the following considerations favors vascularized bone grafting in this patient?

A) Length of time since the initial injury
B) Mechanism of the injury
C) Method of injury stabilization
D) Patient age
E) Size of the bony defect

A

The correct response is Option E.

The injury described is a Gustilo IIIB lower extremity wound complicated by infection. The sequence of reconstruction is often bony stabilization and debridement until bacterial balance. Bony deficits can be reconstructed in a variety of ways, including non-vascularized grafts, vascularized grafts (free of pedicle), and bone transport.

Generally, for defects greater than 6 to 8 cm, vascularized bone grafting is indicated.

Other indications for vascularized bone grafting are the presence of infection and prior failure of conventional (non-vascularized) grafting.

Age of the patient is not a contraindication to reconstruction per se, as long as he or she is medically stable to undergo a prolonged operation.

Gustilo Classification
I: open fracture; clean; wound less than 1 cm
II: open fracture; wound greater than 1 cm
IIIA: open fracture; extensive soft-tissue injury but adequate tissue for coverage
IIIB: open fracture; extensive soft-tissue injury but inadequate tissue for coverage
IIIC: any of the above with a vascular (arterial) injury

93
Q

A 37-year-old man comes to the office because of wound breakdown 2 weeks after he sustained a calcaneal fracture. Orthopaedic stabilization was performed in the emergency department at the time of the injury. Examination today shows a 3 × 4-cm wound over the lateral calcaneus. Coverage with a propeller fasciocutaneous flap from the lateral leg is planned. Which of the following blood vessels supplies the perforators of this flap?

A) Anterior tibial artery
B) Lateral plantar artery
C) Lateral sural artery
D) Peroneal artery
E) Posterior tibial artery

A

The correct response is Option D.

Propeller flaps are a useful method of lower extremity reconstruction. These flaps were initially popularized by Teo and have been utilized for a variety of defects in the lower leg and foot. The propeller flap is based on perforating blood vessels from the peroneal artery to reconstruct lateral defects and perforators from the posterior tibial artery to reconstruct defects on the medial aspect of the leg wound. The propeller flaps can often replace the need for a sural artery, neurocutaneous artery flap or a free tissue transfer. Laterally based plantar flaps may be used to cover small defects on the weight-bearing surface of the foot but are not reliable for larger or lateral defects. The posterior and anterior tibial arteries do not have perforator in the desired location for a distally based propeller flap. The lateral sural vessels are too proximal for heel coverage.

94
Q

is restored after 6 hours from the time of injury. On examination, the foot is cold, and no plantar sensation is noted. The posterior tibial nerve is disrupted. A temporary external fixator is placed. A photograph and an x-ray study are shown. Which of the following is the most appropriate classification of this injury and recommendation for management?

A) Gustilo type IIIB; amputation
B) Gustilo type IIIB; reconstruction
C) Gustilo type IIIC; amputation
D) Gustilo type IIIC; reconstruction

A

The correct response is Option C.

The patient described has a Gustilo IIIC injury. Based on the best available data, he should undergo amputation.

Ultimately, the choice to reconstruct versus amputate is a gestalt of the situation and the patient, as well as the capabilities of the hospital and the care team. In this case, the factors influencing the decision would be the warm ischemia time of 6 hours and severed posterior tibial nerve, as well as the extensive soft-tissue injury.

Some of the newer data suggests that absence of plantar sensation is no longer criteria for amputation in and of itself. However, an anatomically disrupted nerve in an adult strongly favors amputation.

In addition, there is evidence supporting the notion that limb salvage might involve less cost in the long term versus reconstruction.

Gustilo Classification
I: open fracture; clean; wound less than 1 cm
II: open fracture; wound greater than 1 cm
IIIA: open fracture; extensive soft-tissue injury but adequate tissue for coverage
IIIB: open fracture; extensive soft-tissue injury but inadequate tissue for coverage
IIIC: any of the above with a vascular (arterial) injury

95
Q

A 48-year-old man comes to the office because of drainage from the distal aspect of the wound 2 weeks after he underwent soft-tissue coverage with a free latissimus dorsi muscle flap for a degloving injury of the left lower extremity sustained in a motorcycle collision. Examination in the emergency department showed a Gustilo type IIIB tibia/fibula fracture and 6 cm of tibial loss. Debridement of the bone and stabilization with an external fixator were performed at the time of the injury. The patient underwent soft-tissue coverage 8 days after the injury. Examination today shows purulent drainage at the lower portion of the flap. Which of the following is the most likely cause of this complication?

A) Age of patient
B) Delay in soft-tissue coverage
C) Distal flap necrosis
D) Inadequate debridement

A

The correct response is Option D.

Posttraumatic reconstruction of the lower extremities with significant soft-tissue defects that expose bone, joints and tendons generally require free tissue transfer. The Gustilo-Anderson fracture classification system is widely used to describe the injury when associated with a long bone fracture. Gustilo type IIIB fractures are associated with periosteal stripping and exposure of bone with contamination. In 1986, Godina emphasized coverage of these soft-tissue defects within the first 72 hours of injury. Given the nature of these poly-trauma injuries, coverage of these wounds is not always possible in the given time frame. Staged debridement and negative pressure therapy have become common in the management of these injuries. Inadequate bone or soft-tissue debridement prior to reconstruction is a common cause of failure. The wound has to be free of all contaminants prior to reconstruction; thus, it is unlikely that an 8-day delay is the cause of breakdown. Distal flap necrosis would show more superficial breakdown and, in this case, is not the cause for a deep space infection. The patient’s age does not have any bearing on this complication.

96
Q

Which of the following is the most likely cause of congenital constriction band syndrome of the lower extremity?

A) Defect in the zone of polarizing activity
B) Deletion of the gene responsible for the apical ectodermal ridge
C) Early amnion rupture
D) Exposure to retinoic acid
E) Use of thalidomide during pregnancy

A

The correct response is Option C.

Early amnion rupture with subsequent entanglement of fetal parts (mostly limbs and appendages) by amniotic strands is the primary theory of pathogenesis.

A wide spectrum of clinical deformities is encountered and range from simple ring constrictions to major visceral defects. Lower extremity limb malformations are extremely common and consist of asymmetric digital ring constrictions, distal atrophy, congenital intrauterine amputations, acrosyndactyly, lymphedema, and clubfoot.

Amniotic band syndrome is not a rare anomaly, as first described more than 150 years ago, and appears to be rising. Once believed to have an incidence of 1:100,000, recent literature supports the incidence today as 1:1200 to 1:5000 births.

No distinct sex predilection has been determined. Nearly 60% of the cases documented have some sort of abnormal gestation history. Prenatal risk factors associated with amniotic band syndrome include prematurity (less than 37 weeks), low birth weight (less than 2500 g), maternal illness during pregnancy, maternal drug exposure, and maternal hemorrhage. Attempted abortion in the first trimester is also a highly associated finding. Family history seldom shows any direct inheritance pattern since the syndrome occurs in no particular association with known genetic or chromosomal disorders.

The zone of polarizing activity signals the developing limb bud towards anterior/posterior polarity and does not result in truncation.

Proximodistal limb growth is the result of the apical ectodermal ridge. Deletion of the gene responsible for the apical ectodermal ridge will result in shortening of the limb but not in congenital constriction band syndrome. Experiments in which the apical ectodermal ridge has been removed show truncated limb growth. Scientific evidence supports that the pattern of limb anomalies in the offspring of mothers exposed to exogenous retinoids, such as retinoic acid, causes interference with apical ectodermal ridge function, resulting in similar deformities.

Thalidomide results in various limb deformities, including phocomelia, dysmelia, amelia, and bone hypoplasticity, with as little as a single dose of thalidomide during gestation. It does not, however, result in constriction bands. It is postulated that thalidomide-associated malformations are the result of the drug’s interference with vasculogensis.

97
Q

A 17-year-old boy sustains an avulsion injury to the anterior ankle with exposed tendon in a motorcycle collision. The wound is evaluated and reconstruction with a reverse sural artery flap is planned. During elevation of the flap, which of the following is most likely to compromise flap viability?

A ) Injury to the lesser saphenous vein
B ) Injury to the median superficial sural artery
C ) Injury to the sural nerve
D ) Ligation of the gastrocnemius muscle perforators
E ) Ligation of the peroneal artery perforators

A

The correct response is Option E.

The predominant blood supply to the reverse sural artery flap relies on perforators from the peroneal artery, which primarily originate at a point 5 cm proximal to the lateral malleolus.

The median superficial sural artery and perforators from the gastrocnemius are involved in anterograde blood supply to the flap. Thus, these vessels are typically ligated in the reverse form of the flap.

Likewise, injury to the lesser saphenous vein would not cause significant problems with venous outflow. In dissections and venous flow studies, small concomitant veins were found along both sides of the lesser saphenous vein and were considered to be venae comitantes of accompanying arteries of the vein. These accompanying veins played a role in bypassing the valves of the lesser saphenous vein.

The sural artery flap can be harvested as a neural-veno-adipofascial flap. In this context, the sural nerve can used to provide sensation to the flap. This nerve does not provide primary blood supply to the flap, and injury would not compromise flap viability.

98
Q

A 55-year-old man comes for evaluation of a nonhealing foot ulcer. He has a 10-year history of type 1 diabetes mellitus. Physical examination shows a deep, 4-cm plantar ulcer at the great toe metatarsal head. Which of the following is the most likely cause of this ulcer?

A ) Atherosclerosis
B ) Lymphatic obstruction
C ) Peripheral neuropathy
D ) Poor hygiene
E ) Venous stasis

A

The correct response is Option C.

Sensory neuropathy leads to loss of protective sensations. This then leads to neuropathic imbalance with loss of coordination which increases mechanical stresses, unperceived trauma, Charcot foot, and ulcers. Sympathetic neuropathy leads to warm, dry feet which are prone to skin breakdowns. Patients with diabetes who also have peripheral vascular disease are more prone to ischemic ulcers. Patients with diabetic foot ulcers have not been associated with large-vessel atherosclerosis.

99
Q

A 25-year-old man undergoes open reduction and internal fixation of a Gustilo Type IIIB tibia-fibula fracture. Coverage of the resultant soft-tissue defect with a free flap is planned. Which of the following is the ideal length of time after the injury that this procedure should be performed?

A ) 3 Days
B ) 10 Days
C ) 21 Days
D ) 60 Days
E ) 90 Days

A

The correct response is Option A.

Godina defined the timing of posttraumatic microsurgical lower extremity reconstruction in 1986 with recommendations for flap coverage of Gustilo Type IIIB/C fractures within 72 hours of injury. Godina’s study showed the highest risk of infection and flap loss in the delayed period (72 hours to 90 days). Subsequent authors have also cited lower rates of flap loss and infection when repair was performed.

Many authors have reported good flap survival when surgery is done within 3 days and 3 months after injury, but it is accepted that this delayed time frame necessitates more meticulous dissection and going well outside the zone of injury to a patent venous system to decrease the incidence of free flap loss.

100
Q

A 66-year-old man comes to the office with a 2-year history of progressive ulceration at the right medial malleolus. He says he has marked calf pain when he is active, and that it has worsened progressively over time. Medical history includes cardiovascular disease and a 40-year history of smoking. Physical examination shows that the wound margins are well demarcated with a yellow fibrinous wound base. Which of the following is the most appropriate management of this patient?s condition?

A ) Biopsy of the wound edge
B ) Debridement and dressing changes
C ) Debridement and local flap coverage
D ) Revascularization by vascular surgery
E ) Use of topical debriding agents

A

The correct response is Option D.

Arterial ulcers most commonly occur in male patients with atherosclerosis, usually in association with cardiovascular disease, diabetes, hypertension, or smoking. Patients typically have symptoms of claudication and rest pain. The leg pain improves when the leg is dependent and is exacerbated when the leg is elevated. The base of the arterial ulcer generally does not bleed and has a ?punched-out? appearance. Ulcers associated with an ankle brachial index of less than 0.45 (normal range 0.90 to 1.30) generally do not heal without revascularization. Aggressive debridement would create a larger wound without the potential to heal. Continued wound care would provide no long-term benefit. The clinical picture described is not consistent with malignant degeneration of the wound.

101
Q

An otherwise healthy 25-year-old man is evaluated because of footdrop and paresthesia 4 weeks after undergoing open reduction and internal fixation of a fracture of the proximal tibia. Electrodiagnostic testing shows no conduction of the nerve. Which of the following is the most appropriate next step in management?

A ) Ankle arthrodesis
B ) Peroneal nerve decompression
C ) Posterior tibial nerve decompression
D ) Posterior tibial tendon transfer
E ) Repeat electrodiagnostic testing in 4 weeks

A

The correct response is Option E.

Peroneal nerve palsy may lead to severe disability with footdrop and paresthesia. Traumatic peroneal palsy may result from supracondylar femoral fracture, knee dislocation, and proximal tibial fracture involving both motor and sensory changes. On examination, the motor deficit manifests as a footdrop, and the sensory deficit results in a loss of sensation along the dorsum of the foot. Unlike the loss of sensation associated with the peroneal nerve, the posterior tibial nerve is clinically morbid, as it is on the plantar aspect of the foot. The chronic injury may result in wounding of the plantar aspect of the foot, which may ultimately result in amputation.

Electrodiagnostic testing should be performed within 1 month of the injury to confirm sensory and motor deficits. In patients with partial nerve palsy, greater than 80% will recover completely and can be expected to recover with observation alone. A patient with complete palsy will have a much worse prognosis, with only 40% of patients with a functional recovery. If there is no neurologic improvement after 2 to 3 months, operative decompression should be performed. If the nerve is noted to be in continuity, and nerve action potentials are transmitted across a lesion, then there may be continuity of the nerve, and neurolysis is performed. If there is a transected nerve, a direct repair or nerve grafting may be performed.

When neurolysis and nerve grafting do not provide satisfactory results, the remaining surgical options are salvage procedures such as tendon transfer or arthrodesis. Posterior tibial tendon transfer is the most commonly used tendon transfer for this injury.

102
Q

A 58-year-old man comes to the office for follow-up 3 weeks after undergoing mandibular reconstruction with a fibular free flap. The patient reports numbness of the operated extremity and says he has difficulty walking. Which of the following sensory and motor deficits is most likely in this patient?

A

The correct response is Option A.

The injury in the patient described is to the common peroneal nerve.

Both the first web space of the foot and the extensor hallucis longus and tibialis anterior muscles are innervated by branches of the common peroneal nerve. The finding of absence of sensation over the medial aspect of the calf and weakness of the soleus and extensor digitorum brevis muscles is eliminated by numbness of the medial calf, which is innervated by the saphenous nerve, a branch of the femoral nerve. Absence of sensation over the medial malleolus and heel with weakness of the soleus and tibialis anterior muscles is eliminated by numbness of the medial malleolus, which is also innervated by the saphenous nerve. In addition, the soleus is innervated by the tibial nerve. The finding of absence of sensation in the third web space of the foot and weakness of the soleus and extensor digitorum brevis muscles is eliminated by the soleus muscle. The finding of absence of sensation over the upper lateral calf and weakness of the flexor hallucis longus and extensor digitorum brevis muscles is eliminated by the flexor hallucis longus, which is innervated by the tibial nerve.

103
Q

A 46-year-old man is brought to the emergency department after sustaining an open comminuted fracture of the left tibia during a motor vehicle collision. Physical examination shows a wound greater than 10 cm with marked periosteal stripping near the mid shaft. Which of the following is the appropriate Gustilo classification of this patient’s injury?

A ) I
B ) II
C ) IIIA
D ) IIIB
E ) IIIC

A

The correct response is Option D.

The Gustilo classification of open fractures is based on the severity of soft-tissue injury and the presence or absence of concurrent vascular injury. A higher grade implies a more severe injury, and correlates with a greater incidence of complications and need for amputation.

The patient in this item has a wound greater than 10 cm and a heavily comminuted tibia fracture. There is no mention of vascular injury. He has a Gustilo type IIIB injury.

The Gustilo grading scale:

TypeFindings

IClean wound bed, simple/minimally comminuted bone injury, wound < 1 cm

IIWound contaminated, moderate comminution of bone, wound > 1 cm

IIIAWound highly contaminated, severe comminution, wound 1 to 10 cm

IIIBWound highly contaminated, severe comminution, wound > 10 cm

IIICMajor vascular injury requiring repair for limb salvage

104
Q

A 46-year-old man is brought to the emergency department after sustaining an open comminuted fracture of the left tibia during a motor vehicle collision. Physical examination shows a wound greater than 10 cm with marked periosteal stripping near the mid shaft. Which of the following is the appropriate Gustilo classification of this patient?s injury? A ) I
B ) II
C ) IIIA
D ) IIIB
E ) IIIC

A

The correct response is Option D.

The Gustilo classification of open fractures is based on the severity of soft-tissue injury and the presence or absence of concurrent vascular injury. A higher grade implies a more severe injury, and correlates with a greater incidence of complications and need for amputation.

The patient in this item has a wound greater than 10 cm and a heavily comminuted tibia fracture. There is no mention of vascular injury. He has a Gustilo type IIIB injury. The Gustilo grading scale:

TypeFindings

IClean wound bed, simple/minimally comminuted bone injury, wound < 1 cm

IIWound contaminated, moderate comminution of bone, wound > 1 cm

IIIAWound highly contaminated, severe comminution, wound 1 to 10 cm

IIIBWound highly contaminated, severe comminution, wound > 10 cm

IIICMajor vascular injury requiring repair for limb salvage

105
Q

A 23-year-old man is brought to the emergency department 4 hours after sustaining an open tibia and fibula fracture to the middle third of the leg in a motorcycle collision. A proximally based soleus flap is chosen to reconstruct the defect. Which of the following arteries is the most likely blood supply of this flap?

A ) Anterior tibial
B ) Inferior geniculate
C ) Peroneal
D ) Profunda femoris
E ) Superior geniculate

A

The correct response is Option C.

The soleus muscle is a broad, powerful muscle of the posterior calf which, along with the gastrocnemius muscle, forms the triceps surae. The soleus takes its origin from the upper third of the fibula and medial border of the tibia and inserts into the calcaneus with the gastrocnemius muscle by way of the Achilles tendon. The blood supply of the soleus muscle is from the peroneal artery proximally and the posterior tibial artery distally. This soleus muscle flap is used as a pedicle muscle flap for coverage of defects of the middle third of the lower leg.

106
Q

Which of the following is the most likely indication to consider replantation of a lower extremity?

A ) Bilateral sharp injury
B ) Crush mechanism of injury
C ) Ischemia time over 8 hours
D ) Multilevel injury
E ) Patient over age 50 years

A

The correct response is Option A.

The indications for lower extremity replantation are limited. The best candidate for a lower extremity replantation would be a young patient who has had both legs amputated with a clean mechanism. Contraindications for lower extremity replantation include crush mechanism of injury, ischemia time over 8 hours, multiple-level injury, poor baseline health, and a patient of advanced age.

107
Q

A 17-year-old boy undergoes resection of the right distal femur to treat osteosarcoma. A large allogeneic corticocancellous bone graft is used for skeletal reconstruction of the 10-cm bony defect. An intramedullary vascularized fibular free flap is also placed in the construct. Which of the following is the most likely purpose of this additional procedure?

A) Decrease the immunogenicity of the allograft
B) Decrease the rate of infection
C) Increase creeping substitution in the allograft
D) Increase the immediate strength of the construct
E) Shorten union time

A

The correct response is Option E.

The most likely reason to place an intramedullary fibular free flap in the allogeneic bone graft is to shorten union time. Free fibular flaps contain an intrinsic blood supply and osteogenic cells. These offer an alternative to the allograft and the capability of osteogenesis through osteoinduction. Thus, the biologic advantage of fibular free flaps is a shortened union time compared with the allograft, which solely depends upon creeping substitution for healing. Union times may be as short as 3 to 5 months. The healing of massive allografts is generally slow, superficial, and incomplete. The union times for allograft healing have been reported at 14 to 23 months for intercalary defects.

Nonvascularized massive allografts provide a biologic spacer with strong cortical bone. These grafts give great strength to the construct; however, despite this advantage they have many disadvantages, including their lack of blood supply, lack of osteogenic cells, and potential for immunologic reaction. The fibula may add some strength to the overall construct but this is not the primary reason for its use in the scenario described. It also does not mitigate any potential antigenicity that the graft may have. The graft heals by creeping substitution and as such is only osteoconductive, unlike vascularized bone. This process in the allograft remains unchanged by the presence of the fibular flap; however, it does provide the additional process of osteoinduction as described above, which helps in healing. It is the avascular status of the allograft that predisposes these reconstructions to infection, not the dead space in the medullary canal.

108
Q

A 67-year-old man comes to the office because of an open wound of the right groin and exposed distal anastomosis of a vascular graft 3 weeks after aortofemoral bypass grafting. Reconstruction with a sartorius muscle flap is planned to cover the graft. The blood supply of the sartorius muscle flap originates directly from which of the following arteries?

A) Deep circumflex iliac
B) Lateral circumflex femoral
C) Medial circumflex femoral
D) Profunda femoris
E) Superficial femoral

A

The correct response is Option E.

The sartorius muscle has a Type IV vascularization pattern consisting of 8 to 10 pedicles from the superficial femoral vessel, which enter the muscle medially. The other arteries do not contribute to the blood supply of the sartorius muscle.

109
Q

A 70-year-old man undergoes open reduction and internal fixation of an open fracture of the ankle. Debridement of nonviable tissue results in the exposure of the joint and hardware. Coverage with a reverse sural artery flap is planned. A photograph is shown. Which of the following is the most likely adverse outcome associated with the use of this flap?

A) Infection
B) Insufficient bulk
C) Insufficient flap coverage
D) Partial flap loss
E) Wound dehiscence

A

The correct response is Option D.

The utility of the flap has been proven in both healthy and compromised wounds; neither peripheral arterial disease nor diabetes has precluded its success. Of the possible complications, partial flap loss has occurred most commonly. As a result, numerous modifications to the flap have been proposed. These include maintaining a narrow (2 cm wide) pedicle, including a cutaneous ?tail? along the length of the pedicle and maintaining a mesentery between the sural nerve and the deep fascia.

Infection rates have been low, ranging from 0 to 2.5%. The fasciocutaneous variety of the reverse sural flap is often malleable enough to conform to most defects. When a deeper defect exists, a fasciomusculocutaneous variation of the flap has been designed to carry a portion of the gastrocnemius muscle via one of the larger proximal perforators typically found deep to the cutaneous paddle in the proximal leg.

Cutaneous paddles have been harvested with dimensions of up to 12 × 15 cm, allowing coverage of most ankle and heel wounds.

Reconstruction of larger donor defects with split-thickness skin grafts makes dehiscence unlikely.

110
Q

A 33-year-old woman is brought to the emergency department after sustaining a 6 × 8-cm soft-tissue defect over the tibial tuberosity after being hit by a motor vehicle while riding her bicycle. Physical examination shows exposed bone at the proximal third of the leg. Which of the following muscle flaps is most appropriate for this patient?

A) Lateral gastrocnemius
B) Lateral soleus
C) Medial gastrocnemius
D) Medial soleus
E) Sartorius

A

The correct response is Option C.

The gastrocnemius is a powerful muscle in the superficial posterior compartment of the leg that acts to plantar flex the foot at the ankle joint and flex the leg at the knee joint. It is involved in standing, walking, running, and jumping.

The lateral head originates from the lateral condyle of the femur, while the medial head originates from the medial condyle of the femur. Its other end forms a common tendon with the soleus muscle; this tendon is known as the calcaneal tendon or ?Achilles tendon? and inserts onto the posterior surface of the calcaneus.The medial head is longer, and its muscular fibers extend more inferiorly. The gastrocnemius can be harvested as a muscular or musculocutaneous flap if the soleus is intact and plantar flexion of the foot will be preserved. Medial and lateral heads may be used independently based on the tissue defect present. Footdrop is possible with the use of the lateral muscle belly.The soleus is the other powerful muscle of the superficial posterior compartment of the leg. With the gastrocnemius, it acts to plantar flex the foot at the ankle joint. Specifically, the soleus plays an important role in standing; if not for its constant pull, the body would fall forward. The muscle originates from the posterior surfaces of the head of the fibula and its upper quarter, as well as the middle third of the medial border of the tibia. Its other end forms a common calcaneal tendon with the gastrocnemius muscle.The soleus muscle, when freed from its insertion on the Achilles tendon and based proximally, covers defects in the middle third of the tibia. It may also be used as a hemisoleus to cover distal third tibia defects as well.The sartorius flap may be raised as a muscle or myocutaneous flap based on segmental branches of the superficial femoral artery and vein. Because it has a Type IV segmental blood supply, the arc of rotation both superiorly and inferiorly is limited. Its best indication is for coverage of the femoral vessels.

111
Q

A 40-year-old woman is referred by her orthopedic surgeon for reconstruction because of an exquisitely tender scar over the site of the anterolateral portal for arthroscopy of the left ankle. Examination shows a 1-cm scar over the lateral malleolus. Tinel sign is present over the scar. Sensation is diminished over the middle part of the dorsum of the foot. Which of the following nerves is most likely to have been injured during the arthroscopy?

A) Lateral malleolar
B) Medial plantar
C) Saphenous
D) Superficial peroneal
E) Sural

A

The correct response is Option D.

In the patient described, the dermatomal distribution of numbness of the middle of the dorsum of the foot suggests injury of the superficial peroneal nerve (SPN). The SPN supplies the sensation to the middle portion of the dorsum of the foot, except for the skin in the great toe web space (which is innervated by the deep peroneal nerve). The lateral malleolar nerve is a terminal branch nerve supplying the skin of the lateral malleolus. The medial plantar nerve supplies the medial three fourths of the plantar surface of the foot. The saphenous nerve travels to the dorsum of the foot, medial malleolus, and the area of the head of the first metatarsal. At the level of the ankle, the saphenous nerve is found between the medial malleolus and the anterior tibial tendon, just lateral to the saphenous vein. The sural nerve is located at an average of 7 mm posterior to the lateral malleolus and supplies sensation to the lateral aspect of the foot. A diagram is shown.

According to one study, in ankle arthroscopy the risk of SPN injury is maximal in the 0 to 3 mm lateral to the peroneus tertius tendon. To avoid injury to the SPN, the safest placement of the anterolateral portal is 4 mm lateral to the peroneus tertius tendon.

In another study, in 82% of specimens, the SPN ran between the lateral border of the talocrural joint and the peroneus tertius tendon at the talocrural joint level, where the anterolateral portal was placed. Therefore, the SPN was at high risk for injury with anterolateral portal placement.

112
Q

A 62-year-old man is brought to the emergency department by helicopter after sustaining severe injuries to the head, neck, and right femur during a motor vehicle collision. The patient’s condition is stabilized, and the femur is temporarily reduced and splint immobilized. Peripheral pulses in the right leg are not palpable and capillary refill is noted; handheld Doppler shows weak pulses. Which of the following is the most appropriate next step to establish lower extremity vascular injury in this patient?

A) CT angiography
B) Doppler ultrasonography
C) Measurement of ankle brachial index
D) Serial physical examinations

A

The correct response is Option A.

As with many patients who have sustained severe upper or lower extremity trauma, the vascular status of the limb in the patient described is in question. Because of significant collateral blood flow in the upper and lower extremities, capillary refill and handheld Doppler tones can often be found even with complete disruption of major arteries. Although traditional angiography is known as the ?gold standard? for the diagnosis of vascular injuries, it is not without its difficulties. A special suite, technicians, and physicians are needed to perform traditional angiography, and the potential for morbidity has been noted. As a result, CT angiography is fast becoming the new ?gold standard? for the diagnosis of vascular injuries. Coupled with the fact that many trauma patients will be brought to the CT suite for other injuries, CT angiography is a rapid and natural next step to be taken when the head or abdomen is being scanned. Serial physical examination, ankle brachial index, and Doppler ultrasonography are adequate techniques, but they may be operator-dependent or sometimes have difficulty localizing the actual injury. Both traditional and CT angiography will localize the injury, but, for obvious reasons, CT angiography has overtaken traditional angiography in the diagnosis of acute vascular injury in the trauma patient.

113
Q

A 25-year-old man is brought to the emergency department after he was hit by a motor vehicle while walking across the street. Physical examination shows a Gustilo Type IIIB open fracture of the tibia. Reconstruction with an anterolateral thigh flap is planned. An incision between which of the following structures is the most appropriate approach to access the posterior tibial vessels?

A) Lateral malleolus and the Achilles tendon
B) Lateral malleolus and the extensor hallucis longus tendon
C) Medial malleolus and the Achilles tendon
D) Tibialis anterior and the extensor hallucis longus tendons

A

The correct response is Option C.

The sural nerve is located at the distal leg between the lateral malleolus and the Achilles tendon.

The greater saphenous vein is located between the medial malleolus and the extensor hallucis longus tendon.

The posterior tibial vessels are located between the medial malleolus and the Achilles tendon.

The dorsalis pedis artery is located between the tendons of the tibialis anterior and the extensor hallucis longus.

114
Q

An otherwise healthy 35-year-old man is brought to the emergency department after he collided with a parked bus while riding his motorcycle at 20 mph. On admission, he is cleared by the trauma service to undergo orthopedic exploration of an open fracture of the tibia and fibula. The surgeon achieves external fixation and begins washout of a 15-cm anterior wound with exposed denuded bone. A consultation regarding initial evaluation for soft-tissue coverage is requested. Which of the following additional findings is most likely to lead to the decision to recommend eventual below-the-knee amputation rather than reconstruction for this patient?

A) Gap of the tibial nerve of 4 cm
B) Grade 2 liver laceration
C) Ipsilateral full-thickness anterior plantar skin avulsion with associated metatarsal fractures
D) New-onset absence of the pulses in the foot following external fixation
E) Parietal contusion requiring endotracheal intubation and neurosurgical consultation

A

The correct response is Option A.

Avulsion of the tibial nerve is one of the two absolute indications for amputation in the context of Gustilo Type IIIB and IIIC open tibial fractures. This is because outcomes of microsurgical reconstruction of the insensate foot in these cases are very poor.

The other absolute contraindication to reconstruction is a warm ischemia time of 6 hours or greater, which the patient described does not have. The patient has just lost pulses after manipulation and fixation. This is a new finding, with very recent ischemia time. This is a potentially fixable situation, and not necessarily a contraindication to reconstruction. Recent loss of pulses could contribute to a need for amputation in the future but would not require that decision now.

Polytrauma, such as a liver laceration or an intracranial injury, can evolve into relative contraindications to reconstruction, but they are not absolute contraindications by themselves. The liver laceration could resolve and allow safe microsurgical reconstruction. Even the delay of time to reconstruction that polytrauma can necessitate can be managed with bony fixation, wound care, and vacuum-assisted closure placement until construction of a flap can be performed.

The loss of plantar skin and metatarsal fractures by themselves are also not an absolute contraindication to reconstruction. This is a potentially reconstructible problem that should not push the plastic surgeon to recommend amputation. The additional foot trauma can potentially be reconstructed either with a skin graft or a flap.

The intracranial injury can evolve to a point where neurosurgery would eventually release the patient for a free flap. So, the parietal contusion is not necessarily an absolute contraindication to reconstruction, depending on the eventual outcome from the head injury. The patient may very well get better from that, and become a limb salvage candidate. It all depends on severity.

115
Q

A 17-year-old boy is brought to the emergency department after sustaining a traumatic injury to the left lower extremity in a motor vehicle collision. Physical examination shows a large area of crush injury, loss of soft tissue, and open fracture of the tibia with exposed bone. The lower leg is cool to touch and pale. No distal pulses are palpable. Angiography shows transsection of the popliteal artery. Which of the following is the most appropriate Gustilo classification of this patient’s fracture?

A ) Type I

B ) Type II

C ) Type IIIA

D ) Type IIIB

E ) Type IIIC

A

The correct response is Option E.

The most appropriate classification for this injury is Gustilo Type IIIC.

The Gustilo classification is the most widely accepted method for characterizing open fractures of the lower extremity. Injuries are divided into three grades. Type I open fractures involve soft-tissue lacerations smaller than 1 cm. Type II fractures include lacerations of 1 to 10 cm, with moderate soft-tissue damage. Type III fractures are greater than 10 cm and involve extensive soft-tissue damage. These injuries create difficulties in coverage of bone or fixation hardware. Gustilo Type III fractures are further subdivided into A, B, and C subtypes. Type IIIA fractures have sufficient soft tissue to provide for bony coverage. Type IIIB fractures involve periosteal stripping and extensive tissue damage, and local soft-tissue coverage is not possible. These typically result from high-energy mechanisms, such as high-velocity gunshot wounds or significant crush injuries. Gustilo Type IIIB fractures are the most common injuries for which plastic surgeons are consulted. Gustilo Type IIIC fractures include vascular injuries that require repair. The presence of a vascular injury significantly increases the probability of amputation.

116
Q

A 15-year-old boy is brought to the emergency department 1 hour after his left foot was severed when his leg was run over by a train. Photographs are shown. Which of the following is the most appropriate management?

A ) Construction of a foot filet free flap

B ) Replantation and ankle fusion

C ) Revision amputation to the level of the skin edge and primary closure

D ) Split-thickness skin grafting

E ) Temporary revascularization with shunts and delayed replantation

A

The correct response is Option C.

The patient described has a limb amputation that includes a significant avulsion component, exemplified in the long tendon stumps that appear ripped off from their muscle bellies, and ankle disarticulation. Avulsion amputations of the lower extremity are not suitable for replantation because of the extended neurovascular damage that is present on both ends well beyond the level of the injury. The incidence of thrombosis at the anastomosis is high. Use of vein grafts on a free flap could be considered, but, in this case, they may have to be connected as high as knee level to ensure patency. The amputated part likely has massive microscopic endothelial damage extending to terminal vessels and could only be used as a donor site for skin grafts. Therefore, there is no role for immediate or delayed replantation.

While preservation of maximum tibial length is desirable in preparing the amputation stump, prosthetic fitting for ankle disarticulations is fraught with trouble. Some tibial shortening is required to have a comfortable, reliable prosthesis. In the scenario described, tibial shortening of 2 and 4 in would have equivalent results because there is still ample length of proximal tibia

available to provide the same functionality and equivalent energy expenditure. Therefore, use of a free flap to protect an additional 2 in of tibia adds no advantage for this patient. When the level of amputation is through the proximal tibia, some benefit could be found in that a minimum of 6 in of proximal tibia is available for prosthetic fitting. Split-skin grafts are generally unstable in weight-bearing surfaces and take longer to heal and mature, which causes delay in rehabilitation. Skin grafts will not take or perform well when applied over an articular surface.

117
Q

A 43-year-old man has footdrop and numbness of the left foot following reconstruction of a soft-tissue defect resulting from a Gustilo Type IIIB fracture of the proximal tibia. Which of the following pedicled flaps was most likely used for reconstruction?

A ) Gracilis

B ) Lateral gastrocnemius

C ) Reversed sural artery

D ) Sartorius

E ) Soleus

A

The correct response is Option B.

The pedicled gastrocnemius flap has been shown to be a reliable source of vascularized soft tissue for injuries of the distal thigh, knee, and proximal leg. The lateral or medial head may be harvested. Lateral gastrocnemius harvest risks damaging the common peroneal nerve, with an incidence of 7.7%. Medial gastrocnemius harvest, for obvious anatomical reasons, does not carry this risk.

There are few data on the morbidity of donor sites, but those studies that have been done demonstrate no functional debility at a walking gait for less than 200 m. Patients do notice difficulty standing on their toes and have slowing with variable calcaneal gait when walking fast. Forty-two percent of patients could run, 22% had pain in the donor site at rest, and 20% had pain when walking more than 200 m. Seventy percent had pain and weakness in the operated leg when attempting to run. Range-of-motion deficit existed in the operated limb, with average loss of 27% flexion and 14% extension.

The gracilis and sartorius muscles are not routinely used for pedicled reconstruction of the proximal tibia, nor would their harvest cause the symptoms of the patient described.

The reversed sural artery flap is an adipofascial flap that could be used for this described defect; however, its dissection does not jeopardize the common peroneal nerve.

The soleus muscle has better venous muscle pump function than the gastrocnemius, and edema, not nerve damage, is more common after use of the gastrocnemius. The limited data available suggest that the functional deficit with resulting limitation to ankle flexion is also more severe with soleus harvest.

118
Q

A 62-year-old man is scheduled to undergo reconstruction of a 7-cm bone defect (shown) resulting from excision of a tumor of the distal radius. He has hypertension and has smoked one pack of cigarettes daily for 30 years. Which of the following is the most appropriate technique for reconstruction of the defect?

A ) Allograft bone graft

B ) Fibula free flap

C ) Locking fixation plate

D ) Pedicled ulna bone flap

A

The correct response is Option B.

Composite tissue reconstruction after tumor ablation is a significant challenge to the plastic surgeon. Generally, bony defects greater than 6 cm require vascularized bone for reconstruction. For large defects, the optimum choice of bone is the vascularized fibula. The fibula also has a reliable blood supply from the peroneal artery and can be harvested with a cutaneous skin paddle. The fibula free flap is the most appropriate technique. Reconstruction with free flaps can be successfully performed in a patient with a smoking history and, in fact, is often the optimal method of reconstruction in smokers. The use of allograft cadaver bone is not indicated, especially in a patient with a history of smoking. For defects of the size shown in the photograph, the locking fixation plate would not give adequate long-term support. A local bone flap is also not a viable option.

119
Q

A 70-year-old man is brought to the emergency department after he was hit by a car while walking across the street. Numerous fractures of the ribs, a nonoperative laceration of the spleen, a fracture of the right humerus, and a Gustilo Type IIIC fracture of the middle third of the left lower leg are noted. Examination shows an 8 - 7-cm soft-tissue defect over the anterior aspect with exposed bone. The patient is unable to plantar flex his left foot, and sensation is absent over the plantar aspect. Surgical exploration shows transection of the posterior tibial artery and tibial nerve. Which of the following is the most appropriate management of the injured leg?

A ) Above-knee amputation

B ) Below-knee amputation

C ) Irrigation, debridement, external fixation, free tissue transfer, and delayed repair of the nerve

D ) Irrigation, debridement, external fixation, immediate repair of the artery and nerve, and free tissue transfer

E ) Irrigation, debridement, internal fixation, immediate repair of the artery and nerve, and skin grafting

A

The correct response is Option B.

The treatment goal in the management of open tibial fractures and lower extremity salvage is to preserve a limb that will be more functional than an amputation. If the extremity cannot be salvaged, the goal is to maintain the maximum functional length.

Given the advanced age of the patient described, his associated injuries, and an insensate injured lower extremity, the most appropriate management of the injured leg would be a below-knee amputation.

Below-knee amputations provide better prosthetic function and require less energy for ambulation than above-knee amputations.

Although improved microvascular techniques have allowed for nerve repair and nerve grafting, the results of nerve repair and grafting in the lower extremity have been poor. In addition, the repair of the posterior tibial artery would not be necessary if the anterior tibial and peroneal arteries were to remain patent.

Free tissue transfer might be indicated for the soft-tissue defect. However, any anticoagulation used would increase the chance of serious bleeding complications because the patient is being treated nonoperatively for a splenic laceration.

Skin grafting over exposed bone would not be a good option.

120
Q

An 18-year-old man is brought to the trauma center after sustaining an injury to the right lower extremity. Examination shows an open fracture of the right tibia. Which of the following mechanisms of the injury is most likely to require the most extensive surgical debridement?

A ) A collision on the rink during which one skater runs over another

B ) A fall from a bicycle onto a curb after a collision with a pedestrian

C ) A fall from a 6-ft ladder onto a ceramic floor

D ) A fall from a shopping cart onto a parking lot

E ) A vehicle crash into a highway barrier while speeding

A

The correct response is Option E.

All of the mechanisms could have caused an open fracture requiring surgical intervention. Only the vehicle crash represents a high-energy injury. The approach to high-energy injuries must take into account a wider zone of injury beyond just the fracture site and skin laceration.

Open fractures in high-energy soft-tissue injuries have a high incidence of malunion and infection, especially with tibia fractures. These injuries require emergent debridement of both devitalized soft tissue and bone. Aggressive debridement will decrease the incidence of infection and increase the likelihood of successful reconstruction. Wounds frequently require multiple debridements followed by soft-tissue coverage including pedicled and free flaps. Vacuum-assisted closure applies negative pressure to an open wound causing increased granulation tissue, decreased edema, and decreased wound size. This technique has lowered the need for free flaps even in high-energy Gustilo Type III fractures.

The other options listed are not appropriate because the mechanisms of injury are low energy. Free-fall physics shows the housepainter hit the ceramic floor at a maximum of 13 mph. The patient €™s weight does not affect speed at impact. Distance = 1/2 × Gravity × Time (squared), Velocity at impact = Gravity × Time, Gravity = 32 ft/s2. The ice-skater sustained a sharp injury of low energy. The incident with the shopping cart is a low-energy injury, as is the bicyclist/pedestrian collision. The bicyclist hit a pedestrian and then fell. Average speed for a serious bicyclist is 13 to 18 mph. Colliding with a pedestrian would further decrease that speed.

121
Q

A 40-year-old man undergoes open reduction and internal fixation of an open fracture of the ankle. Debridement of nonviable tissue results in the exposure of the lateral joint and hardware. Coverage of the lateral malleolus with the flap shown is planned. Which of the following arteries must be intact for this flap to be viable?

A ) Anterior tibial

B ) Dorsalis pedis

C ) Lateral genicular

D ) Lateral plantar

E ) Peroneal

A

The correct response is Option E.

The reverse-flow sural flap has become one of the more dependable solutions in distal third leg wound and heel reconstruction. Survival of the flap depends on intact flow into the superficial

sural arterial network via perforators from the peroneal system. The largest perforator arises roughly 5 cm cephalad to the lateral malleolus and typically marks the lowest pivot point for the flap.

None of the remaining choices would provide adequate perfusion for this flap.

122
Q

A 48-year-old woman undergoes coverage of a defect with an anterolateral thigh flap. Which of the following branches of the lateral femoral circumflex artery is the most likely dominant vascular supply for the flap perforator?

A ) Ascending

B ) Descending

C ) Oblique

D ) Transverse

A

The correct response is Option B.

In recent studies by Wong, et al, and Rozen, et al, attempts have been made to clarify the vascular perforator anatomy of the anterolateral thigh flap. It has been noted that sizeable vascular perforators may arise from the ascending, transverse, or descending branch of the lateral circumflex femoral artery. The most common supply, however, comes from the descending branch, followed by the transverse and ascending branches.

The descending branch is dominant in about 85% of cases.

A variable oblique branch, which can occasionally be the primary supply, has also been noted. This branch, when it exists, is usually the primary supply of the flap in 15% of cases. It typically is an offshoot of the descending or transverse branch.

123
Q

A 70-year-old man has a mildly tender, clean, 6-cm wound to the lower leg that occurred spontaneously seven months ago. Physical examination of the ankle shows a brown-red discoloration and edema of the surrounding tissue. Which of the following is the most appropriate initial management of the wound?

A ) Hyperbaric oxygen therapy

B ) Silver sulfadiazine dressing

C ) Surgical debridement

D ) Unna boot compression dressing

E ) Vacuum-assisted closure therapy

A

The correct response is Option D.

The patient described has signs and symptoms of a venous stasis ulcer, a common cause of chronic lower leg wounds. Venous valve incompetence leads to chronic venous hypertension, capillary hydrostatic pressure elevation, and leakage of fluid and proteins to the extracellular space. Oxygen transport to the tissues is impaired, causing localized cellular necrosis and ulceration.

The hallmark of treatment is compression of the edematous limb to reduce the severe interstitial edema. By decreasing the tissue pressure, oxygen delivery is enhanced and wound healing mechanisms may be gradually restored. The open wound is best treated with an absorptive and occlusive dressing such as Unna wrap. Care must be taken to avoid overcompression and arterial compromise. Unna boots can be changed weekly or more frequently as needed. After reepithelialization has occurred, compression stocking or elastic bandage wraps are essential to prevent occurrence. Compression therapy is also a cost-effective treatment. Venous insufficiency affects up to 5% of the population; more than 500,000 patients in the United States suffer from ulceration. The economic costs from treatment and lost productivity are enormous.

Hyperbaric oxygen therapy and vacuum-assisted compression dressings may augment healing or help prepare the wound for grafting, but they are not the first line of treatment in uncomplicated ulcers. Recurrent ulceration rates are high, therefore unnecessary grafting should be avoided. Silver sulfadiazine and other topical antimicrobials may help reduce surface bacterial loads but do little to promote healing in the presence of venous hypertension. Debridement is reserved for necrotic wounds.

124
Q

The sural nerve is best localized at which of the following locations?

A ) 1 cm anterior to the fibular head

B ) 1 cm posterior to the lateral malleolus

C ) 1 cm anterior to the lateral malleolus

D ) 1 cm posterior to the medial malleolus

E ) 1 cm anterior to the medial malleolus

A

The correct response is Option B.

According to Sunderland and Ray, desirable features of cutaneous nerve grafts include (1) large fascicles with little interfascicular connective tissue, (2) separate parallel fascicles, (3) overall large diameter, (4) large-caliber axons, (5) easy accessibility, with little anatomic variation, (6) long unbranched segments, and (7) minimal sensory deficit. Because the sural nerve meets most of these criteria, it has long been the preferred donor for nerve reconstruction. This purely sensory nerve is comprised of contributions from the medial sural cutaneous nerve (branch of the tibial nerve) and the lateral sural cutaneous nerve (branch of the peroneal nerve). It has a mean diameter of 3.61 mm, which is ideal for group fascicular reconstruction; is easily harvested; and leaves minimal residual sensory deficit (the lateral foot and ankle). Up to 30 cm of nerve graft can be obtained. The distal portion of the nerve is consistently found between 1 and 1.5 cm posterior to the lateral malleolus.

125
Q

A 1-year-old boy is scheduled to undergo primary surgical reconstruction of congenital talipes equinovarus. Subsequent skin shortage and wound complications are best prevented by which of the following methods?

A ) Free fasciocutaneous flap transfer

B ) Healing via second intention

C ) Preoperative tissue expansion

D ) Split-thickness skin grafting

E ) Two-stage local flap delay and transfer

A

The correct response is Option C.

Although the treatment of congenital talipes equinovarus (CTEV) has trended toward conservative routines of manipulation and limited surgical release (eg, Ponseti technique), surgical treatment of CTEV may be necessary in cases of delayed treatment or failure of the conservative regime. Surgical correction may be attempted via a gradual technique (Ilizarov) or as an acute correction with release of the contracted posterior and medial elements.

Many acute surgical CTEV corrections can be accomplished without skin or wound difficulties; when such difficulties are anticipated, preoperative placement of tissue expanders has proven useful in allowing primary closure of the release sites with minimal morbidity.

Second intention healing and split-thickness skin grafting may not be appropriate depending on the purposeful injury and potential exposure of tendons during the contracture release.

Adjacent tissue transfer may be compromised by altered local anatomy, and creation of a second wound around the ankle by the donor site may threaten future surgical approaches for additional correction of the deformity.

Free tissue transfer may be warranted in late correction cases with severe tissue shortage but rarely would be necessary in the setting of primary correction.

126
Q

A 60-year-old man comes to the office because he has had tingling and numbness on the plantar aspects of the toes of both feet for the past seven months. He says that his symptoms are more noticeable after long periods of walking or standing and are relieved by elevation of the legs. Physical examination shows weakness of flexion of the toes. Tarsal tunnel syndrome is suspected. The most likely cause of these findings is palsy of which of the following nerves?

A ) Deep peroneal nerve to flexor digitorum superficialis muscle

B ) Interosseous nerve to quadratus plantae muscle

C ) Medial plantar nerve to flexor digitorum brevis muscle

D ) Superficial peroneal nerve to interosseus muscles

E ) Tibial nerve to flexor digitorum longus muscle

A

The correct response is Option C.

The tibial nerve bifurcates distally into the medial and lateral plantar nerves, and it is these two nerves that are impinged in the tarsal tunnel.

The flexor digitorum superficialis is in the forearm, not the leg. The flexor digitorum brevis flexes both the proximal and the middle phalanges of the toes and is innervated by the medial plantar nerve. Other intrinsic toe flexors are flexor hallucis brevis (medial plantar nerve), flexor digiti minimi brevis (lateral plantar nerve), and the interossei (lateral plantar nerve) and lumbricals (medial and lateral plantar nerves). In the most severe situation, a patient with tarsal tunnel syndrome may have €œclaw toes. €

Quadratus plantae is a flexor of the lateral four toes through the flexor digitorum longus but is innervated by the lateral plantar nerve. While flexor digitorum longus is a toe flexor and is also innervated by the tibial nerve, this nerve would not be involved at the tarsal tunnel.

127
Q

A 34-year-old man who works as a police officer is brought to the emergency department after sustaining a traumatic avulsion of the right heel. Examination shows a 3 * 3-cm area of exposed calcaneus on the weight-bearing aspect of the heel. Which of the following arteries and nerves supply the flap that will provide the most appropriate sensate coverage?

A

The correct response is Option B.

Sensate flaps include an intact sensory nerve supplying the overlying skin being transferred or, as in the case of a free tissue transfer, the cutaneous nerve is preserved and reanastomosed to a recipient cutaneous nerve. Sensate flaps are useful for areas where sensation is important for the appropriate function of a reconstructed region. Sensate flaps are beneficial in resurfacing wounds located on weight-bearing areas as in pressure sores or the heel, as in the scenario described.

The most reliable sensate flap for coverage of the plantar calcaneus comes from the medial plantar flap. This flap receives its sensation from the medial plantar nerve (L4-5) and receives its blood supply from the medal plantar artery. This flap comes from the instep of the foot between the head of the first metacarpal and the midpoint of the heel. The size of the flap can be up to 12 * 6 cm. Due to its proximity to the heel and the minimal donor morbidity with loss of sensation to the instep of the foot, this flap is the most appropriate for coverage of this patient €™s defect.

The dorsalis pedis flap receives its blood supply from the dorsalis pedis artery. This flap can be transferred as a sensate flap using the superficial peroneal nerve. However, this flap would not reach the area needed to be covered in this patient. The dorsalis pedis flap is used most frequently for coverage of the anterior ankle and dorsal distal foot.

The medial sural artery supplies the medial gastrocnemius flap. This flap is best used for soft-tissue coverage of the upper third of the leg and the distal knee. Preservation of the saphenous nerve with the medial gastrocnemius musculocutaneous flap will provide a neurosensory flap.

However, due to the distance from the flap, it would require a free tissue transfer and is typically not used for this type of coverage.

The proximal branches of the posterior tibial artery supply the medial soleus muscle flap. The posterior tibial nerve supplies only motor fibers to the soleus muscle and can therefore not be used as a sensate flap. This flap is best used for soft-tissue coverage of the middle third of the leg.

The radial forearm flap receives its blood supply from the radial artery and its septocutaneous perforating vessels. A neurosensory flap can be harvested based on the medial or lateral antebrachial cutaneous nerves to reconstruct the plantar surface of the foot. This would be a second choice to the medial plantar artery flap as it requires free tissue transfer.

128
Q

A 52-year-old man is brought to the emergency department after sustaining a gunshot wound to the right forearm. History includes well-controlled diabetes mellitus type 2, coronary artery disease, and renal insufficiency. Examination of the forearm shows viable muscle coverage and gross instability. A radiograph is shown. Following debridement and stabilization of the wound, reconstruction of the defect is planned via a vascularized free fibular transfer. In addition to clinical examination, which of the following is the most appropriate preoperative evaluation of this patient €™s lower extremity?

A ) Ankle brachial indices

B ) Color-flow Doppler imaging

C ) CT angiography

D ) Traditional angiography

E ) Transcutaneous partial pressure of oxygen

A

The correct response is Option B.

While anatomic variants in the vasculature of the leg are rare, failure to recognize these prior to sacrifice of the peroneal artery during harvest of a fibular free flap can result in disastrous ischemic complications. Variations occur more commonly in patients who have vascular insufficiency due to underlying atherosclerotic changes. In these patients, clinical examination alone is insufficient to adequately define blood flow patterns to the leg and foot.

Color-flow Doppler imaging has demonstrated excellent ability to define the presence of flow-limiting lesions in the leg. Monophasic signals in any of the three major runoff vessels strongly suggest that the limb will be at risk for ischemia following sacrifice of the peroneal artery. Comparison of the findings on color-flow Doppler with angiography demonstrates that angiography is an unnecessary addition to the initial study.

Angiography and CT angiography offer excellent road maps of the vasculature of the lower extremity, but each does involve the use of intravenous contrast. The dose of contrast used in CT angiography is significantly lower than that in traditional angiography, but neither would be justified in this patient with pre-existing renal insufficiency.

MR angiography provides a map of the vessels comparable to both angiography and CT angiography without the use of nephrotoxic contrast agents. The additional information gained by seeing the vessels preoperatively (by MRA, CT angiography, or traditional angiography) may be justified when very long segments of the bone need to be harvested or when the perfusion to the overlying skin paddle may be better defined.

Transcutaneous partial pressure of oxygen measures perfusion and does not offer any anatomic detail.

The postoperative image shown demonstrates the free fibula reconstruction of the radius.

129
Q

The response options for the next two items are the same. You will be required to select one answer for each item in the set.

For each of the following lower extremity flaps, select the dominant arterial inflow (A-E).

A ) Anterior tibial artery

B ) Peroneal artery

C ) Popliteal artery

D ) Posterior tibial artery

E ) Sural artery

Free fibula osseocutaneous flap

Proximally based soleus muscle flap

A

The correct response for Item 1. is Option B and for Item 2. is Option C.

The fibula osseocutaneous flap is a composite bone, fascia, and skin flap supplied by the peroneal artery.

The proximally based soleus muscle flap is supplied by branches of the popliteal artery.

A distally based soleus muscle flap is supplied by branches of the posterior tibial artery.

None of the mentioned flaps is supplied by the anterior tibial artery.

130
Q

A 17-year-old boy sustains a traumatic injury to the right lower extremity. Examination shows weakness of plantar flexion and loss of sensation over the plantar surface of the foot. Which of the following nerves is most likely injured?

A ) Deep peroneal

B ) Femoral

C ) Obturator

D ) Superficial peroneal

E ) Tibial

A

The correct response is Option E.

The most likely nerve to be injured is the tibial nerve.

The tibial nerve is a branch of the sciatic nerve. It travels through the popliteal fossa and gives off branches to the gastrocnemius, soleus, plantaris, and popliteus muscles. The tibial nerve travels in proximity to the posterior tibial artery. In the leg, it gives off branches to the flexor digitorum longus, tibialis posterior, and flexor hallucis longus. Distally in the foot, it branches to give rise to the medial and lateral plantar nerves, which provide sensation to the plantar surface of the foot. Injury to the tibial nerve results in deficits of plantar flexion, as well as anesthesia to the plantar surface of the foot. Tibial nerve disruption has been considered an indication for amputation as opposed to limb salvage in traumatic injuries of the lower extremity. However, one study suggests that limb salvage can be performed even in the face of absent plantar sensation at the time of presentation. This may indicate that absence of plantar sensation is not a reliable correlate for tibial nerve disruption.

The deep peroneal nerve arises from the common peroneal nerve at the fibular neck. It travels in the anterior compartment of the leg and gives branches to the tibialis anterior, extensor hallucis longus, and extensor digitorum longus and brevis, as well as peroneus tertius. The sensory distribution is in the area of the first web space. Injury to the deep peroneal nerve causes weakness in dorsiflexion of the foot.

The femoral nerve innervates muscles of the anterior thigh, including the quadriceps group, iliacus, and sartorius. Injury to the femoral nerve results in weakness of leg extension.

The superficial peroneal nerve arises from the common peroneal nerve at the fibular neck. It supplies the lateral compartment of the leg, giving motor branches to peroneus longus and brevis, as well as sensory contribution to the lateral aspect of the leg. Injury to the superficial peroneal nerve results in anesthesia of the lateral aspect of the leg and weakness in eversion and plantar flexion of the foot.

The obturator nerve provides innervation to the medial thigh muscles (adductor group), including adductor brevis, longus, and magnus, as well as the gracilis and obturator externus. The cutaneous branch provides sensation of the medial thigh. Injury to the obturator nerve results in weakness in thigh adduction and sensory deficits in the medial thigh.

131
Q

A 19-year-old man is brought to the emergency department after sustaining a heavily contaminated, open fracture of the tibia during an all-terrain vehicle collision. The injury is classified as a Gustilo Type IIIB fracture of the tibia in the distal third of the right leg. A comminuted 4-cm segment of the tibia is debrided. A soft-tissue defect measuring 7 * 4 cm overlies the fracture site. Which of the following is the most appropriate initial method to stabilize the fracture?

A ) Contralateral free fibula flap with intramedullary rod stabilization

B ) Iliac crest bone graft with plate stabilization

C ) Ipsilateral pedicled fibula flap with intramedullary rod stabilization

D ) Placement of an antibiotic-impregnated spacer

E ) Placement of an external fixation device

A

The correct response is Option E.

The most appropriate initial step for fracture stabilization is placement of an external fixation device. The patient described will require multiple debridements for the heavy contamination, followed by a free muscle flap to provide soft-tissue coverage of the exposed tibia fracture, and a future avascular bone graft for the 4-cm segmental bone loss.

Intramedullary rod stabilization for the initial management of open fractures of the tibia has gained increasing popularity in the orthopaedic literature, but the bone fixation is converted from external fixation to an intramedullary rod within the first 10 days of injury immediately prior to muscle flap coverage.

Although reported in the literature, the use of the ipsilateral fibula has been criticized, especially in trauma cases, because of (1) the loss of the mechanical integrity of the limb, which is especially useful to maintain limb length; (2) the loss of the peroneal artery in a traumatized limb; and (3) the loss of the origins of the deep muscles, which contributes an additional 30% loss of strength in an already traumatized and weakened extremity.

A 4-cm segmental bone loss does not require vascularized bone by either a pedicled or free bone flap.

Plate stabilization of the tibia fracture described is not indicated because of the heavy contamination and the soft-tissue loss. Also, avascular bone grafts are not recommended in the initial management of open lower extremity wounds because of the risk of graft loss secondary to infection.

An antibiotic-impregnated spacer could be used to help prevent infection, but it is not the initial step for fracture stabilization.

132
Q

A 38-year-old man comes to the office for follow-up examination two months after undergoing open reduction and internal fixation of a fracture of the right proximal fibula. A preoperative radiograph is shown. On current physical examination, he can evert the right foot, but he is unable to dorsiflex it. This patient has most likely sustained an injury to which of the following nerves?

A ) Common peroneal B ) Deep peroneal C ) Sciatic D ) Superficial peroneal E ) Sural

A

The correct response is Option B.

The patient described has sustained an injury to his deep peroneal nerve. An electromyelogram should be ordered to confirm the diagnosis. The common peroneal nerve divides into two branches. The deep branch of the peroneal nerve innervates the extensor hallucis longus and the anterior tibial muscles. The superficial branch of the peroneal nerve provides motor innervation to the peroneus longus and brevis muscles. If the common peroneal nerve had been injured, eversion would not have been likely. A sural nerve injury would have a sensory component, not a motor component. The sciatic nerve injury would have more global consequences, though this pattern is theoretically possible in a Sunderland Grade VI injury. Thus, the most likely injured nerve is the deep peroneal nerve.

133
Q

A 42-year-old man with type 1 diabetes mellitus comes to the office because he has had a sore on the heel of the left foot for the past 12 months. Physical examination of the foot shows a 4 * 4-cm ulcer. Pulses are difficult to palpate on the dorsalis pedis and posterior tibial arteries. No other abnormalities are noted. Noninvasive vascular studies are performed. Reconstruction with a free flap is planned. Which of the following is most likely to indicate the need for major arterial inflow reconstruction prior to reconstruction of the defect?

A ) Ankle-brachial index of less than 0.7

B ) Biphasic arterial Doppler waveform

C ) Delay in capillary refill of the toes of more than 3 seconds

D ) Doppler signal loss from the dorsalis pedis artery

E ) Great toe pressure of less than 30 mmHg

A

The correct response is Option E.

Thirty percent of diabetic patients have significant arterial calcifications in these arteries, which impedes blood pressure cuff compression, rendering the ankle-brachial pressure index inaccurate for ischemia. The ankle-arm index of 0.7 may thus be falsely elevated in this patient. Ordinarily, one would attempt to raise this index to at least 0.5 or 0.6 before performing soft-tissue reconstruction. Biphasic or triphasic arterial waveforms are usually consistent with sufficient vascularity for wound healing. Monophasic waveforms are associated more with extremity ischemia. However, even monophasic waveforms, as long as there is good amplitude and a narrow waveform complex, may be acceptable prior to attempting soft-tissue reconstruction.

Digital vessels are frequently spared from calcification and absolute toe pressures are a better indicator of peripheral ischemia than ankle pressures. A toe pressure of less than 30 mmHg indicates ischemia and the need to obtain an angiogram with a view to prior vascular inflow improvement (such as angioplasty, stenting, or surgical reconstruction) before proceeding to soft-tissue reconstruction. Capillary refill time is a subjective clinical evaluation that is not as accurate or objective as pressure testing. Incomplete plantar arch communication between the dorsalis pedis and posterior tibial arterial systems is not an accurate indication for ischemia.

134
Q

A 55-year-old man who is employed as a police officer is brought to the emergency department after sustaining injuries to the left lower extremity during a motor vehicle collision. Physical examination shows fracture of the tibia with 2 cm of bone loss, a large amount of tissue loss, and soft-tissue contamination extending over the calf, ankle, and forefoot. Sensation is absent in the plantar surface. Which of the following factors is most likely to influence the surgeon to proceed with immediate amputation of the leg?

A ) Age of patient

B ) Amount of bone loss

C ) Contamination of soft tissue

D ) Injury to posterior tibial nerve

A

The correct response is Option D.

In recent publications from the Lower Extremity Assessment Project (LEAP), a multicenter prospective study to determine outcomes following lower extremity trauma, the most significant factor in a surgeon €™s decision to immediately amputate was the absence of plantar sensation and the severity of soft-tissue injury. While age, soft-tissue contamination, and bone loss may be significant predictors of long-term limb salvage and function, they were not as significant in impacting the surgeon €™s initial decision to salvage or amputate the leg.

The Gustilo grade is used to categorize the amount of soft-tissue injury and periosteal stripping.

Gustilo II Moderate soft-tissue injury and stripping Gustilo IIIA A high-energy injury, but there is adequate soft tissue for closure despite laceration or undermining Gustilo IIIB The presence of extensive soft-tissue injury with periosteal stripping and gross contamination Gustilo IIIC Limb ischemia in addition to the findings in a IIIB injury

135
Q

A 50 year-old man is scheduled to undergo repair of a Gustilo grade IIIB open fracture of the tibia and fibula, resulting in an 8 €‘cm segmental defect of the tibia. Which of the following is an advantage of a vascularized fibula graft over a nonvascularized autograft for this patient?

(A) Greater bone resorption

(B) Greater osteocyte viability

(C) Incorporation by creeping substitution

(D) Remodeling

(E) Replacement of necrotic bone

A

The correct response is Option B.

Compared with a conventional nonvascularized autograft, a vascularized fibula graft has increased osteocyte viability. In one study, osteocyte counts in the cortex of vascularized grafts were 89% of those observed in normal control subjects. There were substantially fewer surviving osteocytes in conventional grafts.

Nonvascularized bone autograft incorporation occurs by creeping substitution, which involves gradual vascular ingrowth, resorption, and replacement of necrotic bone. The revascularization process that occurs causes significant mechanical weakening due to bone resorption at six to 12 months.

Vascularized bone grafts allow living bone tissue to be transplanted to an adjacent or remote location. They have been used in segmental bone defects larger than 6 cm due to tumor, trauma, osteomyelitis, or infected nonunion. Other indications include avascular necrosis of bone, composite tissue loss requiring complex reconstruction, and longitudinal growth with physeal transfer.

The transfer of vascularized bone grafts increases cell survival and as a result the process of bone graft incorporation by creeping substitution is obviated. Significant osteopenia is not seen and bone mass is maintained after transfer. The result is improved strength, healing, and stress response. Cell survival results in less remodeling during revascularization.

136
Q

A 35-year-old man comes to the office for evaluation of purulent drainage from the surgical wound one year after open reduction and internal fixation of tibia and fibula fractures sustained in a motorcycle collision. Radiographs show osteomyelitis and nonunion of tibia and fibula fractures. He is taken to the operating room for debridement of scarred and fibrotic soft tissue, removal of hardware, and application of external fixation. A cutaneous scapula free flap is used to close an 8-cm defect. Intravenous antibiotics are initiated and continued for six weeks. Three months later, the purulent drainage recurs. Which of the following is the most likely cause of treatment failure?

(A) Flap closure was under tension

(B) Inadequate antibiotic therapy

(C) Inadequate bone debridement

(D) Use of a cutaneous scapula free flap rather than a muscle free flap

(E) Use of a one-stage rather than a two-stage procedure

A

The correct response is Option C.

Lower extremity osteomyelitis following trauma is a challenging problem with high complication rates. The key to success is adequate debridement of both bone and soft tissue followed by coverage with vascularized tissue. The patient described had debridement of soft tissue but not bone. The bone is a persistent source of infection and the scarred fibrotic soft tissue is poorly vascularized, thus crippling the immune response. Without adequate bone debridement, early recurrence is likely. Muscle flaps fill the resultant defect, enhance blood flow, and improve immunologic defense against microorganisms.

The scapula flap can easily provide skin paddles of 10 × 15 cm; therefore, closure under tension would not be a problem.

A bone biopsy is helpful in identifying the bacteria and choosing the best antibiotic, but it does not substitute for bone debridement.

Though muscle flaps are generally more effective in the treatment of osteomyelitis than cutaneous flaps, many authors have demonstrated that well-vascularized skin flaps can be successful.

Many authors have demonstrated success with one €‘stage reconstructions for traumatic osteomyelitis. Again, success is dependent upon performing adequate bone and soft-tissue debridement followed by a well €‘vascularized flap.

137
Q

An 18 year old man comes to the office for follow-up examination becaus he has a four-month history of tripping with numbness and tingling of the right leg and foot. He underwent arthroscopic anterior cruciate ligament (ACL) grafting five months ago after he dislocated the right knee and ruptured the ACL during a football game. Physical examination shows weakness of dorsiflexion of the foot. Decreased sensibility to light touch over the lateral aspect of the right leg is noted. Which of the following is the most likely cause of his symptoms?

(A) Compartment syndrome

(B) Compression of the common peroneal nerve

(C) Laceration of the deep peroneal nerve

(D) Laceration of the superficial peroneal nerve

(E) Neurapraxia of the tibial nerve

A

The correct response is Option B.

Compression of the common peroneal nerve is the third most common nerve compression syndrome (carpal tunnel syndrome is first and cubital tunnel syndrome is second) and the most common nerve compression syndrome in the lower extremity. Knee dislocations are a common cause of common peroneal nerve compression. The patient described has a chronic injury that has not caused complete paralysis or loss of sensibility. The initial injuries would likely cause scar tissue around the knee but are not sharp in nature. Compression of the common peroneal nerve would result in weakness in the anterior compartment muscles caused by slowing of conduction in the deep peroneal nerve and paresthesias to the superolateral foot caused by slowing of conduction in the superficial peroneal nerve.

Compartment syndrome is caused by elevation of interstitial pressure (>30 mmHg) in the closed fascial compartment that results in microvascular compromise often secondary to a crushing injury or a high-force trauma. As duration and magnitude of interstitial pressure increase, myoneural function is impaired and necrosis of soft tissues eventually develops. Symptoms of an anterior compartment syndrome include extreme pain out of proportion to the injury and painful plantar flexion of the foot and toes.

The superficial peroneal nerve travels in and supplies the muscles of the lateral compartment of the leg. This nerve can be injured during an open reduction and internal fixation of a fracture of the ankle. A laceration of this nerve would not cause foot drop. The deep peroneal nerve travels in and supplies the muscles to the anterior compartment. Laceration of this nerve would cause foot drop but not decreased sensibility to the leg.

A neurapraxia of the tibial nerve at the level of the knee would affect the plantar flexors of the foot and toes. A tibial nerve injury would also cause planar foot paresthesias, which is the opposite of what the patient described is experiencing.

138
Q

A 24-year-old man is brought to the emergency department by ambulance because of injuries to the right leg he sustained in a motorcycle collision. Physical examination shows fracture of the tibia and fibula, thrombosis of the anterior and posterior tibial arteries, and transection of the posterior tibial and peroneal nerves just below the knee. Which of the following is the most appropriate management?
(A) External fixation
(B) Skeletal traction and vacuum-assisted closure of the wound
(C) Revascularization and coverage with a gastrocnemius flap
(D) Revascularization and coverage with a latissimus dorsi flap
(E) Below-the-knee amputation

A

The correct response is Option E.

This patient has sustained multiple traumas including injury to the skin, bone, arteries, and nerves. Using the injury severity scale, the prognosis of this injury is very poor. In an adult, there is little chance of regaining useful protective sensation or motor function after nerve repair in light of the open fractures and vascular injury. The best immediate management is amputation. Additionally, the return to weight-bearing and work is significantly shorter with amputation.

139
Q

A 14-year-old girl comes to the office for follow-up examination seven days after undergoing intramedullary nail fixation of a fracture of the right tibia. Along with bony stabilization, the degloved skin from the posterior middle and distal third of the leg was tacked back in place. On physical examination, necrosis of the replaced skin is noted. After debridement, the tibia is exposed in the middle to distal third of the leg. Which of the following is the most appropriate management?
(A) Split-thickness skin graft
(B) Sural artery flap
(C) Soleus flap
(D) Free latissimus flap
(E) Amputation

A

The correct response is Option D.

This Gustilo IIIB wound is best covered with a free muscle flap. The latissimus muscle flap is an effective and frequently used flap for large wounds. This flap also has the advantage of large vessels and a long pedicle length.

Dressing changes followed by a skin graft will not provide durable coverage over the tibia, is not appropriate in a healthy teenager who is not at surgical risk, and the time delay to coverage may predispose the patient to the development of osteomyelitis. A neuroadipofascial flap based on the vessels accompanying the sural nerve would be a poor choice, as this would be harvested from the zone of injury. The soleus flap is traditionally used for middle third defects of the leg and also is within the zone of injury. This is a salvageable extremity in a young teenager, which makes amputation inappropriate.

140
Q

A 24-year-old man is brought to the emergency department by ambulance because of injuries to the right leg he sustained in a motorcycle collision. Physical examination shows fracture of the tibia and fibula, thrombosis of the anterior and posterior tibial arteries, and transection of the posterior tibial and peroneal nerves just below the knee. Which of the following is the most appropriate management?
(A) External fixation
(B) Skeletal traction and vacuum-assisted closure of the wound
(C) Revascularization and coverage with a gastrocnemius flap
(D) Revascularization and coverage with a latissimus dorsi flap
(E) Below-the-knee amputation

A

The correct response is Option E.

This patient has sustained multiple traumas including injury to the skin, bone, arteries, and nerves. Using the injury severity scale, the prognosis of this injury is very poor. In an adult, there is little chance of regaining useful protective sensation or motor function after nerve repair in light of the open fractures and vascular injury. The best immediate management is amputation. Additionally, the return to weight-bearing and work is significantly shorter with amputation.

141
Q

A 14-year-old girl comes to the office for follow-up examination seven days after undergoing intramedullary nail fixation of a fracture of the right tibia. Along with bony stabilization, the degloved skin from the posterior middle and distal third of the leg was tacked back in place. On physical examination, necrosis of the replaced skin is noted. After debridement, the tibia is exposed in the middle to distal third of the leg. Which of the following is the most appropriate management?
(A) Split-thickness skin graft
(B) Sural artery flap
(C) Soleus flap
(D) Free latissimus flap
(E) Amputation

A

The correct response is Option D.

This Gustilo IIIB wound is best covered with a free muscle flap. The latissimus muscle flap is an effective and frequently used flap for large wounds. This flap also has the advantage of large vessels and a long pedicle length.

Dressing changes followed by a skin graft will not provide durable coverage over the tibia, is not appropriate in a healthy teenager who is not at surgical risk, and the time delay to coverage may predispose the patient to the development of osteomyelitis. A neuroadipofascial flap based on the vessels accompanying the sural nerve would be a poor choice, as this would be harvested from the zone of injury. The soleus flap is traditionally used for middle third defects of the leg and also is within the zone of injury. This is a salvageable extremity in a young teenager, which makes amputation inappropriate.

142
Q

Which of the following anatomic structures is an important landmark in raising a reverse sural artery flap?

(A) Achilles tendon
(B) Deep peroneal nerve
(C) Lesser saphenous vein
(D) Plantaris tendon
(E) Posterior tibial artery

A

The correct response is Option C.

When raising a reverse sural artery flap, the important landmarks are the lesser saphenous vein and sural nerve, which should bisect the cutaneous paddle. The blood supply to this flap depends on the medial superficial sural artery and the lesser saphenous vein with its two small accompanying arteries. The pivot point of the pedicle is typically 5 cm above the lateral malleolus, where the perforators of the flap enter a more superficial plane.

The Achilles tendons are not landmarks for raising this flap but, when exposed, are good indications for this type of flap. The deep peroneal nerve is located in the lateral compartment. The posterior tibial artery is found in the deep compartment and is not associated with this flap. The plantaris tendon is deep to the dissection of the flap.

143
Q

A 53-year-old man with a comminuted fracture of the midtibia has a 4 H 3-cm defect of the midanterior surface of the leg at the level of the fracture. He currently smokes two packs of cigarettes daily. Physical examination shows no palpable dorsalis pedis pulse. Which of the following surgical interventions is the most appropriate method of reconstruction in this patient?

(A) Anterior tibialis muscle flap
(B) Below-knee amputation
(C) Gastrocnemius muscle flap
(D) Gracilis free tissue transfer
(E) Soleus muscle flap

A

The correct response is Option E.

The soleus muscle flap is most appropriate for reconstruction in this patient. The soleus is a bipenniform muscle; its medial head originates from the posterior tibia, and the lateral head originates from the proximal fibula. It is located deep to the gastrocnemius in the superficial posterior compartment. Blood to the medial head is predominantly supplied by the popliteal and posterior tibial arteries and the lateral head is predominantly supplied by the peroneal artery. Depending on the size of the defect, a hemisoleus muscle flap can be used to preserve flexor function.

Below-knee amputation is an option if salvage of the leg is not possible or if the extremity is insensate, particularly in older patients.

An anterior tibialis muscle flap can be used for small defects. In this patient with an absent dorsalis pedis pulse and possible injury to the anterior tibial artery, this is not an optimum choice.

For lower-extremity reconstruction, the gastrocnemius muscle flap is used for knee wounds and proximal tibial defects, the soleus for middle third defects, and free tissue transfer for distal third defects. The gastrocnemius muscle flap might not reach the defect in the middle third and therefore is not the best option. Free tissue transfer is often used for reconstruction of high-velocity injuries to avoid the use of muscle in the zone of injury. Free tissue transfer, however, is not the best option for this 53-year-old man because his history of cigarette smoking and absent pedal pulse suggest the possibility of peripheral vascular disease.

144
Q

A 53-year-old man with diabetes mellitus has a nonhealing wound over the right calcaneus. A bone scan shows increased uptake at the site of the wound. Which of the following is the most definitive diagnostic test?

(A) Bone biopsy and culture
(B) CT scan
(C) Indium scan
(D) MRI
(E) Wound biopsy and culture

A

The correct response is Option A.

This 53-year-old man with a nonhealing wound has most likely developed osteomyelitis of the foot, which may be caused by direct, penetrating trauma to the bone or contiguous spread from adjacent soft tissue. The calcaneus is the most common site of involvement in the foot, followed by the metatarsals and the cuboid bone. Although osteomyelitis may be suspected clinically, definitive diagnosis is accomplished with bone biopsy and culture. This can be obtained by needle aspiration of the interosseous or subperiosteal space.

Cultures of a draining wound are less accurate diagnostically than bone biopsy and culture. Findings on bone scan are nonspecific and are often positive in patients with local wound infection or cellulitis. CT scans are not used in the diagnosis of osteomyelitis. Although MRI is more accurate than bone scan, it cannot be used to establish a definitive diagnosis.

145
Q

A 27-year-old man develops osteomyelitis after sustaining an open fracture of the distal tibia and fibula in a motorcycle accident. Following bony debridement, there is a 9-cm segmental loss of the distal tibia. An external fixator is applied, and a muscle flap is to be used to cover the skin defect. Which of the following is the most appropriate management of the bony defect?

(A) Free fibula flap
(B) Iliac crest bone grafting
(C) Papineau bone grafting
(D) Tibiofibular synostosis
(E) Ilizarov bone lengthening

A

The correct response is Option A.

The most appropriate management of this patient’s bone defect is a free fibula surrounding flap. Management typically depends on the volume of the bony defect along with soft-tissue stability and vascularity. If the tissue is unstable and poorly vascularized, a regional flap or free flap is required. For most bone defects smaller than 6 cm, traditional tricortical iliac crest bone grafts can be placed beneath the muscle flap. In patients with larger defects, a vascularized bone flap, such as the free fibula flap, is required. Vascularized bone also minimizes the risk for nonunion. However, weight-bearing ambulation should not occur for many months after flap coverage to allow for healing of bone without infection.

Other sources of free vascularized bone include the iliac crest and scapula. However, the iliac crest has both inadequate length and excessive curvature, and the scapula has inadequate length and lacks the tubular depth and width of the fibula.

Papineau grafts are cancellous grafts that are used to fill the bone defect but are packed flush with the skin. These grafts are seldom used clinically and are appropriate only for defects smaller than 6 cm. Tibiofibular synostosis is not a reasonable option in a patient with osteomyelitis of the distal tibia who has compromised stability and vascularity of the soft tissue. The Ilizarov technique for bone transport is less optimal in a patient who requires a long bone segment and has soft tissue of poor quality.

146
Q

The saphenous artery originates from which of the following arteries?

(A) Descending lateral femoral circumflex
(B) Genicular
(C) Profunda femoris
(D) Superficial femoral

A

The correct response is Option B.

The saphenous artery supplies vascularity to the saphenous flap, a fasciocutaneous flap that provides only limited coverage of small defects in the region of the knee. The saphenous artery is a branch of the descending genicular artery. The genicular artery arises from the superficial femoral artery.

The descending branch of the lateral femoral circumflex artery arises from the profunda femoris and continues distally in the leg as the superior lateral genicular artery. The profunda femoris artery gives off its major branches more proximally in the leg. The superficial femoral artery becomes the popliteal artery at the popliteal fossa.

147
Q

Which of the following vascular structures supply the secondary vascular pedicle of the gracilis muscle flap?

(A) Lateral femoral circumflex artery and vein
(B) Medial femoral circumflex artery and vein
(C) Obturator artery and vein
(D) Profunda femoris artery and vein
(E) Superficial femoral artery and vein

A

The correct response is Option E.

The gracilis flap is a type II muscle flap (one dominant pedicle and one secondary pedicle) that is used for pedicled and free tissue reconstructive procedures. This flap is comprised of one primary vascular pedicle, which is supplied by the profunda femoris artery via the ascending branch of the medial femoral circumflex artery and vein, and one secondary vascular pedicle, which is supplied by the superficial femoral artery and vein.
The lateral femoral circumflex artery and vein supply the rectus femoris and vastus lateralis muscle flaps.

The obturator artery is a branch of the internal iliac artery and divides further to give off vascular branches, primarily in the thigh.

148
Q

A 28-year-old man has a markedly diminished dorsalis pedis pulse on the right after being stabbed with an ice pick in the right lower leg. In this patient, the most likely finding on physical examination is numbness at which of the following sites?

(A) Dorsal foot
(B) First web space
(C) Lateral foot
(D) Medial foot
(E) Plantar foot

A

The correct response is Option B.

This 28-year-old man has a markedly diminished dorsalis pedis pulse after sustaining a laceration of the anterior tibial artery when he was stabbed in the leg. Therefore, he is most likely to have numbness of the first web space of the foot on physical examination. The first web space is innervated by the deep peroneal nerve; this nerve courses within the anterior compartment of the leg with the anterior tibial artery, which becomes the dorsalis pedis artery in the foot.

The dorsal aspect of the foot (with the exception of the first web space) is innervated by the superficial peroneal nerve. The lateral foot is innervated by the sural nerve, and the medial foot is innervated by the saphenous nerve. None of these nerves travels with an artery.

The plantar surface of the foot is innervated by the tibial nerve, which courses within the deep posterior compartment of the leg with the posterior tibial artery.

149
Q

Which of the following is the most common primary arterial blood supply to the great toe flap used in toe-to-thumb transfer?

(A) First dorsal metatarsal artery from the communicating branch between the dorsalis pedis and plantar arteries
(B) First dorsal metatarsal artery from the dorsalis pedis artery
(C) First dorsal metatarsal artery from the plantar arch
(D) Second dorsal metatarsal artery from the dorsalis pedis artery
(E) Second dorsal metatarsal artery from the plantar arch

A

The correct response is Option B.

The first dorsal metatarsal artery (FDMA), arising from the dorsalis pedis artery, is the most common primary arterial blood supply to the great toe flap, which is used in toe-to-thumb transfer. Although the vascular anatomy that supplies the great toe flap is somewhat variable, the FDMA typically arises from the dorsalis pedis artery and travels in a relatively dorsal plane. It may travel dorsal or plantar to the interosseous muscle or through the muscle itself.

The FDMA is less likely to arise from the communicating branch between the dorsalis pedis and plantar arteries or from the plantar arch.

The second dorsal metatarsal artery does not contribute to the great toe flap.

150
Q

A 77-year-old man has a 4 ( 3-cm defect of skin and soft tissue over the distal third of the tibia after he had a stroke and fell. He has a history of myocardial infarction and chronic obstructive pulmonary disease. Physical examination of the lower leg shows exposed bone and desiccated periosteum; there is no fracture.

Which of the following is most appropriate for reconstruction of the defect?

(A) Full-thickness skin graft
(B) Cross-leg flap
(C) Fasciocutaneous flap
(D) Gastrocnemius flap
(E) Free tissue transfer

A

The correct response is Option C.

The most appropriate management is coverage of the defect with a fasciocutaneous flap. This flap is ideal for reconstruction of lower extremity wounds in patients with severe illness or multiple trauma, or in patients with small wounds that cannot be covered with a skin graft alone. The fasciocutaneous flap can be based either proximally or distally on various septocutaneous perforators, including those of the medial leg (which lie approximately 3 cm posterior to the tibia), the posterolateral septum, and the anterolateral leg.

Skin grafting should not be performed over bone that is exposed and lacks periosteum. Some surgeons have recently described a technique for grafting over exposed bone, in which holes are drilled into the bone to allow for granulation and grafting is then performed. However, this process is not the best option in an elderly patient with multiple medical problems.

The gastrocnemius flap is appropriate for defects of the upper and sometimes middle third of the leg, but lacks the adequate reach for defects of the distal leg.

Cross-leg flaps are rarely used now because of the availability of free tissue transfer. This flap is more appropriate in children than elderly patients, in whom stiffness is a factor.

Free tissue transfer is not an option in a patient who has had serious medical conditions, including stroke, myocardial infarction, and pulmonary disease.

151
Q

A 50-year-old man has noninfected wound dehiscence with exposure of the hardware two weeks after undergoing open reduction and internal fixation of an ankle fracture. A photograph and radiograph are shown above. Following debridement of the wound, which of the following is the most appropriate management?

(A) Skin grafting
(B) Coverage with a free flap
(C) Application of an external fixator and skin grafting
(D) Removal of the hardware and skin grafting
(E) Removal of the hardware and coverage with a free flap

A

The correct response is Option B.

This patient has early wound dehiscence after undergoing open reduction and internal fixation of a lateral malleolus fracture. Appropriate management involves early coverage of the exposed bone and hardware to promote bony union while preventing bacterial colonization of the hardware, which may lead to the development of osteomyelitis. Because this patient underwent open reduction and internal fixation only two weeks ago, fixation is still required to maintain rigid fracture stabilization. Instead, coverage with a free flap is recommended to provide stability and vascularity and to enhance function once bony union is obtained.

Skin grafting will not provide stable, well vascularized coverage in this patient. External fixation is problematic because of the location of the fracture. Removal of the hardware two weeks after fracture is indicated only for established hardware failure.

152
Q

An ambulatory 75-year-old man has a 6-cm grade IV pressure ulcer on the sacrum. The most appropriate management is reconstruction with which of the following?

(A) Bilateral gluteal myocutaneous advancement flaps
(B) Unilateral superior/inferior gluteal myocutaneous rotation flap
(C) Unilateral superior gluteal artery perforator flap
(D) Thin split-thickness skin graft

A

The correct response is Option C.

The unilateral superior gluteal artery perforator flap is most appropriate for reconstruction of the area of the sacrum affected by the pressure ulcer. Perforator flaps can be based on either the superior or inferior gluteal artery. They provide adequate myocutaneous coverage and at the same time preserve the gluteal muscles on the involved side. As a result, morbidity at the donor site will be minimized, which is especially important in this patient because he is ambulatory.

Although a bilateral superior/inferior gluteal myocutaneous advancement flap supplies sufficient soft-tissue coverage for pressure ulcers, harvest of the inferior and superior gluteal muscles will result in significant morbidity at the donor site. Harvest of a gluteal myocutaneous rotation flap sacrifices muscle function and also is not the procedure of choice in an ambulatory patient. Thin split-thickness skin grafts do not provide stable coverage of pressure ulcers.

153
Q

A 78-year-old man has an infection of the groin wound after undergoing femoral popliteal artery bypass grafting. A sartorius muscle flap is to be used for coverage of the exposed prosthetic graft. Which of the following arteries provide the blood supply for this flap?

(A) Lateral femoral circumflex artery and superficial femoral artery
(B) Lateral femoral circumflex artery and deep femoral artery
(C) Medial circumflex femoral artery and superficial circumflex iliac artery
(D) Medial circumflex femoral artery and deep femoral artery
(E) Superficial circumflex iliac artery and superficial femoral artery

A

The correct response is Option E.

The superficial circumflex iliac and superficial femoral arteries provide vascularity to the sartorius muscle flap. The superficial circumflex iliac artery is a branch of the external iliac artery; it courses laterally to provide the primary blood supply to the groin flap and the proximal part of the sartorius muscle. The external iliac artery then continues as the common femoral artery and branches into the profunda femoris and superficial femoral arteries at a point distal to the inguinal ligament. The superficial femoral artery travels distally in the thigh and gives off segmental branches to the sartorius muscle.

Because the groin is one of the most common areas of graft exposure, the sartorius muscle can be used to cover small defects. This muscle can be detached from its proximal origin on the iliac crest and turned over to cover groin wounds. Other options for coverage in this region include the gracilis, rectus femoris, vastus lateralis, tensor fascia lata, and rectus abdominis muscles.

The lateral femoral circumflex artery, which is a branch of the profunda femoris artery, supplies blood to the rectus femoris, tensor fascia latae, and vastus lateralis muscles. The medial circumflex femoral artery is a branch of the profunda femoris artery. This vessel provides the blood supply to the gracilis muscle.

154
Q

A 27-year-old man has a 12-cm defect of the tibia after sustaining open comminuted fractures of the tibia and fibula in a motorcycle accident. On physical examination, sensation is intact. The wounds are irrigated and debrided, and an external fixator is applied. Which of the following is most appropriate for reconstruction of the defect?

(A) Bone allograft
(B) Contralateral free vascularized fibula graft
(C) Free vascularized iliac crest graft
(D) Ipsilateral pedicle fibula graft
(E) Nonvascularized iliac crest graft

A

The correct response is Option B.

In a patient who has a 12-cm bone defect, the most appropriate management is reconstruction using a well-vascularized, uninjured segment of bone graft, such as a contralateral free vascularized fibula graft. The contralateral fibula will provide more than 20 cm of bony length.

Neither allograft nor nonvascularized autograft would provide the required bone length for revascularization. For this reason, nonvascularized bone graft is also not appropriate.

Vascularized iliac crest graft can be used for reconstruction in the upper and lower extremities but would not provide a large enough segment of bone graft.

Because of this patient’s injuries, the ipsilateral fibula should not be used. In addition, harvest of an ipsilateral bone graft would remove the ipsilateral supplemental support.

155
Q

Six months after undergoing plate fixation and primary wound closure for management of open fractures of the distal tibia and fibula sustained in a motorcycle accident, a 43-year-old man has purulent drainage from the wound site. Radiographs show bony nonunion at the fracture sites.

Which of the following is the most appropriate initial management?

(A) Continuous irrigation
(B) Debridement of bone
(C) Coverage with a muscle flap
(D) Bone grafting
(E) Insertion of an intramedullary rod

A

The correct response is Option B.

Osteomyelitis is a frequent complication of open fractures associated with soft-tissue injury, fibrosis, and localized ischemia. Measures to prevent the development of osteomyelitis, including removal of dead and devitalized bone, closure of dead space, and coverage with well-vascularized soft tissue, are recommended. In patients with established osteomyelitis, the most appropriate initial management is debridement of devascularized bone and necrotic or scarred tissues and removal of any nonautologous material, such as fixation devices. Because local soft tissue is frequently inadequate, free tissue transfer is often performed for soft-tissue coverage in the lower third of the leg. In patients who have unhealed fractures, an external fixation device is used to stabilize the fracture pattern. An Ilizarov frame may be applied if lengthening is required.

Continuous irrigation alone will not treat the osteomyelitis. Coverage with a well-vascularized muscle flap should be performed following bony debridement, and bone grafting should be delayed until the bone and soft tissues are stabilized and the osteomyelitis has resolved. Insertion of an intramedullary rod may further compromise bony perfusion.

156
Q

A 29-year-old woman is brought to the emergency department one hour after sustaining a traumatic avulsion amputation at the level of the proximal third of the lower leg. On examination, the tibia is exposed, and there is not adequate soft tissue for coverage of the bone. The amputated part has been appropriately preserved; examination shows extensive injury in the middle third of the leg but no evidence of injury in the distal third or the foot.

In order to provide the best functional outcome in this patient, which of the following is the most appropriate management?

(A) Above-knee amputation
(B) Immediate coverage of the exposed tibia with a reverse rectus femoris flap
(C) Immediate coverage of the exposed tibia with a filet-of-foot free flap
(D) Immediate coverage of the exposed tibia with a latissimus dorsi muscle flap and split-thickness skin graft
(E) Microvascular replantation of the amputated leg with 12 cm of bone shortening to allow for complete soft-tissue coverage

A

The correct response is Option C.

The most appropriate management is immediate coverage of the exposed tibia using a filet-of-foot free flap. Free tissue from the amputated foot can be transferred to provide appropriate soft-tissue coverage without additional donor site morbidity. The dorsalis pedis artery and/or posterior tibial artery supply vascularity to the flap, and the tibial nerve provides sensory innervation.

Above-knee amputation should be considered only as the last possible option. The energy required for ambulation in a patient with an above-knee amputation is much greater than in a patient with below-knee amputation.

The reverse rectus femoris flap is not based on a distal pedicle; thus, it should not be used in a patient in whom knee extension must be preserved.

A latissimus dorsi flap and split-thickness skin graft could be used for coverage if the amputated part were injured and unavailable as a donor site; however, in this patient there is no evidence of trauma distal to the lower third of the leg.

Microvascular replantation cannot be performed in a patient who has sustained an avulsion amputation. In addition, bone shortening of greater than 10 cm would ultimately result in an abnormal gait.

157
Q

A 34-year-old motorcycle driver sustains a Gustilo type IIIB open fracture of the right tibia. At the time of debridement, stabilization, and coverage of the wound, there is a superficial infection of the tibia. Low-pressure pulse lavage is to be performed.

Which of the following solutions will most likely preserve osteoblast-osteoclast function while clearing bone-adherent bacteria?

(A) 1% Bacitracin
(B) 1% Chlorhexidine
(C) 1% Ethanol
(D) 1% Povidone-iodine
(E) 1% Surgical soap

A

The correct response is Option E.

According to the results of an animal in vitro study, low-pressure pulse lavage with 1% surgical soap resulted in a more significant clearing of bone-adherent bacteria than other solutions. Low-pressure lavage was defined as 14 psi of pressure with 550 pulsations per minute; the solutions were evaluated in 1% and 10% concentrations. Osteoclast and osteoblast function was also shown to be preserved more significantly with use of the 1% soap or detergent solution. In contrast, the other solutions studied were shown to have a deleterious effect on osteoclast and osteoblast function. Low-pressure lavage with chlorhexidine or povidone-iodine was associated with the greatest decrease in osteoblast and osteoclast function.

The antibacterial mechanism of soap is thought to act through its micelle function. The hydrophilic ends of the soap micelle surround the bacteria, interfering with its potential for adherence to bone.

High-pressure lavage should not be used because it results in a significant depth or zone of bony injury.

158
Q

A 24-year-old woman has a 3-cm soft-tissue defect at the medial malleolus 10 months after sustaining an open fracture of the distal third of the tibia in a motorcycle accident. There were no vascular or nerve injuries at the time of the accident. Examination shows exposed bone with dense scarring of the skin and adherence to underlying tissue. Radiographs are consistent with osteomyelitis and fracture nonunion.

Which of the following is the most appropriate management?

(A) Total contact casting
(B) Split-thickness skin grafting
(C) Coverage of the wound with a soleus pedicle flap
(D) Coverage of the wound with a free latissimus dorsi flap
(E) Below-knee amputation

A

The correct response is Option D.

The most appropriate management of this patient with a soft-tissue defect and osteomyelitis is stabilization of the segment, debridement of the affected area, and coverage with a free latissimus dorsi pedicle flap. Application of an external fixator, radical sequestrectomy, placement of antibiotic beads, and autotransplantation of a free latissimus dorsi flap are performed initially; bone grafting of the tibial defect is performed secondarily.

There is controversy regarding the precise advantages offered by free flaps when applied to infected bone. However, the basic principles for use of these flaps include skeletal stabilization, radical debridement of infected or devitalized tissues, and coverage with vascularized tissues. The free flap provides an abundance of vascularized tissue necessary to close large wounds left by aggressive debridement, and the muscle free flap provides pliability to fill the dead spaces.

Total contact casting can be performed in patients with clean, slowly healing wounds of the distal lower extremity, but would only aggravate the fracture nonunion and osteomyelitis in this patient. Adequate debridement of only the soft-tissue scar and skin grafting over the wound will not result in a stable, healed site without infection. A soleus pedicle flap would not reliably cover this complex distal leg wound. In addition, the posterior compartment leg muscles and their vascular supply are frequently injured by high-energy injuries. Therefore, the soleus muscle is not likely to be reliable for transfer. A below-knee amputation would not be appropriate in a healthy patient with a sensate distal extremity in which the circulation is fully intact.

159
Q

A 42-year-old man sustains open fractures of the tibia and fibula (Gustilo type IIIB) when he is thrown from his motorcycle. The wounds are irrigated, debrided, and covered with a free flap. Which of the following will NOT increase this patient’s risk for the development of osteomyelitis?

(A) Dead space at the surgical site
(B) Inadequate soft-tissue coverage
(C) Internal fixation
(D) Presence of nonviable muscle
(E) Retention of devitalized bone

A

The correct response is Option C.

Patients who sustain open fractures of the long bones are susceptible to bacterial invasion of the wound site and the subsequent development of infection, especially osteomyelitis. Factors associated with persistent infection include the presence of dead space at the surgical site, inadequate soft-tissue coverage, the presence of nonviable soft tissue, including muscle, and retention of devitalized bone. Aggressive irrigation and debridement should be performed in any patient with an open fracture. All nonviable tissue must be removed, and the fracture site should be covered with adequate soft tissue from a local site or via a free tissue transfer.

Internal fixation has not been associated with development of osteomyelitis in patients who have open fractures of the tibia and/or fibula.

160
Q

The percentage of persons who have absence of the plantaris muscle in one lower extremity is

(A) less than 5%
(B) 10% to 15%
(C) 25% to 30%
(D) 45% to 50%
(E) 75% to 80%

A

The correct response is Option B.

According to the results of several studies, the plantaris muscle has been shown to be absent in at least one lower extremity in approximately 10% to 15% of persons. This tendon can be used for grafting procedures. It provides a longer donor graft than the palmaris longus in the forearm; however, it is hidden on physical examination, making it more difficult to locate prior to surgery. Because the muscle may be absent in one or both lower extremities, ultrasonography can be helpful in determining its presence and location. Approximately 33% of patients have a plantaris muscle in only one lower extremity.

Other studies have shown that absence of the plantaris muscle occurs with slightly greater frequency in women, and the plantaris muscle is more likely to be absent in the left lower extremity. Differences based on race and/or ethnicity have not been studied. In addition, absence of the plantaris muscle is unrelated to absence of the palmaris longus muscle; in other words, a person who is missing one or both plantaris muscles may in fact have one or both palmaris longus muscles.

161
Q

The above photograph is of a 45-year-old man with insulin-dependent diabetes mellitus who develops a gangrenous toe. Culture of the wound shows mixed aerobic and anaerobic organisms, including Bacteroides, Enterococcus, and Staphylococcus. Noninvasive vascular studies show an ankle-brachial index of 0.76. The patient wishes to undergo a single-stage surgical procedure. ***NO PHOTO PROVIDED***

Which of the following is the most appropriate type of amputation for this patient?

(A) Amputation at the level of the metatarsophalangeal joint
(B) Transmetatarsal amputation
(C) Lisfranc amputation
(D) Syme’s amputation
(E) Below-knee amputation

A

The correct response is Option B.

Amputation is indicated in this patient who has obvious necrosis of the toe. Indeed, ischemic changes and wound problems are often seen in patients with diabetes mellitus. Because these patients are typically predisposed to further, more proximal amputations in the future, a conservative approach to amputation should be used in this instance. Several factors, including ankle-brachial index, help to predict the success rate in patients who undergo partial amputations of the foot; an ankle-brachial index of less than 0.7 indicates a markedly increased risk for wound healing problems following surgery. However, this patient has an ankle-brachial index of 0.76, which is an acceptable risk for complications following amputation. Therefore, an evaluation for the likelihood of revascularization should be undertaken prior to any amputation procedure. The surgeon should attempt to save as much of the foot as possible to allow for primary closure of the defect. As a result, the amputation should be performed at the metatarsal level in this patient. The surgeon should also be aware of the vascular supply to the adjacent toes during the amputation procedure in order to prevent any associated complications.

162
Q

A 37-year-old man who has paraplegia to the level of T9 is referred for evaluation because he has had a chronic penetrating wound on the weight-bearing surface of the exposed left calcaneus for the past 10 years. He has also had several associated episodes of cellulitis that have resolved with conservative therapy consisting of wound care, elevation of the limb, and administration of antibiotics.

In order to exclude a diagnosis of osteomyelitis of the calcaneus, which of the following is the most appropriate diagnostic study?

(A) Measurement of erythrocyte sedimentation rate
(B) Plain radiographs of the heel
(C) High-resolution CT scan
(D) MRI
(E) Triple-phase bone scan

A

The correct response is Option D.

An MRI should be performed in this patient to exclude a diagnosis of osteomyelitis. Although all of the imaging studies listed above will show abnormalities of bone, only a gadolinium-enhanced MRI will delineate the signal characteristics of bone marrow and specific enhancement pattern characteristic of osteomyelitis.

Measurement of erythrocyte sedimentation rate is highly sensitive but nonspecific for osteomyelitis; this test is used instead as a follow-up for determining the efficacy of treatment in patients previously diagnosed with osteomyelitis.
d to an advanced stage. High-resolution CT scans are inferior to MRI for diagnosis of osteomyelitis.

163
Q

A 65-year-old attorney has severe ischemia of the right leg. On examination, the leg is gangrenous and ulcerated; he has pain with motion and at rest. Noninvasive vascular studies show an ankle-brachial index of 0.16. He refuses to undergo amputation.

Which of the following is the most appropriate technique for limb salvage?

(A) Distal arterial bypass
(B) Distal venous arterialization bypass
(C) Endovascular stent placement
(D) Free muscle transfer
(E) Lumbar sympathectomy

A

The correct response is Option B.

The symptoms and findings of severe pain at rest, ulceration, and gangrene seen in this patient are indicative of limb ischemia, a critically urgent condition that results from occlusion of the pedal and crural arteries. Although amputation had been performed in the past for patients with this condition, distal venous arterialization bypass is now a recommended alternative method for limb salvage. According to one small study of 18 patients, limb salvage was successfully accomplished in 83% of patients at surgery and 75% of patients at follow-up examination one year later. In these patients, the distal bypass was performed to the venous vessels of the foot using a conduit of either vein, synthetic graft, or a combination of both. The valves of the venous system were destroyed, and arterial inflow was then provided by the most distal patent artery.

Because both the pedal and crural arteries are occluded in this patient, neither direct arterial bypass nor placement of an endovascular stent will address the problem. Both free muscle transfer and lumbar sympathectomy are associated with lower rates of limb salvage in patients with severe ischemia.

164
Q

A 35-year-old man with insulin-dependent diabetes mellitus develops a 14 * 8-cm ulcer of the left posterior calf overlying the Achilles tendon. He underwent kidney transplantation two years ago and has been taking immunosuppressive agents since that time.

Which of the following is the most appropriate management?

(A) Growth factor therapy
(B) Skin grafting
(C) Free tissue transfer
(D) Syme’s amputation
(E) Below-knee amputation

A

The correct response is Option C.

This 35-year-old man with diabetes mellitus who previously underwent kidney transplantation has a large ulcer overlying the left Achilles tendon. Because of his immunosuppressive state and concomitant diabetes mellitus, he is most likely to experience cardiovascular problems, neuropathy, and poor wound healing following any type of reconstructive surgery. However, despite these potential adverse sequelae, free tissue transfer is most appropriate for management of the ulcer. Appropriate work-up should be obtained before attempting any reconstructive procedure.

Growth factor therapy will not adequately heal this large ulcer. A skin graft alone is inappropriate for coverage of exposed tendons. Because amputations often increase cardiovascular demand in patients with diabetes mellitus, they should only be used as a last resort. An amputation is also inappropriate in a young patient because it will significantly alter his ability to perform activities of daily living. In addition, the Syme’s amputation is used in wound management of the forefoot and not the distal calf.

165
Q
A